pn2 final 18,19,20,64,41-44,46-48 55, 56,57, 66, 11, 12,13, 27, 28,29, 30, 31, 35,36,33,34,39,40

Réussis tes devoirs et examens dès maintenant avec Quizwiz!

A nurse is teaching a client about ear hygiene and health. What client statement indicates a need for further teaching?

*a. A soft cotton swab is alright to clean my ears with*. b. I make sure my ears are dry after I go swimming. c. I use good earplugs when I practice with the band. d. Keeping my diabetes under control helps my ears. ANS: A Clients should be taught not to put anything larger than their fingertip into their ears. Using a cotton swab, although soft, can cause damage to the ears and cerumen buildup. The other statements are accurate.

A nurse in the family clinic is teaching a client newly diagnosed with osteoarthritis (OA) about drugs used to treat the disease. For which medication does the nurse plan primary teaching?

*a. Acetaminophen (Tylenol)* b. Cyclobenzaprine hydrochloride (Flexeril) c. Hyaluronate (Hyalgan) d. Ibuprofen (Motrin) ANS: A All of the drugs are appropriate to treat OA. However, the first-line drug is acetaminophen. Cyclobenzaprine is a muscle relaxant given to treat muscle spasms. Hyaluronate is a synthetic joint fluid implant. Ibuprofen is a nonsteroidal anti-inflammatory drug.

A nurse assesses a client with diabetes mellitus and notes the client only responds to a sternal rub by moaning, has capillary blood glucose of 33 g/dL, and has an intravenous line that is infiltrated with 0.45% normal saline. Which action should the nurse take first?

*a. Administer 1 mg of intramuscular glucagon*. b.Encourage the client to drink orange juice. c. Insert a new intravenous access line. d.Administer 25 mL dextrose 50% (D50) IV push. ANS: A The clients blood glucose level is dangerously low. The nurse needs to administer glucagon IM immediately to increase the clients blood glucose level. The nurse should insert a new IV after administering the glucagon and can use the new IV site for future doses of D50 if the clients blood glucose level does not rise. Once the client is awake, orange juice may be administered orally along with a form of protein such as a peanut butter.

A nurse assesses a client who has diabetes mellitus and notes the client is awake and alert, but shaky, diaphoretic, and weak. Five minutes after administering a half-cup of orange juice, the clients clinical manifestations have not changed. Which action should the nurse take next?

*a. Administer another half-cup of orange juice*. b.Administer a half-ampule of dextrose 50% intravenously. c. Administer 10 units of regular insulin subcutaneously. d.Administer 1 mg of glucagon intramuscularly. ANS: A This client is experiencing mild hypoglycemia. For mild hypoglycemic manifestations, the nurse should administer oral glucose in the form of orange juice. If the symptoms do not resolve immediately, the treatment should be repeated. The client does not need intravenous dextrose, insulin, or glucagon.

What action by the perioperative nursing staff is most important to prevent surgical wound infection in a client having a total joint replacement?

*a. Administer preoperative antibiotic as ordered.* b. Assess the clients white blood cell count. c. Instruct the client to shower the night before. d. Monitor the clients temperature postoperatively. ANS: A To prevent surgical wound infection, antibiotics are given preoperatively within an hour of surgery. Simply taking a shower will not help prevent infection unless the client is told to use special antimicrobial soap. The other options are processes to monitor for infection, not prevent it.

A client has a foreign body in the eye. What action by the nurse takes priority?

*a. Administering ordered antibiotics* b. Assessing the clients visual acuity c. Obtaining consent for enucleation d. Removing the object immediately ANS: A To prevent infection, antibiotics are provided. Visual acuity in the affected eye cannot be assessed. The client may or may not need enucleation. The object is only removed by the ophthalmologist.

A client suffered an episode of anaphylaxis and has been stabilized in the intensive care unit. When assessing the clients lungs, the nurse hears the following sounds. What medication does the nurse prepare to administer? (Click the media button to hear the audio clip.)

*a. Albuterol (Proventil) via nebulizer* b. Diphenhydramine (Benadryl) IM c. Epinephrine 1:10,000 5 mg IV push d. Methylprednisolone (Solu-Medrol) IV push ANS: A The nurse has auscultated wheezing in the clients lungs and prepares to administer albuterol, which is a bronchodilator, or assists respiratory therapy with administration. Diphenhydramine is an antihistamine. Epinephrine is given during an acute crisis in a concentration of 1:1000. Methylprednisolone is a corticosteroid.

A nurse delegates care for a client with Parkinson disease to an unlicensed assistive personnel (UAP). Which statement should the nurse include when delegating this clients care?

*a. Allow the client to be as independent as possible with activities*. b. Assist the client with frequent and meticulous oral care. c. Assess the clients ability to eat and swallow before each meal. d. Schedule appointments early in the morning to ensure rest in the afternoon. ANS: A Clients with Parkinson disease do not move as quickly and can have functional problems. The client should be encouraged to be as independent as possible and provided time to perform activities without rushing. Although oral care is important for all clients, instructing the UAP to provide frequent and meticulous oral is not a priority for this client. This statement would be a priority if the client was immune-compromised or NPO. The nurse should assess the clients ability to eat and swallow; this should not be delegated. Appointments and activities should not be scheduled early in the morning because this may cause the client to be rushed and discourage the client from wanting to participate in activities of daily living.

A nurse is teaching a community group about preventing hearing loss. What instruction is best?

*a. Always wear a bicycle helmet*. b. Avoid swimming in ponds or lakes. c. Dont go to fireworks displays. d. Use a soft cotton swab to clean ears. ANS: A Avoiding head trauma is a practical way to help prevent hearing loss. Swimming can lead to hearing loss if the client has repeated infections. Fireworks displays are loud, but usually brief and only occasional. Nothing smaller than the clients fingertip should be placed in the ear canal.

The nurse is presenting information to a community group on safer sex practices. The nurse should teach that which sexual practice is the riskiest?

*a. Anal intercourse* b. Masturbation c. Oral sex d. Vaginal intercourse ANS: A Anal intercourse is the riskiest sexual practice because the fragile anal tissue can tear, creating a portal of entry for human immune deficiency virus.

A client calls the clinic to report exposure to poison ivy and an itchy rash that is not helped with over-thecounter antihistamines. What response by the nurse is most appropriate?

*a. Antihistamines do not help poison ivy*. b. There are different antihistamines to try. c. You should be seen in the clinic right away. d. You will need to take some IV steroids. ANS: A Since histamine is not the mediator of a type IV reaction such as with poison ivy, antihistamines will not provide relief. The nurse should educate the client about this. The client does not need to be seen right away. The client may or may not need steroids; they may be given either IV or orally.

A client has trigeminal neuralgia and has begun skipping meals and not brushing his teeth, and his family believes he has become depressed. What action by the nurse is best?

*a. Ask the client to explain his feelings related to this disorder*. b. Explain how dental hygiene is related to overall health. c. Refer the client to a medical social worker for assessment. d. Tell the client that he will become malnourished in time. ANS: A Clients with trigeminal neuralgia are often afraid of causing pain, so they may limit eating, talking, dental hygiene, and socializing. The nurse first assesses the client for feelings related to having the disorder to determine if a psychosocial link is involved. The other options may be needed depending on the outcome of the initial assessment.

An older adult in the family practice clinic reports a decrease in hearing over a week. What action by the nurse is most appropriate?

*a. Assess for cerumen buildup*. b. Facilitate audiological testing. c. Perform tuning fork tests. d. Review the medication list. ANS: A All options are possible actions for the client with hearing loss. The first action the nurse should take is to look for cerumen buildup, which can decrease hearing in the older adult. If this is normal, medications should be assessed for ototoxicity. Further auditory testing may be needed for this client.

A client who has had systemic lupus erythematosus (SLE) for many years is in the clinic reporting hip pain with ambulation. Which action by the nurse is best?

*a. Assess medication records for steroid use.* b. Facilitate a consultation with physical therapy. c. Measure the range of motion in both hips. d. Notify the health care provider immediately ANS: A Chronic steroid use is seen in clients with SLE and can lead to osteonecrosis (bone necrosis). The nurse should determine if the client has been taking a steroid. Physical therapy may be beneficial, but there is not enough information about the client yet. Measuring range of motion is best done by the physical therapist. Notifying the provider immediately is not warranted

The nurse on the postoperative inpatient unit assesses a client after a total hip replacement. The clients surgical leg is visibly shorter than the other one and the client reports extreme pain. While a co-worker calls the surgeon, what action by the nurse is best?

*a. Assess neurovascular status in both legs* b. Elevate the affected leg and apply ice. c. Prepare to administer pain medication. d. Try to place the affected leg in abduction ANS: A This client has manifestations of hip dislocation, a critical complication of this surgery. Hip dislocation can cause neurovascular compromise. The nurse should assess neurovascular status, comparing both legs. The nurse should not try to move the extremity to elevate or abduct it. Pain medication may be administered if possible, but first the nurse should thoroughly assess the client.

A client has just been diagnosed with human immune deficiency virus (HIV). The client is distraught and does not know what to do. What intervention by the nurse is best?

*a. Assess the client for support systems*. b. Determine if a clergy member would help. c. Explain legal requirements to tell sex partners. d. Offer to tell the family for the client. ANS: A This client needs the assistance of support systems. The nurse should help the client identify them and what role they can play in supporting him or her. A clergy member may or may not be welcome. Legal requirements about disclosing HIV status vary by state. Telling the family for the client is enabling, and the client may not want the family to know.

A client in the orthopedic clinic has a self-reported history of osteoarthritis. The client reports a low-grade fever that started when the weather changed and several joints started acting up, especially both hips and knees. What action by the nurse is best?

*a. Assess the client for the presence of subcutaneous nodules or Bakers cysts*. b. Inspect the clients feet and hands for podagra and tophi on fingers and toes. c. Prepare to teach the client about an acetaminophen (Tylenol) regimen. d. Reassure the client that the problems will fade as the weather changes again ANS: A Osteoarthritis is not a systemic disease, nor does it present bilaterally. These are manifestations of rheumatoid arthritis. The nurse should assess for other manifestations of this disorder, including subcutaneous nodules and Bakers cysts. Podagra and tophi are seen in gout. Acetaminophen is not used for rheumatoid arthritis. Telling the client that the symptoms will fade with weather changes is not accurate.

A client has been diagnosed with rheumatoid arthritis. The client has experienced increased fatigue and worsening physical status and is finding it difficult to maintain the role of elder in his cultural community. The elder is expected to attend social events and make community decisions. Stress seems to exacerbate the condition. What action by the nurse is best?

*a. Assess the clients culture more thoroughly.* b. Discuss options for performing duties. c. See if the client will call a community meeting. d. Suggest the client give up the role of elder. ANS: A The nurse needs a more thorough understanding of the clients culture, including the meaning of illness and the ramifications of the elder not being able to perform traditional duties. This must be done prior to offering any possible solutions. If the nurse does not understand the consequences of what is suggested, the client may simply be unwilling to listen or participate in problem solving. The other options may be reasonable depending on the outcome of a better cultural understanding.

An older client is hospitalized with Guillain-Barr syndrome. A family member tells the nurse the client is restless and seems confused. What action by the nurse is best?

*a. Assess the clients oxygen saturation*. b. Check the medication list for interactions. c. Place the client on a bed alarm. d. Put the client on safety precautions. ANS: A In the older adult, an early sign of hypoxia is often confusion and restlessness. The nurse should first assess the clients oxygen saturation. The other actions are appropriate, but only after this assessment occurs.

A nurse is talking with a client about a negative enzyme-linked immunosorbent assay (ELISA) test for human immune deficiency virus (HIV) antibodies. The test is negative and the client states Whew! I was really worried about that result. What action by the nurse is most important?

*a. Assess the clients sexual activity and patterns*. b. Express happiness over the test result. c. Remind the client about safer sex practices. d. Tell the client to be retested in 3 months. ANS: A The ELISA test can be falsely negative if testing occurs after the client has become infected but prior to making antibodies to HIV. This period of time is known as the window period and can last up to 36 months. The nurse needs to assess the clients sexual behavior further to determine the proper response. The other actions are not the most important, but discussing safer sex practices is always appropriate.

A client hospitalized for a wound infection has a blood urea nitrogen of 45 mg/dL and creatinine of 4.2 mg/dL. What action by the nurse is best

*a. Assess the ordered antibiotics for ototoxicity*. b. Explain how kidney damage causes hearing loss. c. Use ibuprofen (Motrin) for pain control. d. Teach that hearing loss is temporary. ANS: A Some medications are known to be ototoxic. Diminished kidney function slows the excretion of drugs from the body, worsening the ototoxic effects. The nurse should assess the antibiotics the client is receiving for ototoxicity. The other options are not warranted.

The nurse is working with a client who has rheumatoid arthritis (RA). The nurse has identified the priority problem of poor body image for the client. What finding by the nurse indicates goals for this client problem are being met?

*a. Attends meetings of a book club* b. Has a positive outlook on life c. Takes medication as directed d. Uses assistive devices to protect joints ANS: A All of the activities are appropriate for a client with RA. Clients who have a poor body image are often reluctant to appear in public, so attending public book club meetings indicates that goals for this client problem are being met.

A nurse assesses a client who is recovering from anterior cervical diskectomy and fusion. Which complication should alert the nurse to urgently communicate with the health care provider?

*a. Auscultated stridor* b. Weak pedal pulses c. Difficulty swallowing d. Inability to shrug shoulders ANS: A Postoperative swelling can narrow the trachea, cause a partial airway obstruction, and manifest as stridor. The client may also have trouble swallowing, but maintaining an airway takes priority. Weak pedal pulses and an inability to shrug the shoulders are not complications of this surgery.

A nurse teaches a client who is scheduled for a positron emission tomography scan of the brain. Which statement should the nurse include in this clients teaching?

*a. Avoid caffeine-containing substances for 12 hours before the test*. b. Drink at least 3 liters of fluid during the first 24 hours after the test. c. Do not take your cardiac medication the morning of the test. d. Remove your dentures and any metal before the test begins. ANS: A Caffeine-containing liquids and foods are central nervous system stimulants and may alter the test results. No contrast is used; therefore, the client does not need to increase fluid intake. The client should take cardiac medications as prescribed. Metal does not have to be removed; this is done for magnetic resonance imaging.

A client is taking long-term corticosteroids for myasthenia gravis. What teaching is most important?

*a. Avoid large crowds and people who are ill*. b. Check blood sugars four times a day. c. Use two forms of contraception. d. Wear properly fitting socks and shoes. ANS: A Corticosteroids reduce immune function, so clients taking these medications must avoid being exposed to illness. Long-term use can lead to secondary diabetes, but the client would not need to start checking blood glucose unless diabetes had been detected. Corticosteroids do not affect the effectiveness of contraception. Wearing well-fitting shoes would be important to avoid injury, but not just because the client takes corticosteroids.

23. A nurse is teaching a client with psoriatic arthritis about the medication golimumab (Simponi). What information is most important to include?

*a. Avoid large crowds or people who are ill.* b. Stay upright for 1 hour after taking this drug. c. This drug may cause your hair to fall out. d. You may double the dose if pain is severe. ANS: A This drug has a Food and Drug Administration black box warning about opportunistic or other serious infections. Teach the client to avoid large crowds and people who are ill. The other instructions do not pertain to golimumab

A client had a retinal detachment and has undergone surgical correction. What discharge instruction is most important?

*a. Avoid reading, writing, or close work such as sewing*. b. Dim the lights in your house for at least a week. c. Keep the follow-up appointment with the ophthalmologist. d. Remove your eye patch every hour for eyedrops. ANS: A After surgery for retinal detachment, the client is advised to avoid reading, writing, and close work because they cause rapid eye movements. Dim lights are not indicated. Keeping a postoperative appointment is important for any surgical client. The eye patch is not removed for eyedrops.

A client does not understand why vision loss due to glaucoma is irreversible. What explanation by the nurse is best?

*a. Because eye pressure was too high, the tissue died*. b. Glaucoma always leads to permanent blindness. c. The traumatic damage to your eye was too great. d. The infection occurs so quickly it cant be treated. ANS: A Glaucoma is caused when the intraocular pressure becomes too high and stays high long enough to cause tissue ischemia and death. At that point, vision loss is permanent. Glaucoma does not have to cause blindness. Trauma can cause glaucoma but is not the most common cause. Glaucoma is not an infection.

The nurse reads on a clients chart that the client has exophthalmos. What assessment finding is consistent with this diagnosis?

*a. Bulging eyes* b. Drooping eyelids c. Sunken-in eyes d. Yellow sclera ANS: A Exophthalmos is bulging eyes. Drooping eyelids is ptosis. Sunken-in eyes is enophthalmos. Yellow sclera indicates jaundice

A client had a myringotomy. The nurse provides which discharge teaching?

*a. Buy dry shampoo to use for a week*. b. Drink liquids through a straw. c. Flying is not allowed for 1 month. d. Hot water showers will help the pain. ANS: A The client cannot shower or get the head wet for 1 week after surgery, so using dry shampoo is a good suggestion. The other instructions are incorrect: straws are not allowed for 2 to 3 weeks, flying is not allowed for 2 to 3 weeks, and the client should not shower.

A nurse works in an allergy clinic. What task performed by the nurse takes priority?

*a. Checking emergency equipment each morning* b. Ensuring informed consent is obtained as needed c. Providing educational materials in several languages d. Teaching clients how to manage their allergies ANS: A All actions are appropriate for this nurse; however, client safety is the priority. The nurse should ensure that emergency equipment is available and in good working order and that sufficient supplies of emergency medications are on hand as the priority responsibility. When it is appropriate for a client to give informed consent, the nurse ensures the signed forms are on the chart. Providing educational materials in several languages is consistent with holistic care. Teaching is always a major responsibility of all nurses.

A client has a corneal ulcer. What information provided by the client most indicates a potential barrier to home care?

*a. Chronic use of sleeping pills* b. Impaired near vision c. Slightly shaking hands d. Use of contact lenses ANS: A Antibiotic eyedrops are often needed every hour for the first 24 hours for corneal ulceration. The client who uses sleeping pills may not wake up each hour or may awaken unable to perform this task. This client might need someone else to instill the eyedrops hourly. Impaired near vision and shaking hands can both make administration of eyedrops more difficult but are not the most likely barriers. Contact lenses should be discarded.

A nurse is caring for four clients who have immune disorders. After receiving the hand-off report, which client should the nurse assess first?

*a. Client with acquired immune deficiency syndrome with a CD4+ cell count of 210/mm3 and a temp of 102.4 F (39.1 C)* b. Client with Brutons agammaglobulinemia who is waiting for discharge teaching c. Client with hypogammaglobulinemia who is 1 hour post immune serum globulin infusion d. Client with selective immunoglobulin A deficiency who is on IV antibiotics for pneumonia ANS: A A client who is this immunosuppressed and who has this high of a fever is critically ill and needs to be assessed first. The client who is post immunoglobulin infusion should have had all infusion-related vital signs and assessments completed and should be checked next. The client receiving antibiotics should be seen third, and the client waiting for discharge teaching is the lowest priority. Since discharge teaching can take time, the nurse may want to delegate this task to someone else while attending to the most seriously ill client.

The nurse enters an examination room to help with an eye examination. The client is directed toward the assessment chart shown below: What is the provider assessing?

*a. Color vision* b. Depth perception c. Spatial perception d. Visual acuity ANS: A This is an Ishihara chart, which is used for assessing color vision. Depth and spatial perception are not typically assessed in a routine vision assessment. Visual acuity is usually tested with a Snellen chart.

A nurse works on a unit that has admitted its first client with acquired immune deficiency syndrome. The nurse overhears other staff members talking about the AIDS guy and wondering how the client contracted the disease. What action by the nurse is best?

*a. Confront the staff members about unethical behavior*. b. Ignore the behavior; they will stop on their own soon. c. Report the behavior to the units nursing management. d. Tell the client that other staff members are talking about him or her. ANS: A The professional nurse should be able to confront unethical behavior assertively. The staff should not be talking about clients unless they have a need to do so for client care. Ignoring the behavior may be more comfortable, but the nurse is abdicating responsibility. The behavior may need to be reported, but not as a first step. Telling the client that others are talking about him or her does not accomplish anything.

The nurse providing direct client care uses specific practices to reduce the chance of acquiring infection with human immune deficiency virus (HIV) from clients. Which practice is most effective?

*a. Consistent use of Standard Precautions* b. Double-gloving before body fluid exposure c. Labeling charts and armbands HIV+ d. Wearing a mask within 3 feet of the client ANS: A According to The Joint Commission, the most effective preventative measure to avoid HIV exposure is consistent use of Standard Precautions. Double-gloving is not necessary. Labeling charts and armbands in this fashion is a violation of the Health Information Portability and Accountability Act (HIPAA). Wearing a mask within 3 feet of the client is part of Airborne Precautions and is not necessary with every client contact.

A client takes celecoxib (Celebrex) for chronic osteoarthritis in multiple joints. After a knee replacement, the health care provider has prescribed morphine sulfate for postoperative pain relief. The client also requests the celecoxib in addition to the morphine. What action by the nurse is best?

*a. Consult with the health care provider about administering both drugs to the client*. b. Inform the client that the celecoxib will be started when he or she goes home. c. Teach the client that, since morphine is stronger, celecoxib is not needed. d. Tell the client he or she should not take both drugs at the same time. ANS: A Despite getting an opioid analgesic for postoperative pain, the nurse should be aware that the client may be on other medications for arthritis in other joints. The nonsteroidal anti-inflammatory drug celecoxib will also help with the postoperative pain. The nurse should consult the provider about continuing the celecoxib while the client is in the hospital. The other responses are not warranted, as the client should be restarted on this medication postoperatively.

A client with human immune deficiency virus infection is hospitalized for an unrelated condition, and several medications are prescribed in addition to the regimen already being used. What action by the nurse is most important?

*a. Consult with the pharmacy about drug interactions*. b. Ensure that the client understands the new medications. c. Give the new drugs without considering the old ones. d. Schedule all medications at standard times. ANS: A The drug regimen for someone with HIV/AIDS is complex and consists of many medications that must be given at specific times of the day, and that have many interactions with other drugs. The nurse should consult with a pharmacist about possible interactions. Client teaching is important but does not take priority over ensuring the medications do not interfere with each other, which could lead to drug resistance or a resurgence of symptoms.

A nurse assesses the left plantar reflexes of an adult client and notes the response shown in the photograph below: Which action should the nurse take next?

*a. Contact the provider with this abnormal finding*. b. Assess bilateral legs for temperature and edema. c. Ask the client about pain in the lower leg and calf. d. Document the finding and continue the assessment. ANS: A This finding indicates Babinskis sign. In clients older than 2 years of age, Babinskis sign is considered abnormal and indicates central nervous system disease. The nurse should notify the health care provider and other members of the health care team because further investigation is warranted. This finding does not relate to perfusion of the leg or to pain. This is an abnormal assessment finding and should be addressed immediately

A client has been hospitalized with an opportunistic infection secondary to acquired immune deficiency syndrome. The clients partner is listed as the emergency contact, but the clients mother insists that she should be listed instead. What action by the nurse is best?

*a. Contact the social worker to assist the client with advance directives*. b. Ignore the mother; the client does not want her to be involved. c. Let the client know, gently, that nurses cannot be involved in these disputes. d. Tell the client that, legally, the mother is the emergency contact. ANS: A The client should make his or her wishes known and formalize them through advance directives. The nurse should help the client by contacting someone to help with this process. Ignoring the mother or telling the client that nurses cannot be involved does not help the situation. Legal statutes vary by state; as more states recognize gay marriage, this issue will continue to evolve.

A client presents to the emergency department reporting a foreign body in the eye. For what diagnostic testing should the nurse prepare the client?

*a. Corneal staining* b. Fluorescein angiography c. Ophthalmoscopy d. Tonometry ANS: A Corneal staining is used when the possibility of eye trauma exists, including a foreign body. Fluorescein angiography is used to assess problems of retinal circulation. Ophthalmoscopy looks at both internal and external eye structures. Tonometry tests the intraocular pressure.

The nurse is caring for a client diagnosed with human immune deficiency virus. The clients CD4+ cell count is 399/mm3. What action by the nurse is best?

*a. Counsel the client on safer sex practices/abstinence*. b. Encourage the client to abstain from alcohol. c. Facilitate genetic testing for CD4+ CCR5/CXCR4 co-receptors. d. Help the client plan high-protein/iron meals. ANS: A This client is in the Centers for Disease Control and Prevention stage 2 case definition group. He or she remains highly infectious and should be counseled on either safer sex practices or abstinence. Abstaining from alcohol is healthy but not required. Genetic testing is not commonly done, but an alteration on the CCR5/CXCR4 co-receptors is seen in long-term nonprogressors. High-protein/iron meals are important for people who are immunosuppressed, but helping to plan them does not take priority over stopping the spread of the disease.

The nurse on an inpatient rheumatology unit receives a hand-off report on a client with an acute exacerbation of systemic lupus erythematosus (SLE). Which reported laboratory value requires the nurse to assess the client further?

*a. Creatinine: 3.9 mg/dL* b. Platelet count: 210,000/mm3 c. Red blood cell count: 5.2/mm3 d. White blood cell count: 4400/mm3 ANS: A Lupus nephritis is the leading cause of death in clients with SLE. The creatinine level is very high and the nurse needs to perform further assessments related to this finding. The other laboratory values are normal

A nurse assesses a client and notes the clients position as indicated in the illustration below: How should the nurse document this finding?

*a. Decorticate posturing* b. Decerebrate posturing c. Atypical hyperreflexia d. Spinal cord degeneration ANS: A The client is demonstrating decorticate posturing, which is seen with interruption in the corticospinal pathway. This finding is abnormal and is a sign that the clients condition has deteriorated. The physician, the charge nurse, and other health care team members should be notified immediately of this change in status. Decerebrate posturing consists of external rotation and extension of the extremities. Hyperreflexes present as increased reflex responses. Spinal cord degeneration presents frequently with pain and discomfort.

A nurse is caring for a client with systemic sclerosis. The clients facial skin is very taut, limiting the clients ability to open the mouth. After consulting with a registered dietitian for appropriate nutrition, what other consultation should the nurse facilitate?

*a. Dentist* b. Massage therapist c. Occupational therapy d. Physical therapy ANS: A With limited ability to open the mouth, dental hygiene may be lacking. The nurse should encourage the client to see a dentist. The other referrals are not related to the mouth.

A clients chart indicates anisocoria. For what should the nurse assess?

*a. Difference in pupil size* b. Draining infection c. Recent eye trauma d. Tumor of the eyelid ANS: A Anisocoria is a noticeable difference in the size of a persons pupils. This is a normal finding in a small percentage of the population. Infection, trauma, and tumors are not related.

A nurse assesses a client who demonstrates a positive Rombergs sign with eyes closed but not with eyes open. Which condition does the nurse associate with this finding?

*a. Difficulty with proprioception* b. Peripheral motor disorder c. Impaired cerebellar function d. Positive pronator drift ANS: A The client who sways with eyes closed (positive Rombergs sign) but not with eyes open most likely has a disorder of proprioception and uses vision to compensate for it. The other options do not describe a positive Rombergs sign

A client had proxymetacaine (Ocu-Caine) instilled in one eye in the emergency department. What discharge instruction is most important?

*a. Do not touch or rub the eye until it is no longer numb*. b. Monitor the eye for any bleeding for the next day. c. Rinse the eye with warm saline solution at home. d. Use all the eyedrops as prescribed until they are gone. ANS: A This drug is an ophthalmic anesthetic. The client can injure the numb eye by touching or rubbing it. Bleeding is not associated with this drug. The client should not be told to rinse the eye. This medication was given in the emergency department and is not prescribed for home use.

A nurse obtains a focused health history for a client who is suspected of having bacterial meningitis. Which question should the nurse ask?

*a. Do you live in a crowded residence*? b. When was your last tetanus vaccination? c. Have you had any viral infections recently? d. Have you traveled out of the country in the last month? ANS: A Meningococcal meningitis tends to occur in multiple outbreaks. It is most likely to occur in areas of highdensity population, such as college dormitories, prisons, and military barracks. A tetanus vaccination would not place the client at increased risk for meningitis or protect the client from meningitis. A viral infection would not lead to bacterial meningitis but could lead to viral meningitis. Simply knowing if the client traveled out of the country does not provide enough information. The nurse should ask about travel to specific countries in which the disease is common, for example, sub-Saharan Africa.

A preoperative nurse assesses a client who has type 1 diabetes mellitus prior to a surgical procedure. The clients blood glucose level is 160 mg/dL. Which action should the nurse take?

*a. Document the finding in the clients chart*. b.Administer a bolus of regular insulin IV. c. Call the surgeon to cancel the procedure. d.Draw blood gases to assess the metabolic state ANS: A Clients who have type 1 diabetes and are having surgery have been found to have fewer complications, lower rates of infection, and better wound healing if blood glucose levels are maintained at between 140 and 180 mg/dL throughout the perioperative period. The nurse should document the finding and proceed with other operative care. The need for a bolus of insulin, canceling the procedure, or drawing arterial blood gases is not required.

.A nurse prepares to administer insulin to a client at 1800. The clients medication administration record contains the following information: Insulin glargine: 12 units daily at 1800 Regular insulin: 6 units QID at 0600, 1200, 1800, 2400 Based on the clients medication administration record, which action should the nurse take?

*a. Draw up and inject the insulin glargine first, and then draw up and inject the regular insulin*. b.Draw up and inject the insulin glargine first, wait 20 minutes, and then draw up and inject the regular insulin. c. First draw up the dose of regular insulin, then draw up the dose of insulin glargine in the same syringe, mix, and inject the two insulins together. d.First draw up the dose of insulin glargine, then draw up the dose of regular insulin in the same syringe, mix, and inject the two insulins together. ANS: A Insulin glargine must not be diluted or mixed with any other insulin or solution. Mixing results in an unpredictable alteration in the onset of action and time to peak action. The correct instruction is to draw up and inject first the glargine and then the regular insulin right afterward.

The nurse is teaching a client with gout dietary strategies to prevent exacerbations or other problems. Which statement by the nurse is most appropriate?

*a. Drink 1 to 2 liters of water each day.* b. Have 10 to 12 ounces of juice a day. c. Liver is a good source of iron. d. Never eat hard cheeses or sardines. ANS: A Kidney stones are common in clients with gout, so drinking plenty of water will help prevent this from occurring. Citrus juice is high in ash, which can help prevent the formation of stones, but the value of this recommendation is not clear. Clients with gout should not eat organ meats or fish with bones, such as sardines

A client is scheduled to have a tumor of the middle ear removed. What teaching topic is most important for the nurse to cover?

*a. Expecting hearing loss in the affected ear* b. Managing postoperative pain c. Maintaining NPO status prior to surgery d. Understanding which medications are allowed the day of surgery ANS: A Removal of an inner ear tumor will likely destroy hearing in the affected ear. The other teaching topics are appropriate for any surgical client

A client is started on etanercept (Enbrel). What teaching by the nurse is most appropriate?

*a. Giving subcutaneous injections* b. Having a chest x-ray once a year c. Taking the medication with food d. Using heat on the injection site ANS: A Etanercept is given as a subcutaneous injection twice a week. The nurse should teach the client how to selfadminister the medication. The other options are not appropriate for etanercept.

A client is getting out of bed into the chair for the first time after an uncemented hip replacement. What action by the nurse is most important?

*a. Have adequate help to transfer the client*. b. Provide socks so the client can slide easier. c. Tell the client full weight bearing is allowed. d. Use a footstool to elevate the clients leg. ANS: A The client with an uncemented hip will be on toe-touch only right after surgery. The nurse should ensure there is adequate help to transfer the client while preventing falls. Slippery socks will encourage a fall. Elevating the leg greater than 90 degrees is not allowed.

The nurse has given a community group a presentation on eye health. Which statement by a participant indicates a need for more instruction?

*a. I always lose my sunglasses, so I dont wear them*. b. I have diabetes and get an annual eye exam. c. I will not share my contact solution with others. d. I will wear safety glasses when I mow the lawn ANS: A Clients should be taught to protect their eyes from ultraviolet (UV) exposure by consistently wearing sunglasses when outdoors, when tanning in tanning salons, or when working with UV light. The other statements are correct

A client recently diagnosed with systemic lupus erythematosus (SLE) is in the clinic for a follow-up visit. The nurse evaluates that the client practices good self-care when the client makes which statement?

*a. I always wear long sleeves, pants, and a hat when outdoors*. b. I try not to use cosmetics that contain any type of sunblock. c. Since I tend to sweat a lot, I use a lot of baby powder. d. Since I cant be exposed to the sun, I have been using a tanning bed. ANS: A Good self-management of the skin in SLE includes protecting the skin from sun exposure, using sunblock, avoiding drying agents such as powder, and avoiding tanning beds

A client had a nerve laceration repair to the forearm and is being discharged in a cast. What statement by the client indicates a poor understanding of discharge instructions relating to cast care?

*a. I can scratch with a coat hanger*. b. I should feel my fingers for warmth. c. I will keep the cast clean and dry. d. I will return to have the cast removed. ANS: A Nothing should be placed under the cast to use for scratching. The other statements show good indication that the client has understood the discharge instructions.

A nurse has educated a client on an epinephrine auto-injector (EpiPen). What statement by the client indicates additional instruction is needed?

*a. I dont need to go to the hospital after using it*. b. I must carry two EpiPens with me at all times. c. I will write the expiration date on my calendar. d. This can be injected right through my clothes. ANS: A Clients should be instructed to call 911 and go to the hospital for monitoring after using the EpiPen. The other statements show good understanding of this treatment.

A nurse witnesses a client with late-stage Alzheimers disease eat breakfast. Afterward the client states, I am hungry and want breakfast. How should the nurse respond?

*a. I see you are still hungry. I will get you some toast*. b. You ate your breakfast 30 minutes ago. c. It appears you are confused this morning. d. Your family will be here soon. Lets get you dressed. ANS: A Use of validation therapy with clients who have Alzheimers disease involves acknowledgment of the clients feelings and concerns. This technique has proved more effective in later stages of the disease, when using reality orientation only increases agitation. Telling the client that he or she already ate breakfast may agitate the client. The other statements do not validate the clients concerns.

.After teaching a client who is recovering from pancreas transplantation, the nurse assesses the clients understanding. Which statement made by the client indicates a need for additional education?

*a. If I develop an infection, I should stop taking my corticosteroid*. b.If I have pain over the transplant site, I will call the surgeon immediately. c. I should avoid people who are ill or who have an infection. d.I should take my cyclosporine exactly the way I was taught. ANS: A Immunosuppressive agents should not be stopped without the consultation of the transplantation physician, even if an infection is present. Stopping immunosuppressive therapy endangers the transplanted organ. The other statements are correct. Pain over the graft site may indicate rejection. Anti-rejection drugs cause immunosuppression, and the client should avoid crowds and people who are ill. Changing the routine of antirejection medications may cause them to not work optimally.

After teaching a client with a spinal cord injury, the nurse assesses the clients understanding. Which client statement indicates a correct understanding of how to prevent respiratory problems at home?

*a. Ill use my incentive spirometer every 2 hours while Im awake*. b. Ill drink thinned fluids to prevent choking. c. Ill take cough medicine to prevent excessive coughing. d. Ill position myself on my right side so I dont aspirate. ANS: A Often, the person with a spinal cord injury will have weak intercostal muscles and is at higher risk for developing atelectasis and stasis pneumonia. Using an incentive spirometer every 2 hours helps the client expand the lungs more fully and prevents atelectasis. Clients should drink fluids that they can tolerate; usually thick fluids are easier to tolerate. The client should be encouraged to cough and clear secretions. Clients should be placed in high-Fowlers position to prevent aspiration.

A client has labyrinthitis and is prescribed antibiotics. What instruction by the nurse is most important for this client?

*a. Immediately report headache or stiff neck*. b. Keep all follow-up appointments. c. Take the antibiotics with a full glass of water. d. Take the antibiotic on an empty stomach. ANS: A Meningitis is a complication of labyrinthitis. The client should be taught to take all antibiotics as prescribed and to report manifestations of meningitis such as fever, headache, or stiff neck. Keeping follow-up appointments is important for all clients. Without knowing what antibiotic was prescribed, the nurse cannot instruct the client on how to take it.

An emergency department nurse assesses a client with ketoacidosis. Which clinical manifestation should the nurse correlate with this condition?

*a. Increased rate and depth of respiration* b.Extremity tremors followed by seizure activity c. Oral temperature of 102 F (38.9 C) d.Severe orthostatic hypotension ANS: A Ketoacidosis decreases the pH of the blood, stimulating the respiratory control areas of the brain to buffer the effects of increasing acidosis. The rate and depth of respiration are increased (Kussmaul respirations) in an attempt to excrete more acids by exhalation. Tremors, elevated temperature, and orthostatic hypotension are not associated with ketoacidosis.

A client comes to the family medicine clinic and reports joint pain and stiffness. The nurse is asked to assess the client for Heberdens nodules. What assessment technique is correct?

*a. Inspect the clients distal finger joints*. b. Palpate the clients abdomen for tenderness. c. Palpate the clients upper body lymph nodes. d. Perform range of motion on the clients wrists. ANS: A Herberdens nodules are seen in osteoarthritis and are bony nodules at the distal interphalangeal joints. To assess for this finding, the nurse inspects the clients distal fingertips. These nodules are not found in the abdomen, lymph nodes, or wrists.

A client with myasthenia gravis (MG) asks the nurse to explain the disease. What response by the nurse is best?

*a. MG is an autoimmune problem in which nerves do not cause muscles to contract*. b. MG is an inherited destruction of peripheral nerve endings and junctions. c. MG consists of trauma-induced paralysis of specific cranial nerves. d. MG is a viral infection of the dorsal root of sensory nerve fibers. ANS: A MG is an autoimmune disorder in which nerve fibers are damaged and their impulses do not lead to muscle contraction. MG is not an inherited or viral disorder and does not paralyze specific cranial nerves.

A nurse teaches a client who is diagnosed with diabetes mellitus. Which statement should the nurse include in this clients plan of care to delay the onset of microvascular and macrovascular complications?

*a. Maintain tight glycemic control and prevent hyperglycemia*. b.Restrict your fluid intake to no more than 2 liters a day. c. Prevent hypoglycemia by eating a bedtime snack. d.Limit your intake of protein to prevent ketoacidosis. ANS: A Hyperglycemia is a critical factor in the pathogenesis of long-term diabetic complications. Maintaining tight glycemic control will help delay the onset of complications. Restricting fluid intake is not part of the treatment plan for clients with diabetes. Preventing hypoglycemia and ketosis, although important, are not as important as maintaining daily glycemic control.

A nurse promotes the prevention of lower back pain by teaching clients at a community center. Which instruction should the nurse include in this education?

*a. Participate in an exercise program to strengthen muscles*. b. Purchase a mattress that allows you to adjust the firmness. c. Wear flat instead of high-heeled shoes to work each day. d. Keep your weight within 20% of your ideal body weight ANS: A Exercise can strengthen back muscles, reducing the incidence of low back pain. The other options will not prevent low back pain.

A nurse prepares a client for lumbar puncture (LP). Which assessment finding should alert the nurse to contact the health care provider?

*a. Shingles on the clients back* b. Client is claustrophobic c. Absence of intravenous access d. Paroxysmal nocturnal dyspnea ANS: A An LP should not be performed if the client has a skin infection at or near the puncture site because of the risk of infection. A nurse would want to notify the health care provider if shingles were identified on the clients back. If a client has shortness of breath when lying flat, the LP can be adapted to meet the clients needs. Claustrophobia, absence of IV access, and paroxysmal nocturnal dyspnea have no impact on whether an LP can be performed.

A nurse delegates care to the unlicensed assistive personnel (UAP). Which statement should the nurse include when delegating care for a client with cranial nerve II impairment?

*a. Tell the client where food items are on the breakfast tray*. b. Place the client in a high-Fowlers position for all meals. c. Make sure the clients food is visually appetizing. d. Assist the client by placing the fork in the left hand. ANS: A Cranial nerve II, the optic nerve, provides central and peripheral vision. A client who has cranial nerve II impairment will not be able to see, so the UAP should tell the client where different food items are on the meal tray. The other options are not appropriate for a client with cranial nerve II impairment.

After teaching a client with diabetes mellitus to inject insulin, the nurse assesses the clients understanding Which statement made by the client indicates a need for additional teaching?

*a. The lower abdomen is the best location because it is closest to the pancreas.* b.I can reach my thigh the best, so I will use the different areas of my thighs. c. By rotating the sites in one area, my chance of having a reaction is decreased. d.Changing injection sites from the thigh to the arm will change absorption rates. ANS: A The abdominal site has the fastest rate of absorption because of blood vessels in the area, not because of its proximity to the pancreas. The other statements are accurate assessments of insulin administration.

A nurse performs an assessment of pain discrimination on an older adult client. The client correctly identifies, with eyes closed, a sharp sensation on the right hand when touched with a pin. Which action should the nurse take next?

*a. Touch the pin on the same area of the left hand*. b. Contact the provider with the assessment results. c. Ask the client about current medications. d. Continue the assessment on the clients feet. ANS: A If testing is begun on the right hand and the client correctly identifies the pain stimulus, the nurse should continue the assessment on the left hand. This is a normal finding and does not need to be reported to the provider, but instead documented in the clients chart. Medications do not need to be assessed in response to this finding. The nurse should assess the left hand prior to assessing the feet.

An HIV-negative client who has an HIV-positive partner asks the nurse about receiving Truvada (emtricitabine and tenofovir). What information is most important to teach the client about this drug?

*a. Truvada does not reduce the need for safe sex practices*. b. This drug has been taken off the market due to increases in cancer. c. Truvada reduces the number of HIV tests you will need. d. This drug is only used for postexposure prophylaxis. ANS: A Truvada is a new drug used for pre-exposure prophylaxis and appears to reduce transmission of human immune deficiency virus (HIV) from known HIV-positive people to HIV-negative people. The drug does not reduce the need for practicing safe sex. Since the drug can lead to drug resistance if used, clients will still need HIV testing every 3 months. This drug has not been taken off the market and is not used for postexposure prophylaxis.

A nurse cares for a client with diabetes mellitus who is visually impaired. The client asks, Can I ask my niece to prefill my syringes and then store them for later use when I need them? How should the nurse respond?

*a. Yes. Prefilled syringes can be stored for 3 weeks in the refrigerator in a vertical position with the needle pointing up*. b.Yes. Syringes can be filled with insulin and stored for a month in a location that is protected from light. c. Insulin reacts with plastic, so prefilled syringes are okay, but you will need to use glass syringes. d.No. Insulin syringes cannot be prefilled and stored for any length of time outside of the container. ANS: A Insulin is relatively stable when stored in a cool, dry place away from light. When refrigerated, prefilled plastic syringes are stable for up to 3 weeks. They should be stored in the refrigerator in the vertical position with the needle pointing up to prevent suspended insulin particles from clogging the needle.

A nurse cares for a client who is recovering from a single-photon emission computed tomography (SPECT) with a radiopharmaceutical agent. Which statement should the nurse include when discussing the plan of care with this client?

*a. You may return to your previous activity level immediately*. b. You are radioactive and must use a private bathroom. c. Frequent assessments of the injection site will be completed. d. We will be monitoring your renal functions closely. ANS: A The client may return to his or her previous activity level immediately. Radioisotopes will be eliminated in the urine after SPECT, but no monitoring or special precautions are required. The injection site will not need to be assessed after the procedure is complete.

A nurse cares for a client who has a family history of diabetes mellitus. The client states, My father has type 1 diabetes mellitus. Will I develop this disease as well? How should the nurse respond?

*a. Your risk of diabetes is higher than the general population, but it may not occur.* b.No genetic risk is associated with the development of type 1 diabetes mellitus. c. The risk for becoming a diabetic is 50% because of how it is inherited. d.Female children do not inherit diabetes mellitus, but male children will. ANS: A Risk for type 1 diabetes is determined by inheritance of genes coding for HLA-DR and HLA-DQ tissue types. Clients who have one parent with type 1 diabetes are at increased risk for its development. Diabetes (type 1) seems to require interaction between inherited risk and environmental factors, so not everyone with these genes develops diabetes. The other statements are not accurate.

.A nurse prepares to flush a peripherally inserted central catheter (PICC) line with 50 units of heparin. The pharmacy supplies a multi-dose vial of heparin with a concentration of 100 units/mL. Which of the syringes shown below should the nurse use to draw up and administer the heparin?

Always use a 10-mL syringe when flushing PICC lines because a smaller syringe creates higher pressure, which could rupture the lumen of the PICC.

Chapter 18: Care of Patients with Arthritis and Other Connective Tissue Diseases

Chapter 18: Care of Patients with Arthritis and Other Connective Tissue Diseases

Chapter 19: Care of Patients with HIV Disease

Chapter 19: Care of Patients with HIV Disease

Chapter 20: Care of Patients with Hypersensitivity (Allergy) and Autoimmunity

Chapter 20: Care of Patients with Hypersensitivity (Allergy) and Autoimmunity

Chapter 41: Assessment of the Nervous System

Chapter 41: Assessment of the Nervous System

Chapter 42: Care of Patients with Problems of the CNS: The Brain

Chapter 42: Care of Patients with Problems of the CNS: The Brain

Chapter 43: Care of Patients with Problems of the CNS: The Spinal Cord

Chapter 43: Care of Patients with Problems of the CNS: The Spinal Cord

Chapter 44: Care of Patients with Problems of the Peripheral Nervous System

Chapter 44: Care of Patients with Problems of the Peripheral Nervous System

Chapter 46: Assessment of the Eye and Vision

Chapter 46: Assessment of the Eye and Vision

Chapter 47: Care of Patients with Eye and Vision Problems

Chapter 47: Care of Patients with Eye and Vision Problems

Chapter 48: Assessment and Care of Patients with Ear and Hearing Problems

Chapter 48: Assessment and Care of Patients with Ear and Hearing Problems

Chapter 64: Care of Patients with Diabetes Mellitus

Chapter 64: Care of Patients with Diabetes Mellitus

A client in sickle cell crisis is dehydrated and in the emergency department. The nurse plans to start an IV. Which fluid choice is best?

a. 0.45% normal saline b. 0.9% normal saline c. Dextrose 50% (D50) d. Lactated Ringers solution ANS: A Because clients in sickle cell crisis are often dehydrated, the fluid of choice is a hypotonic solution such as 0.45% normal saline. 0.9% normal saline and lactated Ringers solution are isotonic. D50 is hypertonic and not used for hydration.

A nurse cares for a client who has diabetes mellitus. The nurse administers 6 units of regular insulin and 10 units of NPH insulin at 0700. At which time should the nurse assess the client for potential problems related to the NPH insulin?

a. 0800 *b.1600* c. 2000 d.2300 ANS: B Neutral protamine Hagedorn (NPH) is an intermediate-acting insulin with an onset of 1.5 hours, peak of 4 to 12 hours, and duration of action of 22 hours. Checking the client at 0800 would be too soon. Checking the client at 2000 and 2300 would be too late. The nurse should check the client at 1600.

A nurse prepares to administer prescribed regular and NPH insulin. Place the nurses actions in the correct order to administer these medications. 1. Inspect bottles for expiration dates. 2. Gently roll the bottle of NPH between the hands. 3. Wash your hands. 4. Inject air into the regular insulin. 5. Withdraw the NPH insulin. 6. Withdraw the regular insulin. 7. Inject air into the NPH bottle. 8. Clean rubber stoppers with an alcohol swab.

a. 1, 3, 8, 2, 4, 6, 7, 5 *b.3, 1, 2, 8, 7, 4, 6, 5* c. 8, 1, 3, 2, 4, 6, 7, 5 d.2, 3, 1, 8, 7, 5, 4, 6 ANS: B After washing hands, it is important to inspect the bottles and then to roll the NPH to mix the insulin. Rubber stoppers should be cleaned with alcohol after rolling the NPH and before sticking a needle into either bottle. It is important to inject air into the NPH bottle before placing the needle in a regular insulin bottle to avoid mixing of regular and NPH insulin. The shorter-acting insulin is always drawn up first.

A nurse is caring for a client using oxygen while in the hospital. What assessment finding indicates that goals for a priority diagnosis are being met?

a. 100% of meals being eaten by the client b. Intact skin behind the ears c. The client understanding the need for oxygen d. Unchanged weight for the past 3 days ANS: B Oxygen tubing can cause pressure ulcers, so clients using oxygen have the nursing diagnosis of Risk for Impaired Skin Integrity. Intact skin behind the ears indicates that goals for this diagnosis are being met. Nutrition and weight are not related to using oxygen. Understanding the need for oxygen is important but would not take priority over a physical problem.

A nursing student caring for a client removes the clients oxygen as prescribed. The client is now breathing what percentage of oxygen in the room air?

a. 14% b. 21% c. 28% d. 31% ANS: B Room air is 21% oxygen.

The nurse instructs a client on how to correctly use an inhaler with a spacer. In which order should these steps occur? 1. Press down firmly on the canister to release one dose of medication. 2. Breathe in slowly and deeply. 3. Shake the whole unit vigorously three or four times. 4. Insert the mouthpiece of the inhaler into the nonmouthpiece end of the spacer. 5. Place the mouthpiece into your mouth, over the tongue, and seal your lips tightly around the mouthpiece. 6. Remove the mouthpiece from your mouth, keep your lips closed, and hold your breath for at least 10 seconds.

a. 2, 3, 4, 5, 6, 1 b. 3, 4, 5, 1, 6, 2 c. 4, 3, 5, 1, 2, 6 d. 5, 3, 6, 1, 2, 4 ANS: C The proper order for correctly using an inhaler with a spacer is as follows. Insert the mouthpiece of the inhaler into the nonmouthpiece end of the spacer. Shake the whole unit vigorously three or four times. Place the mouthpiece into the mouth, over the tongue, and seal the lips tightly around it. Press down firmly on the canister of the inhaler to release one dose of medication into the spacer. Breathe in slowly and deeply. Remove the mouthpiece from the mouth, and, keeping the lips closed, hold the breath for at least 10 seconds. Then breathe out slowly. Wait at least 1 minute between puffs.

The nurse instructs a client on the steps needed to obtain a peak expiratory flow rate. In which order should these steps occur? 1. Take as deep a breath as possible. 2. Stand up (unless you have a physical disability). 3. Place the meter in your mouth, and close your lips around the mouthpiece. 4. Make sure the device reads zero or is at base level. 5. Blow out as hard and as fast as possible for 1 to 2 seconds. 6. Write down the value obtained. 7. Repeat the process two additional times, and record the highest number in your chart.

a. 4, 2, 1, 3, 5, 6, 7 b. 3, 4, 1, 2, 5, 7, 6 c. 2, 1, 3, 4, 5, 6, 7 d. 1, 3, 2, 5, 6, 7, 4 ANS: A The proper order for obtaining a peak expiratory flow rate is as follows. Make sure the device reads zero or is at base level. The client should stand up (unless he or she has a physical disability). The client should take as deep a breath as possible, place the meter in the mouth, and close the lips around the mouthpiece. The client should blow out as hard and as fast as possible for 1 to 2 seconds. The value obtained should be written down. The process should be repeated two more times, and the highest of the three numbers should be recorded in the clients chart.

A nurse assesses a client with a brain tumor. The client opens his eyes when the nurse calls his name, mumbles in response to questions, and follows simple commands. How should the nurse document this clients assessment using the Glasgow Coma Scale shown below?

a. 8 b. 10 *c. 12* d. 14 ANS: C The client opens his eyes to speech (Eye opening: To sound = 3), mumbles in response to questions (Verbal response: Inappropriate words = 3), and follows simple commands (Motor response: Obeys commands = 6). Therefore, the clients Glasgow Coma Scale score is: 3 + 3 + 6 = 12.

A nurse assesses clients at a community center. Which client is at greatest risk for lower back pain?

a. A 24-year-old female who is 25 weeks pregnant b. A 36-year-old male who uses ergonomic techniques *c. A 45-year-old male with osteoarthritis* d. A 53-year-old female who uses a walker ANS: C Osteoarthritis causes changes to support structures, increasing the clients risk for low back pain. The other clients are not at high risk.

A nurse assesses clients on the medical-surgical unit. Which client is at greatest risk for bladder cancer?

a. A 25-year-old female with a history of sexually transmitted diseases b. A 42-year-old male who has worked in a lumber yard for 10 years c. A 55-year-old female who has had numerous episodes of bacterial cystitis d. An 86-year-old male with a 50pack-year cigarette smoking history ANS: D The greatest risk factor for bladder cancer is a long history of tobacco use. The other factors would not necessarily contribute to the development of this specific type of cancer.

A nurse assesses clients on the medical-surgical unit. Which client is at greatest risk for development of obstructive sleep apnea?

a. A 26-year-old woman who is 8 months pregnant b. A 42-year-old man with gastroesophageal reflux disease c. A 55-year-old woman who is 50 pounds overweight d. A 73-year-old man with type 2 diabetes mellitus ANS: C The client at highest risk would be the one who is extremely overweight. None of the other clients have risk factors for sleep apnea.

A nurse assesses clients who are at risk for diabetes mellitus. Which client is at greatest risk?

a. A 29-year-old Caucasian b.A 32-year-old AfricanAmerican c. A 44-year-old Asian *d.A 48-year-old American Indian* ANS: D Diabetes is a particular problem among African Americans, Hispanics, and American Indians. The incidence of diabetes increases in all races and ethnic groups with age. Being both an American Indian and middle-aged places this client at highest risk

A nurse plans care for clients with urinary incontinence. Which client is correctly paired with the appropriate intervention?

a. A 29-year-old client after a difficult vaginal delivery Habit training b. A 58-year-old postmenopausal client who is not taking estrogen therapy Electrical stimulation c. A 64-year-old female with Alzheimers-type senile dementia Bladder training d. A 77-year-old female who has difficulty ambulating Exercise therapy ANS: B Exercise therapy and electrical stimulation are used for clients with stress incontinence related to childbirth or low levels of estrogen after menopause. Exercise therapy increases pelvic wall strength; it does not improve ambulation. Physical therapy and a bedside commode would be appropriate interventions for the client who has difficulty ambulating. Habit training is the type of bladder training that will be most effective with cognitively impaired clients. Bladder training can be used only with a client who is alert, aware, and able to resist the urge to urinate.

A nurse is assessing clients for fluid and electrolyte imbalances. Which client should the nurse assess first for potential hyponatremia?

a. A 34-year-old on NPO status who is receiving intravenous D5W b. A 50-year-old with an infection who is prescribed a sulfonamide antibiotic c. A 67-year-old who is experiencing pain and is prescribed ibuprofen (Motrin) d. A 73-year-old with tachycardia who is receiving digoxin (Lanoxin) ANS: A Dextrose 5% in water (D5W) contains no electrolytes. Because the client is not taking any food or fluids by mouth (NPO), normal sodium excretion can lead to hyponatremia. The sulfonamide antibiotic, ibuprofen, and digoxin will not put a client at risk for hyponatremia.

A nurse assesses clients on the medical-surgical unit. Which client is at greatest risk for the development of bacterial cystitis?

a. A 36-year-old female who has never been pregnant b. A 42-year-old male who is prescribed cyclophosphamide c. A 58-year-old female who is not taking estrogen replacement d. A 77-year-old male with mild congestive heart failure ANS: C Females at any age are more susceptible to cystitis than men because of the shorter urethra in women. Postmenopausal women who are not on hormone replacement therapy are at increased risk for bacterial cystitis because of changes in the cells of the urethra and vagina. The middle-aged woman who has never been pregnant would not have a risk potential as high as the older woman who is not using hormone replacement therapy.

A nurse teaches clients at a community center about risks for dehydration. Which client is at greatest risk for dehydration?

a. A 36-year-old who is prescribed long-term steroid therapy b. A 55-year-old receiving hypertonic intravenous fluids c. A 76-year-old who is cognitively impaired d. An 83-year-old with congestive heart failure ANS: C Older adults, because they have less total body water than younger adults, are at greater risk for development of dehydration. Anyone who is cognitively impaired and cannot obtain fluids independently or cannot make his or her need for fluids known is at high risk for dehydration.

A nurse assesses clients on a cardiac unit. Which client should the nurse identify as being at greatest risk for the development of left-sided heart failure?

a. A 36-year-old woman with aortic stenosis b. A 42-year-old man with pulmonary hypertension c. A 59-year-old woman who smokes cigarettes daily d. A 70-year-old man who had a cerebral vascular accident ANS: A Although most people with heart failure will have failure that progresses from left to right, it is possible to have left-sided failure alone for a short period. It is also possible to have heart failure that progresses from right to left. Causes of left ventricular failure include mitral or aortic valve disease, coronary artery disease, and hypertension. Pulmonary hypertension and chronic cigarette smoking are risk factors for right ventricular failure. A cerebral vascular accident does not increase the risk of heart failure

A nurse assesses clients at a community health center. Which client is at highest risk for the development of colorectal cancer?

a. A 37-year-old who drinks eight cups of coffee daily b. A 44-year-old with irritable bowel syndrome (IBS) c. A 60-year-old lawyer who works 65 hours per week d. A 72-year-old who eats fast food frequently ANS: D Colon cancer is rare before the age of 40, but its incidence increases rapidly with advancing age. Fast food tends to be high in fat and low in fiber, increasing the risk for colon cancer. Coffee intake, IBS, and a heavy workload do not increase the risk for colon cancer.

The nurse is evaluating a 3-day diet history with a client who has an elevated lipid panel. What meal selection indicates the client is managing this condition well with diet?

a. A 4-ounce steak, French fries, iceberg lettuce b. Baked chicken breast, broccoli, tomatoes c. Fried catfish, cornbread, peas d. Spaghetti with meat sauce, garlic bread ANS: B The diet recommended for this client would be low in saturated fats and red meat, high in vegetables and whole grains (fiber), low in salt, and low in trans fat. The best choice is the chicken with broccoli and tomatoes. The French fries have too much fat and the iceberg lettuce has little fiber. The catfish is fried. The spaghetti dinner has too much red meat and no vegetables.

An emergency department nurse triages clients who present with chest discomfort. Which client should the nurse plan to assess first?

a. A 42-year-old female who describes her pain as a dull ache with numbness in her fingers b. A 49-year-old male who reports moderate pain that is worse on inspiration c. A 53-year-old female who reports substernal pain that radiates to her abdomen d. A 58-year-old male who describes his pain as intense stabbing that spreads across his chest ANS: D All clients who have chest pain should be assessed more thoroughly. To determine which client should be seen first, the nurse must understand common differences in pain descriptions. Intense stabbing, vise-like substernal pain that spreads through the clients chest, arms, jaw, back, or neck is indicative of a myocardial infarction. The nurse should plan to see this client first to prevent cardiac cell death. A dull ache with numbness in the fingers is consistent with anxiety. Pain that gets worse with inspiration is usually related to a pleuropulmonary problem. Pain that spreads to the abdomen is often associated with an esophageal-gastric problem, especially when this pain is experienced by a male client. Female clients may experience abdominal discomfort with a myocardial event. Although clients with anxiety, pleuropulmonary, and esophageal-gastric problems should be seen, they are not a higher priority than myocardial infarction.

A nurse is assessing clients on a medical-surgical unit. Which client should the nurse identify as being at greatest risk for atrial fibrillation?

a. A 45-year-old who takes an aspirin daily b. A 50-year-old who is post coronary artery bypass graft surgery c. A 78-year-old who had a carotid endarterectomy d. An 80-year-old with chronic obstructive pulmonary disease ANS: B Atrial fibrillation occurs commonly in clients with cardiac disease and is a common occurrence after coronary artery bypass graft surgery. The other conditions do not place these clients at higher risk for atrial fibrillation.

A pulmonary nurse cares for clients who have chronic obstructive pulmonary disease (COPD). Which client should the nurse assess first?

a. A 46-year-old with a 30pack-year history of smoking b. A 52-year-old in a tripod position using accessory muscles to breathe c. A 68-year-old who has dependent edema and clubbed fingers d. A 74-year-old with a chronic cough and thick, tenacious secretions ANS: B The client who is in a tripod position and using accessory muscles is working to breathe. This client must be assessed first to establish how well the client is breathing and provide interventions to minimize respiratory failure. The other clients are not in acute distress.

A nurse assesses several clients who have a history of asthma. Which client should the nurse assess first?

a. A 66-year-old client with a barrel chest and clubbed fingernails b. A 48-year-old client with an oxygen saturation level of 92% at rest c. A 35-year-old client who has a longer expiratory phase than inspiratory phase d. A 27-year-old client with a heart rate of 120 beats/min ANS: D Tachycardia can indicate hypoxemia as the body tries to circulate the oxygen that is available. A barrel chest is not an emergency finding. Likewise, a pulse oximetry level of 92% is not considered an acute finding. The expiratory phase is expected to be longer than the inspiratory phase in someone with airflow limitation.

A nurse cares for adult clients who experience urge incontinence. For which client should the nurse plan a habit training program?

a. A 78-year-old female who is confused b. A 65-year-old male with diabetes mellitus c. A 52-year-old female with kidney failure d. A 47-year-old male with arthritis ANS: A For a bladder training program to succeed in a client with urge incontinence, the client must be alert, aware, and able to resist the urge to urinate. Habit training will work best for a confused client. This includes going to the bathroom (or being assisted to the bathroom) at set times. The other clients may benefit from another type of bladder training.

A client has fibromyalgia and is prescribed duloxetine hydrochloride (Cymbalta). The client calls the clinic and asks the nurse why an antidepressant drug has been prescribed. What response by the nurse is best?

a. A little sedation will help you get some rest. b. Depression often accompanies fibromyalgia. *c. This drug works in the brain to decrease pain*. d. You will have more energy after taking this drug ANS: C Duloxetine works to increase the release of the neurotransmitters serotonin and norepinephrine, which reduces the pain from fibromyalgia. The other answers are inaccurate

A nurse assesses a clients recent memory. Which client statement confirms that the clients remote memory is intact?

a. A young girl wrapped in a shroud fell asleep on a bed of clouds. b. I was born on April 3, 1967, in Johnstown Community Hospital. c. Apple, chair, and pencil are the words you just stated. *d. I ate oatmeal with wheat toast and orange juice for breakfast*. ANS: D Asking clients about recent events that can be verified, such as what the client ate for breakfast, assesses the clients recent memory. The clients ability to make up a rhyme tests not memory, but rather a higher level of cognition. Asking clients about certain facts from the past that can be verified assesses remote or long-term memory. Asking the client to repeat words assesses the clients immediate memory.

The nurse working in the rheumatology clinic is seeing clients with rheumatoid arthritis (RA). What assessment would be most important for the client whose chart contains the diagnosis of Sjgrens syndrome?

a. Abdominal assessment b. Oxygen saturation c. Renal function studies *d. Visual acuity* ANS: D Sjgrens syndrome is seen in clients with RA and manifests with dryness of the eyes, mouth, and vagina in females. Visual disturbances can occur. The other assessments are not related to RA and Sjgrens syndrome

A client with myasthenia gravis has the priority client problem of inadequate nutrition. What assessment finding indicates that the priority goal for this client problem has been met?

a. Ability to chew and swallow without aspiration b. Eating 75% of meals and between-meal snacks c. Intake greater than output 3 days in a row *d. Weight gain of 3 pounds in 1 month* ANS: D Weight gain is the best indicator that the client is receiving enough nutrition. Being able to chew and swallow is important for eating, but adequate nutrition can be accomplished through enteral means if needed. Swallowing without difficulty indicates an intact airway. Since the question does not indicate what the clients meals and snacks consist of, eating 75% may or may not be adequate. Intake and output refers to fluid balance.

A nurse teaches a client to use a room humidifier after a laryngectomy. Which statement should the nurse include in this clients teaching?

a. Add peppermint oil to the humidifier to relax the airway. b. Make sure you clean the humidifier to prevent infection. c. Keep the humidifier filled with water at all times. d. Use the humidifier when you sleep, even during daytime naps. ANS: B Priority teaching related to the use of a room humidifier focuses on infection control. Clients should be taught to meticulously clean the humidifier to prevent the spread of mold or other sources of infection. Peppermint oil should not be added to a humidifier. The humidifier should be refilled with water as needed and should be used while awake and asleep.

A nurse is weighing and measuring a client with severe kyphosis. What is the best method to obtain this clients height?

a. Add the trunk and leg measurements. b. Ask the client how tall he or she is. c. Estimate by measuring clothing. d. Use knee-height calipers. ANS: D A sliding blade knee-height caliper is used to obtain the height of a client who cannot stand upright, such as those with kyphosis or lower extremity contractures. The other methods will not yield accurate data.

A client with acquired immune deficiency syndrome is hospitalized and has weeping Kaposis sarcoma lesions. The nurse dresses them with sterile gauze. When changing these dressings, which action is most important?

a. Adhering to Standard Precautions b. Assessing tolerance to dressing changes c. Performing hand hygiene before and after care *d. Disposing of soiled dressings properly* ANS: D All of the actions are important, but due to the infectious nature of this illness, ensuring proper disposal of soiled dressings is vital.

A nurse auscultated heart tones on an older adult client. Which action should the nurse take based on heart tones heard? (Click the media button to hear the audio clip.)

a. Administer a diuretic. b. Document the finding. c. Decrease the IV flow rate. d. Evaluate the clients medications. ANS: B The sound heard is an atrial gallop S4. An atrial gallop may be heard in older clients because of a stiffened ventricle. The nurse should document the finding, but no other intervention is needed at this time.

A nurse evaluates the following arterial blood gas and vital sign results for a client with chronic obstructive pulmonary disease (COPD): Arterial Blood Gas Results Vital Signs pH = 7.32 PaCO2 = 62 mm Hg PaO2 = 46 mm Hg HCO3 = 28 mEq/L Heart rate = 110 beats/min Respiratory rate = 12 breaths/min Blood pressure = 145/65 mm Hg Oxygen saturation = 76% Which action should the nurse take first?

a. Administer a short-acting beta2 agonist inhaler. b. Document the findings as normal for a client with COPD. c. Teach the client diaphragmatic breathing techniques. d. Initiate oxygenation therapy to increase saturation to 92%. ANS: D Oxygen should be administered to a client who is hypoxic even if the client has COPD and is a carbon dioxide retainer. The other interventions do not address the clients hypoxia, which is the priority.

.A nurse is caring for a client who is having a subclavian central venous catheter inserted. The client begins to report chest pain and difficulty breathing. After administering oxygen, which action should the nurse take next?

a. Administer a sublingual nitroglycerin tablet. b.Prepare to assist with chest tube insertion. c. Place a sterile dressing over the IV site. d.Re-position the client into the Trendelenburg position. ANS: B An insertion-related complication of central venous catheters is a pneumothorax. Signs and symptoms of a pneumothorax include chest pain and dyspnea. Treatment includes removing the catheter, administering oxygen, and placing a chest tube. Pain is caused by the pneumothorax, which must be taken care of with a chest tube insertion. Use of a sterile dressing and placement of the client in a Trendelenburg position are not indicated for the primary problem of a pneumothorax.

A client is brought to the emergency department after a car crash. The client has a large piece of glass in the left eye. What action by the nurse takes priority?

a. Administer a tetanus booster shot. *b. Ensure the client has a patent airway*. c. Prepare to irrigate the clients eye. d. Turn the client on the unaffected side. ANS: B Airway always comes first. After ensuring a patent airway and providing cervical spine precautions (do not turn the client to the side), the nurse provides other care that may include administering a tetanus shot. The clients eye may or may not be irrigated.

A nurse is caring for a client who received benzocaine spray prior to a recent bronchoscopy. The client presents with continuous cyanosis even with oxygen therapy. Which action should the nurse take next?

a. Administer an albuterol treatment. b. Notify the Rapid Response Team. c. Assess the clients peripheral pulses. d. Obtain blood and sputum cultures. ANS: B Cyanosis unresponsive to oxygen therapy is a manifestation of methemoglobinemia, which is an adverse effect of benzocaine spray. Death can occur if the level of methemoglobin rises and cyanosis occurs. The nurse should notify the Rapid Response Team to provide advanced nursing care. An albuterol treatment would not address the clients oxygenation problem. Assessment of pulses and cultures will not provide data necessary to treat this client.

A client having a tube feeding begins vomiting. What action by the nurse is most appropriate?

a. Administer an antiemetic. b. Check the clients gastric residual. c. Hold the feeding until the nausea subsides. d. Reduce the rate of the tube feeding by half. ANS: C The nurse should hold the feeding until the nausea and vomiting have subsided and consult with the provider on the rate at which to restart the feeding. Giving an antiemetic is not appropriate. After vomiting, a gastric residual will not be accurate. The nurse should not continue to feed the client while he or she is vomiting.

A client is in the hospital and receiving IV antibiotics. When the nurse answers the clients call light, the client presents an appearance as shown below: What action by the nurse takes priority?

a. Administer epinephrine 1:1000, 0.3 mg IV push immediately. b. Apply oxygen by facemask at 100% and a pulse oximeter. *c. Ensure a patent airway while calling the Rapid Response Team*. d. Reassure the client that these manifestations will go away. ANS: C The nurse should ensure the clients airway is patent and either call the Rapid Response Team or delegate this to someone else. Epinephrine needs to be administered right away, but not without a prescription by the physician unless standing orders exist. The client may need oxygen, but a patent airway comes first. Reassurance is important, but airway and calling the Rapid Response Team are the priorities.

A nurse is caring for a client receiving enteral feedings through a Dobhoff tube. What action by the nurse is best to prevent hyperosmolarity?

a. Administer free-water boluses. b. Change the clients formula. c. Dilute the clients formula. d. Slow the rate of infusion. ANS: A Proteins and sugar molecules in the enteral feeding product contribute to dehydration due to increased osmolarity. The nurse can administer free-water boluses after consulting with the provider on the appropriate amount and timing of the boluses, or per protocol. The client may not be able to switch formulas. Diluting the formula is not appropriate. Slowing the rate of the infusion will not address the problem.

A nurse assesses a client who is recovering from extracorporeal shock wave lithotripsy for renal calculi. The nurse notes an ecchymotic area on the clients right lower back. Which action should the nurse take?

a. Administer fresh-frozen plasma. b. Apply an ice pack to the site. c. Place the client in the prone position. d. Obtain serum coagulation test results. ANS: B The shock waves from lithotripsy can cause bleeding into the tissues through which the waves pass. Application of ice can reduce the extent and discomfort of the bruising. Although coagulation test results and fresh-frozen plasma are used to assess and treat bleeding disorders, ecchymosis after this procedure is not unusual and does not warrant a higher level of intervention. Changing the clients position will not decrease bleeding.

A client with peptic ulcer disease is in the emergency department and reports the pain has gotten much worse over the last several days. The clients blood pressure when lying down was 122/80 mm Hg and when standing was 98/52 mm Hg. What action by the nurse is most appropriate?

a. Administer ibuprofen (Motrin). b. Call the Rapid Response Team. c. Start a large-bore IV with normal saline. d. Tell the client to remain lying down. ANS: C This client has orthostatic changes to the blood pressure, indicating fluid volume loss. The nurse should start a large-bore IV with isotonic solution. Ibuprofen will exacerbate the ulcer. The Rapid Response Team is not needed at this point. The client should be put on safety precautions, which includes staying in bed, but this is not the priority.

A nurse assists with the cardioversion of a client experiencing acute atrial fibrillation. Which action should the nurse take prior to the initiation of cardioversion?

a. Administer intravenous adenosine. b. Turn off oxygen therapy. c. Ensure a tongue blade is available. d. Position the client on the left side. ANS: B For safety during cardioversion, the nurse should turn off any oxygen therapy to prevent fire. The other interventions are not appropriate for a cardioversion. The client should be placed in a supine position.

A nurse cares for a client with an intravenous temporary pacemaker for bradycardia. The nurse observes the presence of a pacing spike but no QRS complex on the clients electrocardiogram. Which action should the nurse take next?

a. Administer intravenous diltiazem (Cardizem). b. Assess vital signs and level of consciousness. c. Administer sublingual nitroglycerin. d. Assess capillary refill and temperature. ANS: B In temporary pacing, the wires are threaded onto the epicardial surface of the heart and exit through the chest wall. The pacemaker spike should be followed immediately by a QRS complex. Pacing spikes seen without subsequent QRS complexes imply loss of capture. If there is no capture, then there is no ventricular depolarization and contraction. The nurse should assess for cardiac output via vital signs and level of consciousness. The other interventions would not determine if the client is tolerating the loss of capture.

A nurse assesses a client with a mechanical bowel obstruction who reports intermittent abdominal pain. An hour later the client reports constant abdominal pain. Which action should the nurse take next?

a. Administer intravenous opioid medications. b. Position the client with knees to chest. c. Insert a nasogastric tube for decompression. d. Assess the clients bowel sounds. ANS: D A change in the nature and timing of abdominal pain in a client with a bowel obstruction can signal peritonitis or perforation. The nurse should immediately check for rebound tenderness and the absence of bowel sounds. The nurse should not medicate the client until the provider has been notified of the change in his or her condition. The nurse may help the client to the knee-chest position for comfort, but this is not the priority action. The nurse need not insert a nasogastric tube for decompression.

An older client has returned to the surgical unit after a total hip replacement. The client is confused and restless. What intervention by the nurse is most important to prevent injury?

a. Administer mild sedation. b. Keep all four siderails up. c. Restrain the clients hands. *d. Use an abduction pillow.* ANS: D Older clients often have trouble metabolizing anesthetics and pain medication, leading to confusion or restlessness postoperatively. To prevent the hip from dislocating, the nurse should use an abduction pillow since the client cannot follow directions at this time. Sedation may worsen the clients mental status and should be avoided. Using all four siderails may be considered a restraint. Hand restraints are not necessary in this situation

A client is having an esophagogastroduodenoscopy (EGD) and has been given midazolam hydrochloride (Versed). The clients respiratory rate is 8 breaths/min. What action by the nurse is best?

a. Administer naloxone (Narcan). b. Call the Rapid Response Team. c. Provide physical stimulation. d. Ventilate with a bag-valve-mask. ANS: C For an EGD, clients are given mild sedation but should still be able to follow commands. For shallow or slow respirations after the sedation is given, the nurses first action is to provide a physical stimulation such as a sternal rub and directions to breathe deeply. Naloxone is not the antidote for Versed. The Rapid Response Team is not needed at this point. The client does not need manual ventilation.

A nurse admits a client from the emergency department. Client data are listed below: History Physical Assessment Laboratory Values 70 years of age History of diabetes On insulin twice a day Reports new-onset dyspnea and productive cough Crackles and rhonchi heard throughout the lungs Dullness to percussion LLL Afebrile Oriented to person only WBC: 5,200/mm3 PaO2 on room air 65 mm Hg What action by the nurse is the priority?

a. Administer oxygen at 4 liters per nasal cannula. b.Begin broad-spectrum antibiotics. c. Collect a sputum sample for culture. d.Start an IV of normal saline at 50 mL/hr. ANS: A All actions are appropriate for this client who has manifestations of pneumonia. However, airway and breathing come first, so begin oxygen administration and titrate it to maintain saturations greater than 95%. Start the IV and collect a sputum culture, and then begin antibiotics.

A nurse cares for a client with atrial fibrillation who reports fatigue when completing activities of daily living. What interventions should the nurse implement to address this clients concerns?

a. Administer oxygen therapy at 2 liters per nasal cannula. b. Provide the client with a sleeping pill to stimulate rest. c. Schedule periods of exercise and rest during the day. d. Ask unlicensed assistive personnel to help bathe the client. ANS: C Clients who have atrial fibrillation are at risk for decreased cardiac output and fatigue when completing activities of daily living. The nurse should schedule periods of exercise and rest during the day to decrease fatigue. The other interventions will not assist the client with self-care activities.

A client presents to the emergency department with a severely lacerated artery. What is the priority action for the nurse?

a. Administer oxygen via non-rebreather mask. b. Ensure the client has a patent airway. c. Prepare to assist with suturing the artery. d. Start two large-bore IVs with normal saline. ANS: B Airway always takes priority, followed by breathing and circulation. The nurse ensures the client has a patent airway prior to providing any other care measures.

A client presents to the emergency department in sickle cell crisis. What intervention by the nurse takes priority?

a. Administer oxygen. b. Apply an oximetry probe. c. Give pain medication. d. Start an IV line. ANS: A All actions are appropriate, but remembering the ABCs, oxygen would come first. The main problem in a sickle cell crisis is tissue and organ hypoxia, so providing oxygen helps halt the process.

A client is 4 hours postoperative after a femoropopliteal bypass. The client reports throbbing leg pain on the affected side, rated as 7/10. What action by the nurse takes priority?

a. Administer pain medication as ordered. b. Assess distal pulses and skin color. c. Document the findings in the clients chart. d. Notify the surgeon immediately. ANS: B Once perfusion has been restored or improved to an extremity, clients can often feel a throbbing pain due to the increased blood flow. However, it is important to differentiate this pain from ischemia. The nurse should assess for other signs of perfusion, such as distal pulses and skin color/temperature. Administering pain medication is done once the nurse determines the clients perfusion status is normal. Documentation needs to be thorough. Notifying the surgeon is not necessary.

A client is having a bone marrow biopsy today. What action by the nurse takes priority?

a. Administer pain medication first. b. Ensure valid consent is on the chart. c. Have the client shower in the morning. d. Premedicate the client with sedatives. ANS: B A bone marrow biopsy is an invasive procedure that requires informed consent. Pain medication and sedation are important components of care for this client but do not take priority. The client may or may not need or be able to shower.

A client is scheduled to have a hip replacement. Preoperatively, the client is found to be mildly anemic and the surgeon states the client may need a blood transfusion during or after the surgery. What action by the preoperative nurse is most important?

a. Administer preoperative medications as prescribed. *b. Ensure that a consent for transfusion is on the chart*. c. Explain to the client how anemia affects healing. d. Teach the client about foods high in protein and iron. ANS: B The preoperative nurse should ensure that all valid consents are on the chart, including one for blood transfusions if this may be needed. Administering preoperative medications is important for all preoperative clients and is not specific to this client. Teaching in the preoperative area should focus on immediate concerns.

A client is scheduled to have a tracheostomy placed in an hour. What action by the nurse is the priority?

a. Administer prescribed anxiolytic medication. b. Ensure informed consent is on the chart. c. Reinforce any teaching done previously. d. Start the preoperative antibiotic infusion. ANS: B Since this is an operative procedure, the client must sign an informed consent, which must be on the chart. Giving anxiolytics and antibiotics and reinforcing teaching may also be required but do not take priority.

A client is hospitalized with Pneumocystis jiroveci pneumonia. The client reports shortness of breath with activity and extreme fatigue. What intervention is best to promote comfort?

a. Administer sleeping medication. b. Perform most activities for the client c. Increase the clients oxygen during activity. *d. Pace activities, allowing for adequate rest*. ANS: D This client has two major reasons for fatigue: decreased oxygenation and systemic illness. The nurse should not do everything for the client but rather let the client do as much as possible within limits and allow for adequate rest in between. Sleeping medications may be needed but not as the first step, and only with caution. Increasing oxygen during activities may or may not be warranted, but first the nurse must try pacing the clients activity.

A nurse reviews the chart and new prescriptions for a client with diabetic ketoacidosis: Vital Signs and Assessment Laboratory Results Medications Blood pressure: 90/62 mm Hg Pulse: 120 beats/min Respiratory rate: 28 breaths/min Urine output: 20 mL/hr via catheter Serum potassium: 2.6 mEq/L Potassium chloride 40 mEq IV bolus STAT Increase IV fluid to 100 mL/hr Which action should the nurse take?

a. Administer the potassium and then consult with the provider about the fluid order. *b.Increase the intravenous rate and then consult with the provider about the potassium prescription*. c. Administer the potassium first before increasing the infusion flow rate. d.Increase the intravenous flow rate before administering the potassium. ANS: B The client is acutely ill and is severely dehydrated and hypokalemic. The client requires more IV fluids and potassium. However, potassium should not be infused unless the urine output is at least 30 mL/hr. The nurse should first increase the IV rate and then consult with the provider about the potassium.

The nurse is caring for a client with peptic ulcer disease who reports sudden onset of sharp abdominal pain. On palpation, the clients abdomen is tense and rigid. What action takes priority?

a. Administer the prescribed pain medication. *b. Notify the health care provider immediately.* c. Percuss all four abdominal quadrants. d. Take and document a set of vital signs. ANS: B This client has manifestations of a perforated ulcer, which is an emergency. The priority is to get the client medical attention. The nurse can take a set of vital signs while someone else calls the provider. The nurse should not percuss the abdomen or give pain medication since the client may need to sign consent for surgery.

.A nurse is caring for a client with a peripheral vascular access device who is experiencing pain, redness, and swelling at the site. After removing the device, which action should the nurse take to relieve pain?

a. Administer topical lidocaine to the site. b.Place warm compresses on the site. c. Administer prescribed oral pain medication. d.Massage the site with scented oils. ANS: B At the first sign of phlebitis, the catheter should be removed and warm compresses used to relieve pain. The other options are not appropriate for this type of pain.

The nurse is preparing a client for a Tensilon (edrophonium chloride) test. What action by the nurse is most important?

a. Administering anxiolytics b. Having a ventilator nearby *c. Obtaining atropine sulfate* d. Sedating the client ANS: C Atropine is the antidote to edrophonium chloride and should be readily available when a client is having a Tensilon test. The nurse would not want to give medications that might cause increased weakness or sedation. A ventilator is not necessary to have nearby, although emergency equipment should be available.

A nurse is discharging a client to a short-term rehabilitation center after a joint replacement. Which action by the nurse is most important?

a. Administering pain medication before transport b. Answering any last-minute questions by the client c. Ensuring the family has directions to the facility *d. Providing a verbal hand-off report to the facility* ANS: D As required by The Joint Commission and other accrediting agencies, a hand-off report must be given to the new provider to prevent error. The other options are valid responses but do not take priority.

The nurse is caring for clients on the medical-surgical unit. What action by the nurse will help prevent a client from having a type II hypersensitivity reaction?

a. Administering steroids for severe serum sickness b. Correctly identifying the client prior to a blood transfusion c. Keeping the client free of the offending agent d. Providing a latex-free environment for the client ANS: B A classic example of a type II hypersensitivity reaction is a blood transfusion reaction. These can be prevented by correctly identifying the client and cross-checking the unit of blood to be administered. Serum sickness is a type III reaction. Avoidance therapy is the cornerstone of treatment for a type IV hypersensitivity. Latex allergies are a type I hypersensitivity.

A nurse cares for a client who has an 80% blockage of the right coronary artery (RCA) and is scheduled for bypass surgery. Which intervention should the nurse be prepared to implement while this client waits for surgery?

a. Administration of IV furosemide (Lasix) b. Initiation of an external pacemaker c. Assistance with endotracheal intubation d. Placement of central venous access ANS: B The RCA supplies the right atrium, the right ventricle, the inferior portion of the left ventricle, and the atrioventricular (AV) node. It also supplies the sinoatrial node in 50% of people. If the client totally occludes the RCA, the AV node would not function and the client would go into heart block, so emergency pacing should be available for the client. Furosemide, intubation, and central venous access will not address the primary complication of RCA occlusion, which is AV node malfunction.

.A nurse cares for a client experiencing diabetic ketoacidosis who presents with Kussmaul respirations. Which action should the nurse take?

a. Administration of oxygen via face mask b.Intravenous administration of 10% glucose c. Implementation of seizure precautions *d.Administration of intravenous insulin* ANS: D The rapid, deep respiratory efforts of Kussmaul respirations are the bodys attempt to reduce the acids produced by using fat rather than glucose for fuel. Only the administration of insulin will reduce this type of respiration by assisting glucose to move into cells and to be used for fuel instead of fat. The client who is in ketoacidosis may not experience any respiratory impairment and therefore does not need additional oxygen. Giving the client glucose would be contraindicated. The client does not require seizure precautions.

.The charge nurse on a medical unit is preparing to admit several clients who have possible pandemic flu during a preparedness drill. What action by the nurse is best?

a. Admit the clients on Contact Precautions. b.Cohort the clients in the same area of the unit. c. Do not allow pregnant caregivers to care for these clients. d.Place the clients on enhanced Droplet Precautions ANS: B Preventing the spread of pandemic flu is equally important as caring for the clients who have it. Clients can be cohorted together in the same set of rooms on one part of the unit to use distancing to help prevent the spread of the disease. The other actions are not appropriate.

A client in the family practice clinic has restless leg syndrome. Routine laboratory work reveals white blood cells 8000/mm3, magnesium 0.8 mEq/L, and sodium 138 mEq/L. What action by the nurse is best?

a. Advise the client to restrict fluids. b. Assess the client for signs of infection. c. Have the client add table salt to food. *d. Instruct the client on a magnesium supplement*. ANS: D Iron and magnesium deficiencies can sometimes exacerbate or increase symptoms of restless leg syndrome. The clients magnesium level is low, and the client should be advised to add a magnesium supplement. The other actions are not needed.

. A client has severe tinnitus that cannot be treated adequately. What action by the nurse is best?

a. Advise the client to take antianxiety medication. b. Educate the client on nerve cutting procedures. *c. Refer the client to online or local support groups*. d. Teach the client side effects of furosemide (Lasix). ANS: C If the clients tinnitus cannot be treated, he or she will have to learn to cope with it. Referring the client to tinnitus support groups can be helpful. The other options are not warranted.

A nurse is interested in providing community education and screening on hypertension. In order to reach a priority population, to what target audience should the nurse provide this service?

a. African-American churches b. Asian-American groceries c. High school sports camps d. Womens health clinics ANS: A African Americans in the United States have one of the highest rates of hypertension in the world. The nurse has the potential to reach this priority population by providing services at African-American churches. Although hypertension education and screening are important for all groups, African Americans are the priority population for this intervention.

A nurse is assessing a client who has acute pancreatitis and is at risk for an acid-base imbalance. For which manifestation of this acid-base imbalance should the nurse assess?

a. Agitation b. Kussmaul respirations c. Seizures d. Positive Chvosteks sign ANS: B The pancreas is a major site of bicarbonate production. Pancreatitis can cause a relative metabolic acidosis through underproduction of bicarbonate ions. Manifestations of acidosis include lethargy and Kussmaul respirations. Agitation, seizures, and a positive Chvosteks sign are manifestations of the electrolyte imbalances that accompany alkalosis.

A client is scheduled for a total gastrectomy for gastric cancer. What preoperative laboratory result should the nurse report to the surgeon immediately?

a. Albumin: 2.1 g/dL b. Hematocrit: 28% c. Hemoglobin: 8.1 mg/dL d. International normalized ratio (INR): 4.2 ANS: D An INR as high as 4.2 poses a serious risk of bleeding during the operation and should be reported. The albumin is low and is an expected finding. The hematocrit and hemoglobin are also low, but this is expected in gastric cancer.

A nurse is reviewing laboratory values for several clients. Which value causes the nurse to conduct nutritional assessments as a priority?

a. Albumin: 3.5 g/dL b. Cholesterol: 142 mg/dL c. Hemoglobin: 9.8 mg/dL d. Prealbumin: 28 mg/dL ANS: B A cholesterol level below 160 mg/dL is a possible indicator of malnutrition, so this client would be at highest priority for a nutritional assessment. The albumin and prealbumin levels are normal. The low hemoglobin could be from several problems, including dietary deficiencies, hemodilution, and bleeding.

A client has been taking isoniazid (INH) for tuberculosis for 3 weeks. What laboratory results need to be reported to the health care provider immediately?

a. Albumin: 5.1 g/dL b.Alanine aminotransferase (ALT): 180 U/L c. Red blood cell (RBC) count: 5.2/mm3 d.White blood cell (WBC) count: 12,500/mm3 ANS: B INH can cause liver damage, especially if the client drinks alcohol. The ALT (one of the liver enzymes) is extremely high and needs to be reported immediately. The albumin and RBCs are normal. The WBCs are slightly high, but that would be an expected finding in a client with an infection.

A client is being taught about drug therapy for Helicobacter pylori infection. What assessment by the nurse is most important?

a. Alcohol intake of 1 to 2 drinks per week b. Family history of H. pylori infection c. Former smoker still using nicotine patches d. Willingness to adhere to drug therapy ANS: D Treatment for this infection involves either triple or quadruple drug therapy, which may make it difficult for clients to remain adherent. The nurse should assess the clients willingness and ability to follow the regimen. The other assessment findings are not as critical.

11. A client asks the nurse about drugs for weight loss. What response by the nurse is best?

a. All weight-loss drugs can cause suicidal ideation. b. No drugs are currently available for weight loss. c. Only over-the-counter medications are available. d. There are three drugs currently approved for this. ANS: D There are three drugs available by prescription for weight loss, including orlistat (Xenical), lorcaserin (Belviq), and phentermine-topiramate (Qsymia). Suicidal thoughts are possible with lorcaserin and phenterminetopiramate. Orlistat is also available in a reduced-dose over-the-counter formulation.

A nurse assesses a client who was started on intraperitoneal therapy 5 days ago. The client reports abdominal pain and feeling warm. For which complication of this therapy should the nurse assess this client?

a. Allergic reaction b.Bowel obstruction c. Catheter lumen occlusion d.Infection ANS: D Fever, abdominal pain, abdominal rigidity, and rebound tenderness may be present in the client who has peritonitis related to intraperitoneal therapy. Peritonitis is preventable by using strict aseptic technique in handling all equipment and infusion supplies. An allergic reaction would occur earlier in the course of treatment. Bowel obstruction and catheter lumen occlusion can occur but would present clinically in different ways.

A client has been prescribed brinzolamide (Azopt). What assessment by the nurse requires consultation with the provider?

a. Allergy to eggs *b. Allergy to sulfonamides* c. Use of contact lenses d. Use of beta blockers ANS: B Brinzolamide is similar to sulfonamides, so an allergic reaction could occur. The other assessment findings are not related to brinzolamide.

A client is recovering from an esophagogastroduodenoscopy (EGD) and requests something to drink. What action by the nurse is best?

a. Allow the client cool liquids only. b. Assess the clients gag reflex. c. Remind the client to remain NPO. d. Tell the client to wait 4 hours. ANS: B The local anesthetic used during this procedure will depress the clients gag reflex. After the procedure, the nurse should ensure that the gag reflex is intact before offering food or fluids. The client does not need to be restricted to cool beverages only and is not required to wait 4 hours before oral intake is allowed. Telling the client to remain NPO does not inform the client of when he or she can have fluids, nor does it reflect the clients readiness for them

A client who is near blind is admitted to the hospital. What action by the nurse is most important?

a. Allow the client to feel his or her way around. b. Let the client arrange objects on the bedside table. *c. Orient the client to the room using a focal point*. d. Speak loudly and slowing when talking to the client. ANS: C Using a focal point, orient the client to the room by giving descriptions of items as they relate to the focal point. Letting the client arrange the bedside table is a good idea, but not as important as orienting the client to the room for safety. Allowing the client to just feel around may cause injury. Unless the client is also hearing impaired, use a normal tone of voice.

A nurse prepares to discharge a client with Alzheimers disease. Which statement should the nurse include in the discharge teaching for this clients caregiver?

a. Allow the client to rest most of the day. b. Place a padded throw rug at the bedside. *c. Install deadbolt locks on all outside doors*. d. Provide a high-calorie and high-protein diet. ANS: C Clients with Alzheimers disease have a tendency to wander, especially at night. If possible, alarms should be installed on all outside doors to alert family members if the client leaves. At a minimum, all outside doors should have deadbolt locks installed to prevent the client from going outdoors unsupervised. The client should be allowed to exercise within his or her limits. Throw rugs are a slip and fall hazard and should be removed. The client should eat a well-balanced diet. There is no need for a high-calorie or high-protein diet.

A nurse cares for a client who had a chest tube placed 6 hours ago and refuses to take deep breaths because of the pain. Which action should the nurse take?

a. Ambulate the client in the hallway to promote deep breathing. b. Auscultate the clients anterior and posterior lung fields. c. Encourage the client to take shallow breaths to help with the pain. d. Administer pain medication and encourage the client to take deep breaths. ANS: D A chest tube is placed in the pleural space and may be uncomfortable for a client. The nurse should provide pain medication to minimize discomfort and encourage the client to take deep breaths. The other responses do not address the clients discomfort and need to take deep breaths to prevent complications.

A nurse plans care for a client with Parkinson disease. Which intervention should the nurse include in this clients plan of care?

a. Ambulate the client in the hallway twice a day. b. Ensure a fluid intake of at least 3 liters per day. c. Teach the client pursed-lip breathing techniques. *d. Keep the head of the bed at 30 degrees or greater*. ANS: D Elevation of the head of the bed will help prevent aspiration. The other options will not prevent aspiration, which is the greatest respiratory complication of Parkinson disease, nor do these interventions address any of the complications of Parkinson disease. Ambulation in the hallway is usually implemented to prevent venous thrombosis. Increased fluid intake flushes out toxins from the clients blood. Pursed-lip breathing increases exhalation of carbon dioxide.

A client has a deep vein thrombosis (DVT). What comfort measure does the nurse delegate to the unlicensed assistive personnel (UAP)?

a. Ambulate the client. b. Apply a warm moist pack. c. Massage the clients leg. d. Provide an ice pack. ANS: B Warm moist packs will help with the pain of a DVT. Ambulation is not a comfort measure. Massaging the clients legs is contraindicated to prevent complications such as pulmonary embolism. Ice packs are not recommended for DVT.

A nurse is caring for a client with a deep vein thrombosis (DVT). What nursing assessment indicates a priority outcome has been met?

a. Ambulates with assistance b. Oxygen saturation of 98% c. Pain of 2/10 after medication d. Verbalizing risk factors ANS: B A critical complication of DVT is pulmonary embolism. A normal oxygen saturation indicates that this has not occurred. The other assessments are also positive, but not the priority.

.A nurse assesses a client who has a radial artery catheter. Which assessment should the nurse complete first?

a. Amount of pressure in fluid container b.Date of catheter tubing change c. Percent of heparin in infusion container d.Presence of an ulnar pulse ANS: D An intra-arterial catheter may cause arterial occlusion, which can lead to absent or decreased perfusion to the extremity. Assessment of an ulnar pulse is one way to assess circulation to the arm in which the catheter is located. The nurse would note that there is enough pressure in the fluid container to keep the system flushed, and would check to see whether the catheter tubing needs to be changed. However, these are not assessments of greatest concern. Because of heparin-induced thrombocytopenia, heparin is not used in most institutions for an arterial catheter.

A nurse assesses clients on a medical-surgical unit. Which client should the nurse identify as having the greatest risk for cardiovascular disease?

a. An 86-year-old man with a history of asthma b. A 32-year-old Asian-American man with colorectal cancer c. A 45-year-old American Indian woman with diabetes mellitus d. A 53-year-old postmenopausal woman who is on hormone therapy ANS: C The incidence of coronary artery disease and hypertension is higher in American Indians than in whites or Asian Americans. Diabetes mellitus increases the risk for hypertension and coronary artery disease in people of any race or ethnicity. Asthma, colorectal cancer, and hormone therapy do not increase risk for cardiovascular disease.

A client is awaiting bariatric surgery in the morning. What action by the nurse is most important?

a. Answering questions the client has about surgery b. Beginning venous thromboembolism prophylaxis c. Informing the client that he or she will be out of bed tomorrow d. Teaching the client about needed dietary changes ANS: B Morbidly obese clients are at high risk of venous thromboembolism and should be started on a regimen to prevent this from occurring as a priority. Answering questions about the surgery is done by the surgeon. Teaching is important, but safety comes first.

.The emergency department (ED) manager is reviewing client charts to determine how well the staff performs when treating clients with community-acquired pneumonia. What outcome demonstrates that goals for this client type have been met?

a. Antibiotics started before admission b.Blood cultures obtained within 20 minutes c. Chest x-ray obtained within 30 minutes d.Pulse oximetry obtained on all clients ANS: A Goals for treatment of community-acquired pneumonia include initiating antibiotics prior to inpatient admission or within 6 hours of presentation to the ED. Timely collection of blood cultures, chest x-ray, and pulse oximetry are important as well but do not coincide with established goals.

A client with Guillain-Barr syndrome is admitted to the hospital. The nurse plans caregiving priority to interventions that address which priority client problem?

a. Anxiety b. Low fluid volume *c. Inadequate airway* d. Potential for skin breakdown ANS: C Airway takes priority. Anxiety is probably present, but a physical diagnosis takes priority over a psychosocial one. The client has no reason to have low fluid volume unless he or she has been unable to drink for some time. If present, airway problems take priority over a circulation problem. An actual problem takes precedence over a risk for a problem.

A nurse is caring for a client with acute pericarditis who reports substernal precordial pain that radiates to the left side of the neck. Which nonpharmacologic comfort measure should the nurse implement?

a. Apply an ice pack to the clients chest. b. Provide a neck rub, especially on the left side. c. Allow the client to lie in bed with the lights down. d. Sit the client up with a pillow to lean forward on. ANS: D Pain from acute pericarditis may worsen when the client lays supine. The nurse should position the client in a comfortable position, which usually is upright and leaning slightly forward. Pain is decreased by using gravity to take pressure off the heart muscle. An ice pack and neck rub will not relieve this pain.

.A nurse assesses a clients peripheral IV site, and notices edema and tenderness above the site. Which action should the nurse take next?

a. Apply cold compresses to the IV site. b.Elevate the extremity on a pillow. c. Flush the catheter with normal saline. d.Stop the infusion of intravenous fluids. ANS: D Infiltration occurs when the needle dislodges partially or completely from the vein. Signs of infiltration include edema and tenderness above the site. The nurse should stop the infusion and remove the catheter. Cold compresses and elevation of the extremity can be done after the catheter is discontinued to increase client comfort. Alternatively, warm compresses may be prescribed per institutional policy and may help speed circulation to the area.

A client has a sickle cell crisis with extreme lower extremity pain. What comfort measure does the nurse delegate to the unlicensed assistive personnel (UAP)?

a. Apply ice packs to the clients legs. b. Elevate the clients legs on pillows. c. Keep the lower extremities warm. d. Place elastic bandage wraps on the clients legs. ANS: C During a sickle cell crisis, the tissue distal to the occlusion has decreased blood flow and ischemia, leading to pain. Due to decreased blood flow, the clients legs will be cool or cold. The UAP can attempt to keep the clients legs warm. Ice and elevation will further decrease perfusion. Elastic bandage wraps are not indicated and may constrict perfusion in the legs.

A nurse assesses a client with diabetes mellitus who self-administers subcutaneous insulin. The nurse notes a spongy, swelling area at the site the client uses most frequently for insulin injection. Which action should the nurse take?

a. Apply ice to the site to reduce inflammation. b.Consult the provider for a new administration route. c. Assess the client for other signs of cellulitis. *d.Instruct the client to rotate sites for insulin injection* ANS: D The clients tissue has been damaged from continuous use of the same site. The client should be educated to rotate sites. The damaged tissue is not caused by cellulitis or any type infection, and applying ice may cause more damage to the tissue. Insulin can only be administered subcutaneously and intravenously. It would not be appropriate or practical to change the administration route.

A nurse is caring for a client who has just experienced a 90-second tonic-clonic seizure. The clients arterial blood gas values are pH 6.88, PaO2 50 mm Hg, PaCO2 60 mm Hg, and HCO3 22 mEq/L. Which action should the nurse take first?

a. Apply oxygen by mask or nasal cannula. b. Apply a paper bag over the clients nose and mouth. c. Administer 50 mL of sodium bicarbonate intravenously. d. Administer 50 mL of 20% glucose and 20 units of regular insulin. ANS: A The client has experienced a combination of metabolic and acute respiratory acidosis through heavy skeletal muscle contractions and no gas exchange. When the seizures have stopped and the client can breathe again, the fastest way to return acid-base balance is to administer oxygen. Applying a paper bag over the clients nose and mouth would worsen the acidosis. Sodium bicarbonate should not be administered because the clients arterial bicarbonate level is normal. Glucose and insulin are administered together to decrease serum potassium levels. This action is not appropriate based on the information provided.

A nurse cares for an older adult client who has Salmonella food poisoning. The clients vital signs are heart rate: 102 beats/min, blood pressure: 98/55 mm Hg, respiratory rate: 22 breaths/min, and oxygen saturation: 92%. Which action should the nurse complete first?

a. Apply oxygen via nasal cannula. b. Administer intravenous fluids. c. Provide perineal care with a premedicated wipe. d. Teach proper food preparation to prevent contamination. ANS: B Dehydration caused by diarrhea can occur quickly in older clients with Salmonella food poisoning, so maintenance of fluid balance is a high priority. Monitoring vital signs and providing perineal care are important nursing actions but are of lower priority than fluid replacement. The nurse should teach the client about proper hand hygiene to prevent the spread of infection, and preparation of food and beverages to prevent contamination.

A client is receiving oxygen at 4 liters per nasal cannula. What comfort measure may the nurse delegate to unlicensed assistive personnel (UAP)?

a. Apply water-soluble ointment to nares and lips. b. Periodically turn the oxygen down or off. c. Remove the tubing from the clients nose. d. Turn the client every 2 hours or as needed. ANS: A Oxygen can be drying, so the UAP can apply water-soluble lubricant to the clients lips and nares. The UAP should not adjust the oxygen flow rate or remove the tubing. Turning the client is not related to comfort measures for oxygen.

A client has external otitis. On what comfort measure does the nurse instruct the client?

a. Applying ice four times a day b. Instilling vinegar-and-water drops *c. Use of a heating pad to the ear* d. Using a home humidifier ANS: C A heating pad on low or a warm moist pack can provide comfort to the client with otitis externa. The other options are not warranted.

A student is practicing suctioning a tracheostomy in the skills laboratory. What action by the student demonstrates that more teaching is needed?

a. Applying suction while inserting the catheter b. Preoxygenating the client prior to suctioning c. Suctioning for a total of three times if needed d. Suctioning for only 10 to 15 seconds each time ANS: A Suction should only be applied while withdrawing the catheter. The other actions are appropriate.

A client had a femoropopliteal bypass graft with a synthetic graft. What action by the nurse is most important to prevent wound infection?

a. Appropriate hand hygiene before giving care b. Assessing the clients temperature every 4 hours c. Clean technique when changing dressings d. Monitoring the clients daily white blood cell count ANS: A Hand hygiene is the best way to prevent infections in hospitalized clients. Dressing changes should be done with sterile technique. Assessing vital signs and white blood cell count will not prevent infection.

A client has been admitted for suspected inhalation anthrax infection. What question by the nurse is most important?

a. Are any family members also ill? b.Have you traveled recently? c. How long have you been ill? d.What is your occupation? ANS: D Inhalation anthrax is rare and is an occupational hazard among people who work with animal wool, bone meal, hides, and skin, such as taxidermists and veterinarians. Inhalation anthrax seen in someone without an occupational risk is considered a bioterrorism event and must be reported to authorities immediately. The other questions are appropriate for anyone with an infection.

The clinic nurse assesses a client with diabetes during a checkup. The client also has osteoarthritis (OA). The nurse notes the clients blood glucose readings have been elevated. What question by the nurse is most appropriate?

a. Are you compliant with following the diabetic diet? *b. Have you been taking glucosamine supplements?* c. How much exercise do you really get each week? d. Youre still taking your diabetic medication, right? ANS: B All of the topics are appropriate for a client whose blood glucose readings have been higher than usual. However, since this client also has OA, and glucosamine can increase blood glucose levels, the nurse should ask about its use. The other questions all have an element of nontherapeutic communication in them. Compliant is a word associated with negative images, and the client may deny being noncompliant. Asking how much exercise the client really gets is accusatory. Asking if the client takes his or her medications right? is patronizing.

An emergency department nurse assesses a client with a history of urinary incontinence who presents with extreme dry mouth, constipation, and an inability to void. Which question should the nurse ask first?

a. Are you drinking plenty of water? b. What medications are you taking? c. Have you tried laxatives or enemas? d. Has this type of thing ever happened before? ANS: B Some types of incontinence are treated with anticholinergic medications such as propantheline (Pro-Banthine). Anticholinergic side effects include dry mouth, constipation, and urinary retention. The nurse needs to assess the clients medication list to determine whether the client is taking an anticholinergic medication. If he or she is taking anticholinergics, the nurse should further assess the clients manifestations to determine if they are related to a simple side effect or an overdose. The other questions are not as helpful to understanding the current situation.

A nurse assesses the health history of a client who is prescribed ziconotide (Prialt) for chronic back pain. Which assessment question should the nurse ask?

a. Are you taking a nonsteroidal anti-inflammatory drug? *b. Do you have a mental health disorder?* c. Are you able to swallow medications? d. Do you smoke cigarettes or any illegal drugs? ANS: B Clients who have a mental health or behavioral health problem should not take ziconotide. The other questions do not identify a contraindication for this medication.

A nurse observes that a clients anteroposterior (AP) chest diameter is the same as the lateral chest diameter. Which question should the nurse ask the client in response to this finding?

a. Are you taking any medications or herbal supplements? b. Do you have any chronic breathing problems? c. How often do you perform aerobic exercise? d. What is your occupation and what are your hobbies? ANS: B The normal chest has a lateral diameter that is twice as large as the AP diameter. When the AP diameter approaches or exceeds the lateral diameter, the client is said to have a barrel chest. Most commonly, barrel chest occurs as a result of a long-term chronic airflow limitation problem, such as chronic obstructive pulmonary disease or severe chronic asthma. It can also be seen in people who have lived at a high altitude for many years. Therefore, an AP chest diameter that is the same as the lateral chest diameter should be rechecked but is not as indicative of underlying disease processes as an AP diameter that exceeds the lateral diameter. Medications, herbal supplements, and aerobic exercise are not associated with a barrel chest. Although occupation and hobbies may expose a client to irritants that can cause chronic lung disorders and barrel chest, asking about chronic breathing problems is more direct and should be asked first.

A client has dumping syndrome after a partial gastrectomy. Which action by the nurse would be most helpful?

a. Arrange a dietary consult. b. Increase fluid intake. c. Limit the clients foods. d. Make the client NPO. ANS: A The client with dumping syndrome after a gastrectomy has multiple dietary needs. A referral to the registered dietitian will be extremely helpful. Food and fluid intake is complicated and needs planning. The client should not be NPO.

A nurse is caring for a client who is about to receive a bone marrow transplant. To best help the client cope with the long recovery period, what action by the nurse is best?

a. Arrange a visitation schedule among friends and family. b. Explain that this process is difficult but must be endured. c. Help the client find things to hope for each day of recovery. d. Provide plenty of diversionary activities for this time. ANS: C Providing hope is an essential nursing function during treatment for any disease process, but especially during the recovery period after bone marrow transplantation, which can take up to 3 weeks. The nurse can help the client look ahead to the recovery period and identify things to hope for during this time. Visitors are important to clients, but may pose an infection risk. Telling the client the recovery period must be endured does not acknowledge his or her feelings. Diversionary activities are important, but not as important as instilling hope.

.A client is in the family practice clinic reporting a severe cough that has lasted for 5 weeks. The client is so exhausted after coughing that work has become impossible. What action by the nurse is most appropriate?

a. Arrange for immediate hospitalization. b.Facilitate polymerase chain reaction testing. c. Have the client produce a sputum sample. d.Obtain two sets of blood cultures. ANS: B Polymerase chain reaction testing is used to diagnose pertussis, which this client is showing manifestations of. Hospitalization may or may not be needed but is not the most important action. The client may or may not be able to produce sputum, but sputum cultures for this disease must be obtained via deep suctioning. Blood cultures will be negative.

A nurse attempted to assist a morbidly obese client back to bed and had immediate pain in the lower back. What action by the nurse is most appropriate?

a. Ask another nurse to help next time. b. Demand better equipment to use. c. Fill out and file a variance report. d. Refuse to assist the client again. ANS: C The nurse should complete a variance report per agency policy. Asking another nurse to help and requesting better equipment are both good ideas, but the nurse may have an injury that needs care. It would be unethical to refuse to care for this client again.

. A nurse is caring for an older adult client who is admitted with moderate dehydration. Which intervention should the nurse implement to prevent injury while in the hospital?

a. Ask family members to speak quietly to keep the client calm. b. Assess urine color, amount, and specific gravity each day. c. Encourage the client to drink at least 1 liter of fluids each shift. d. Dangle the client on the bedside before ambulating. ANS: D An older adult with moderate dehydration may experience orthostatic hypotension. The client should dangle on the bedside before ambulating. Although dehydration in an older adult may cause confusion, speaking quietly will not help the client remain calm or decrease confusion. Assessing the clients urine may assist with the diagnosis of dehydration but would not prevent injury. Clients are encouraged to drink fluids, but 1 liter of fluid each shift for an older adult may cause respiratory distress and symptoms of fluid overload, especially if the client has heart failure or renal insufficiency.

A nurse is working with a client who takes atorvastatin (Lipitor). The clients recent laboratory results include a blood urea nitrogen (BUN) of 33 mg/dL and creatinine of 2.8 mg/dL. What action by the nurse is best?

a. Ask if the client eats grapefruit. b. Assess the client for dehydration. c. Facilitate admission to the hospital. d. Obtain a random urinalysis. ANS: A There is a drug-food interaction between statins and grapefruit that can lead to acute kidney failure. This client has elevated renal laboratory results, indicating some degree of kidney involvement. The nurse should assess if the client eats grapefruit or drinks grapefruit juice. Dehydration can cause the BUN to be elevated, but the elevation in creatinine is more specific for a kidney injury. The client does not necessarily need to be admitted. A urinalysis may or may not be ordered.

A nurse assessing a client with colorectal cancer auscultates high-pitched bowel sounds and notes the presence of visible peristaltic waves. Which action should the nurse take?

a. Ask if the client is experiencing pain in the right shoulder. b. Perform a rectal examination and assess for polyps. c. Contact the provider and recommend computed tomography. d. Administer a laxative to increase bowel movement activity. ANS: C The presence of visible peristaltic waves, accompanied by high-pitched or tingling bowel sounds, is indicative of partial obstruction caused by the tumor. The nurse should contact the provider with these results and recommend a computed tomography scan for further diagnostic testing. This assessment finding is not associated with right shoulder pain; peritonitis and cholecystitis are associated with referred pain to the right shoulder. The registered nurse is not qualified to complete a rectal examination for polyps, and laxatives would not help this client.

A nurse is assessing an obese client in the clinic for follow-up after an episode of deep vein thrombosis. The client has lost 20 pounds since the last visit. What action by the nurse is best?

a. Ask if the weight loss was intended. b. Encourage a high-protein, high-fiber diet. c. Measure for new compression stockings. d. Review a 3-day food recall diary. ANS: C Compression stockings must fit correctly in order to work. After losing a significant amount of weight, the client should be re-measured and new stockings ordered if needed. The other options are appropriate, but not the most important.

A client is taking timolol (Timoptic) eyedrops. The nurse assesses the clients pulse at 48 beats/min. What action by the nurse is the priority?

a. Ask the client about excessive salivation. b. Assess the client for shortness of breath. c. Give the drops using punctal occlusion. *d. Hold the eyedrops and notify the provider.* ANS: D The nurse should hold the eyedrops and notify the provider because beta blockers can slow the heart rate. Excessive salivation can occur with cholinergic agonists. Shortness of breath is not related. If the drops are given, the nurse uses punctal occlusion to avoid systemic absorption

The family of a neutropenic client reports the client is not acting right. What action by the nurse is the priority?

a. Ask the client about pain. b. Assess the client for infection. c. Delegate taking a set of vital signs. d. Look at todays laboratory results. ANS: B Neutropenic clients often do not have classic manifestations of infection, but infection is the most common cause of death in neutropenic clients. The nurse should assess for infection. The nurse should assess for pain but this is not the priority. The nurse should take the clients vital signs instead of delegating them since the client has had a change in status. Laboratory results may be inconclusive.

A client presents to the family practice clinic reporting a week of watery, somewhat bloody diarrhea. The nurse assists the client to obtain a stool sample. What action by the nurse is most important?

a. Ask the client about recent exposure to illness. b. Assess the clients stool for obvious food particles. c. Include the date and time on the specimen container. d. Put on gloves prior to collecting the sample. ANS: D To avoid possible exposure to infectious agents, the nurse dons gloves prior to handling any bodily secretions. Recent exposure to illness is not related to collecting a stool sample. The nurse can visually inspect the stool for food particles, but it still needs analysis in the laboratory. The container should be dated and timed, but safety for the staff and other clients comes first.

A client scheduled for a percutaneous transhepatic cholangiography (PTC) denies allergies to medication. What action by the nurse is best?

a. Ask the client about shellfish allergies. b. Document this information on the chart. c. Ensure that the client has a ride home. d. Instruct the client on bowel preparation. ANS: A PTC uses iodinated dye, so the client should be asked about seafood allergies, specifically to shellfish. Documentation should occur, but this is not the priority. The client will need a ride home afterward if the procedure is done on an outpatient basis. There is no bowel preparation for PTC.

A client with human immune deficiency virus (HIV) has had a sudden decline in status with a large increase in viral load. What action should the nurse take first?

a. Ask the client about travel to any foreign countries. *b. Assess the client for adherence to the drug regimen*. c. Determine if the client has any new sexual partners. d. Request information about new living quarters or pets. ANS: B Adherence to the complex drug regimen needed for HIV treatment can be daunting. Clients must take their medications on time and correctly at a minimum of 90% of the time. Since this clients viral load has increased dramatically, the nurse should first assess this factor. After this, the other assessments may or may not be needed.

A client is in the family practice clinic. Today the client weighs 186.4 pounds (84.7 kg). Six months ago the client weighed 211.8 pounds (96.2 kg). What action by the nurse is best?

a. Ask the client if the weight loss was intentional. b. Determine if there are food allergies or intolerances. c. Perform a comprehensive nutritional assessment. d. Perform a rapid bedside blood glucose test. ANS: A This client has had a 12% weight loss. The nurse first determines if the weight loss was intentional. If not, then the nurse proceeds to a comprehensive nutritional assessment. Food intolerances are part of this assessment. Depending on risk factors and other findings, a blood glucose test may be warranted.

A client had a colonoscopy and biopsy yesterday and calls the gastrointestinal clinic to report a spot of bright red blood on the toilet paper today. What response by the nurse is best?

a. Ask the client to call back if this happens again today. b. Instruct the client to go to the emergency department. c. Remind the client that a small amount of bleeding is possible. d. Tell the client to come in to the clinic this afternoon. ANS: C After a colonoscopy with biopsy, a small amount of bleeding is normal. The nurse should remind the client of this and instruct him or her to go to the emergency department for large amounts of bleeding, severe pain, or dizziness.

A nurse cares for a client after radiation therapy for lung cancer. The client reports a sore throat. Which action should the nurse take first?

a. Ask the client to gargle with mouthwash containing lidocaine. b. Administer prescribed intravenous pain medications. c. Explain that soreness is normal and will improve in a couple days. d. Assess the clients neck for redness and swelling. ANS: A Mouthwashes and throat sprays containing a local anesthetic agent such as lidocaine or diphenhydramine can provide relief from a sore throat after radiation therapy. Intravenous pain medications may be used if local anesthetics are unsuccessful. The nurse should explain to the client that this is normal and assess the clients neck, but these options do not decrease the clients discomfort.

.A client is in the family medicine clinic reporting a dry, sore throat. The provider asks the nurse to assess for odynophagia. What assessment technique is most appropriate?

a. Ask the client what foods cause trouble swallowing. b.Assess the client for pain when swallowing. c. Determine if the client can swallow saliva. d.Palpate the clients jaw while swallowing. ANS: B Odynophagia is painful swallowing. The nurse should assess the client for this either by asking or by having the client attempt to drink water. It is not related to specific foods and is not assessed by palpating the jaw. Being unable to swallow saliva is not odynophagia, but it would be a serious situation.

A client is in the preoperative holding area waiting for cataract surgery. The client says Oh, yeah, I forgot to tell you that I take clopidogrel, or Plavix. What action by the nurse is most important?

a. Ask the client when the last dose was. b. Check results of the prothrombin time (PT) and international normalized ratio (INR). c. Document the information in the chart. *d. Notify the surgeon immediately*. ANS: D Clopidogrel is an antiplatelet aggregate and could increase bleeding. The surgeon should be notified immediately. The nurse should find out when the last dose of the drug was, but the priority is to notify the provider. This drug is not monitored with PT and INR. Documentation should occur but is not the priority.

An older client has gastric cancer and is scheduled to have a partial gastrectomy. The family does not want the client told about her diagnosis. What action by the nurse is best?

a. Ask the family why they feel this way. b. Assess family concerns and fears. c. Refuse to go along with the familys wishes. d. Tell the family that such secrets cannot be kept. ANS: B The nurse should use open-ended questions and statements to fully assess the familys concerns and fears. Asking why questions often puts people on the defensive and is considered a barrier to therapeutic communication. Refusing to follow the familys wishes or keep their confidence will not help move this family from their position and will set up an adversarial relationship.

.A client has been hospitalized with tuberculosis (TB). The clients spouse is fearful of entering the room where the client is in isolation and refuses to visit. What action by the nurse is best?

a. Ask the spouse to explain the fear of visiting in further detail. b.Inform the spouse the precautions are meant to keep other clients safe. c. Show the spouse how to follow the isolation precautions to avoid illness. d.Tell the spouse that he or she has already been exposed, so its safe to visit. ANS: A The nurse needs to obtain further information about the spouses specific fears so they can be addressed. This will decrease stress and permit visitation, which will be beneficial for both client and spouse. Precautions for TB prevent transmission to all who come into contact with the client. Explaining isolation precautions and what to do when entering the room will be helpful, but this is too narrow in scope to be the best answer. Telling the spouse its safe to visit is demeaning of the spouses feelings.

A nurse is teaching a client about magnesium hydroxide with aluminum hydroxide (Maalox). What instruction is most appropriate?

a. Aspirin must be avoided. b. Do not worry about black stools. c. Report diarrhea to your provider. d. Take 1 hour before meals. ANS: C Maalox can cause hypermagnesemia, which causes diarrhea, so the client should be taught to report this to the provider. Aspirin is avoided with bismuth sulfate (Pepto-Bismol). Black stools can be caused by Pepto-Bismol. Maalox should be taken after meals.

A nurse cares for a client who is scheduled for a total laryngectomy. Which action should the nurse take prior to surgery?

a. Assess airway patency, breathing, and circulation. b. Administer prescribed intravenous pain medication. c. Assist the client to choose a communication method. d. Ambulate the client in the hallway to assess gait. ANS: C The client will not be able to speak after surgery. The nurse should assist the client to choose a communication method that he or she would like to use after surgery. Assessing the clients airway and administering IV pain medication are done after the procedure. Although ambulation promotes health and decreases the complications of any surgery, this clients gait should not be impacted by a total laryngectomy and therefore is not a priority.

A nurse cares for a client who is on a cardiac monitor. The monitor displayed the rhythm shown below: Which action should the nurse take first?

a. Assess airway, breathing, and level of consciousness. b. Administer an amiodarone bolus followed by a drip. c. Cardiovert the client with a biphasic defibrillator. d. Begin cardiopulmonary resuscitation (CPR). ANS: A Ventricular tachycardia occurs with repetitive firing of an irritable ventricular ectopic focus, usually at a rate of 140 to 180 beats/min or more. Ventricular tachycardia is a lethal dysrhythmia. The nurse should first assess if the client is alert and breathing. Then the nurse should call a Code Blue and begin CPR. If this client is pulseless, the treatment of choice is defibrillation. Amiodarone is the antidysrhythmic of choice, but it is not the first action.

A client is having a bone marrow biopsy and is extremely anxious. What action by the nurse is best?

a. Assess client fears and coping mechanisms. b. Reassure the client this is a common test. c. Sedate the client prior to the procedure. d. Tell the client he or she will be asleep. ANS: A Assessing the clients specific fears and coping mechanisms helps guide the nurse in providing holistic care that best meets the clients needs. Reassurance will be helpful but is not the best option. Sedation is usually used. The client may or may not be totally asleep during the procedure.

A nurse assesses a client who is admitted with an acid-base imbalance. The clients arterial blood gas values are pH 7.32, PaO2 85 mm Hg, PaCO2 34 mm Hg, and HCO3 16 mEq/L. What action should the nurse take next?

a. Assess clients rate, rhythm, and depth of respiration. b. Measure the clients pulse and blood pressure. c. Document the findings and continue to monitor. d. Notify the physician as soon as possible. ANS: A Progressive skeletal muscle weakness is associated with increasing severity of acidosis. Muscle weakness can lead to severe respiratory insufficiency. Acidosis does lead to dysrhythmias (due to hyperkalemia), but these would best be assessed with cardiac monitoring. Findings should be documented, but simply continuing to monitor is not sufficient. Before notifying the physician, the nurse must have more data to report.

A nurse auscultates a clients lung fields. Which action should the nurse take based on the lung sounds? (Click the media button to hear the audio clip.)

a. Assess for airway obstruction. b. Initiate oxygen therapy. c. Assess vital signs. d. Elevate the clients head. ANS: A Stridor is the sound heard, and it indicates severe airway constriction. The nurse must administer a bronchodilator to get air into the lungs. Administering oxygen, assessing vital signs, and elevating the clients head will not help until the clients airways are open.

While assessing a client who is 12 hours postoperative after a thoracotomy for lung cancer, a nurse notices that the lower chest tube is dislodged. Which action should the nurse take first?

a. Assess for drainage from the site. b. Cover the insertion site with sterile gauze. c. Contact the provider and obtain a suture kit. d. Reinsert the tube using sterile technique. ANS: B Immediately covering the insertion site helps prevent air from entering the pleural space and causing a pneumothorax. The area will not reseal quickly enough to prevent air from entering the chest. The nurse should not leave the client to obtain a suture kit. An occlusive dressing may cause a tension pneumothorax. The site should only be assessed after the insertion site is covered. The provider should be called to reinsert the chest tube or prescribe other treatment options.

A nurse cares for a client who is prescribed pioglitazone (Actos). After 6 months of therapy, the client reports that his urine has become darker since starting the medication. Which action should the nurse take?

a. Assess for pain or burning with urination. *b.Review the clients liver function study results*. c. Instruct the client to increase water intake. d.Test a sample of urine for occult blood. ANS: B Thiazolidinediones (including pioglitazone) can affect liver function; liver function should be assessed at the start of therapy and at regular intervals while the client continues to take these drugs. Dark urine is one indicator of liver impairment because bilirubin is increased in the blood and is excreted in the urine. The nurse should check the clients most recent liver function studies. The nurse does not need to assess for pain or burning with urination and does not need to check the urine for occult blood. The client does not need to be told to increase water intake.

4. While assessing a client on a cardiac unit, a nurse identifies the presence of an S3 gallop. Which action should the nurse take next?

a. Assess for symptoms of left-sided heart failure. b. Document this as a normal finding. c. Call the health care provider immediately. d. Transfer the client to the intensive care unit. ANS: A The presence of an S3 gallop is an early diastolic filling sound indicative of increasing left ventricular pressure and left ventricular failure. The other actions are not warranted.

An emergency room nurse initiates care for a client with a cervical spinal cord injury who arrives via emergency medical services. Which action should the nurse take first?

a. Assess level of consciousness. b. Obtain vital signs. c. Administer oxygen therapy. *d. Evaluate respiratory status* ANS: D The first priority for a client with a spinal cord injury is assessment of respiratory status and airway patency. Clients with cervical spine injuries are particularly prone to respiratory compromise and may even require intubation. The other assessments should be performed after airway and breathing are assessed

A nurse assesses a client with Alzheimers disease who is recently admitted to the hospital. Which psychosocial assessment should the nurse complete?

a. Assess religious and spiritual needs while in the hospital. b. Identify the clients ability to perform self-care activities. *c. Evaluate the clients reaction to a change of environment*. d. Ask the client about relationships with family members. ANS: C As Alzheimers disease progresses, the client experiences changes in emotional and behavioral affect. The nurse should be alert to the clients reaction to a change in environment, such as being hospitalized, because the client may exhibit an exaggerated response, such as aggression, to the event. The other assessments should be completed but are not as important as assessing the clients reaction to environmental change.

A client is in the preoperative holding area prior to surgery. The nurse notes that the client has allergies to avocados and strawberries. What action by the nurse is best?

a. Assess that the client has been NPO as directed. b. Communicate this information with dietary staff. c. Document the information in the clients chart. *d. Ensure the information is relayed to the surgical team*. ANS: D A client with allergies to avocados, strawberries, bananas, or nuts has a higher risk of latex allergy. The nurse should ensure that the surgical staff is aware of this so they can provide a latex-free environment. Ensuring the clients NPO status is important for a client having surgery but is not directly related to the risk of latex allergy. Dietary allergies will be communicated when a diet order is placed. Documentation should be thorough but does not take priority.

A client is receiving total parenteral nutrition (TPN). On assessment, the nurse notes the clients pulse is 128 beats/min, blood pressure is 98/56 mm Hg, and skin turgor is dry. What action should the nurse perform next?

a. Assess the 24-hour fluid balance. b. Assess the clients oral cavity. c. Prepare to hang a normal saline bolus. d. Turn up the infusion rate of the TPN. ANS: A This client has clinical indicators of dehydration, so the nurse calculates the clients 24-hour intake, output, and fluid balance. This information is then reported to the provider. The clients oral cavity assessment may or may not be consistent with dehydration. The nurse may need to give the client a fluid bolus, but not as an independent action. The clients dehydration is most likely due to fluid shifts from the TPN, so turning up the infusion rate would make the problem worse, and is not done as an independent action.

A nurse evaluates a clients arterial blood gas values (ABGs): pH 7.30, PaO2 86 mm Hg, PaCO2 55 mm Hg, and HCO3 22 mEq/L. Which intervention should the nurse implement first?

a. Assess the airway. b. Administer prescribed bronchodilators. c. Provide oxygen. d. Administer prescribed mucolytics. ANS: A All interventions are important for clients with respiratory acidosis; this is indicated by the ABGs. However, the priority is assessing and maintaining an airway. Without a patent airway, other interventions will not be helpful

A client with a bleeding gastric ulcer is having a nuclear medicine scan. What action by the nurse is most appropriate?

a. Assess the client for iodine or shellfish allergies. b. Educate the client on the side effects of sedation. c. Inform the client a second scan may be needed. d. Teach the client about bowel preparation for the scan. ANS: C A second scan may be performed in 1 to 2 days to see if interventions have worked. The nuclear medicine scan does not use iodine-containing contrast dye or sedation. There is no required bowel preparation.

A client is admitted with suspected pneumonia from the emergency department. The client went to the primary care provider a few days ago and shows the nurse the results of what the client calls an allergy test, as shown below: What action by the nurse takes priority?

a. Assess the client for possible items to which he or she is allergic. b.Call the primary care providers office to request records. c. Immediately place the client on Airborne Precautions. d.Prepare to begin administration of intravenous antibiotics. ANS: C This allergy test is actually a positive tuberculosis test. The client should be placed on Airborne Precautions immediately. The other options do not take priority over preventing the spread of the disease.

A client is having a radioisotopic imaging scan. What action by the nurse is most important?

a. Assess the client for shellfish allergies. b. Place the client on radiation precautions. c. Sedate the client before the scan. d. Teach the client about the procedure. ANS: D The nurse should ensure that teaching is done and the client understands the procedure. Contrast dye is not used, so shellfish/iodine allergies are not related. The client will not be radioactive and does not need radiation precautions. Sedation is not used in this procedure.

The nurse is assessing a client on admission to the hospital. The clients leg appears as shown below: What action by the nurse is best?

a. Assess the clients ankle-brachial index. b. Elevate the clients leg above the heart. c. Obtain an ice pack to provide comfort. d. Prepare to teach about heparin sodium. ANS: A This client has dependent rubor, a classic finding in peripheral arterial disease. The nurse should measure the clients ankle-brachial index. Elevating the leg above the heart will further impede arterial blood flow. Ice will cause vasoconstriction, also impeding circulation and perhaps causing tissue injury. Heparin sodium is not the drug of choice for this condition.

A client having severe allergy symptoms has received several doses of IV antihistamines. What action by the nurse is most important?

a. Assess the clients bedside glucose reading. *b. Instruct the client not to get up without help.* c. Monitor the client frequently for tachycardia. d. Record the clients intake, output, and weight. ANS: B Antihistamines can cause drowsiness, so for the clients safety, he or she should be instructed to call for assistance prior to trying to get up. Hyperglycemia and tachycardia are side effects of sympathomimetics. Fluid and sodium retention are side effects of corticosteroids.

A client is in the bariatric clinic 1 month after having gastric bypass surgery. The client is crying and says I didnt know it would be this hard to live like this. What response by the nurse is best?

a. Assess the clients coping and support systems. b. Inform the client that things will get easier. c. Re-educate the client on needed dietary changes. d. Tell the client lifestyle changes are always hard. ANS: A The nurse should assess this clients coping styles and support systems in order to provide holistic care. The other options do not address the clients distress.

A nurse assesses a client who is recovering from a hemorrhoidectomy that was done the day before. The nurse notes that the client has lower abdominal distention accompanied by dullness to percussion over the distended area. Which action should the nurse take?

a. Assess the clients heart rate and blood pressure. b. Determine when the client last voided. c. Ask if the client is experiencing flatus. d. Auscultate all quadrants of the clients abdomen. ANS: B Assessment findings indicate that the client may have an over-full bladder. In the immediate postoperative period, the client may experience difficulty voiding due to urinary retention. The nurse should assess when the client last voided. The clients vital signs may be checked after the nurse determines the clients last void. Asking about flatus and auscultating bowel sounds are not related to a hemorrhoidectomy.

A client with a history of heart failure and hypertension is in the clinic for a follow-up visit. The client is on lisinopril (Prinivil) and warfarin (Coumadin). The client reports new-onset cough. What action by the nurse is most appropriate?

a. Assess the clients lung sounds and oxygenation. b. Instruct the client on another antihypertensive. c. Obtain a set of vital signs and document them. d. Remind the client that cough is a side effect of Prinivil. ANS: A This client could be having an exacerbation of heart failure or be experiencing a side effect of lisinopril (and other angiotensin-converting enzyme inhibitors). The nurse should assess the clients lung sounds and other signs of oxygenation first. The client may or may not need to switch antihypertensive medications. Vital signs and documentation are important, but the nurse should assess the respiratory system first. If the cough turns out to be a side effect, reminding the client is appropriate, but then more action needs to be taken.

An unlicensed assistive personnel (UAP) was feeding a client with a tracheostomy. Later that evening, the UAP reports that the client had a coughing spell during the meal. What action by the nurse takes priority?

a. Assess the clients lung sounds. b. Assign a different UAP to the client. c. Report the UAP to the manager. d. Request thicker liquids for meals. ANS: A The priority is to check the clients oxygenation because he or she may have aspirated. Once the client has been assessed, the nurse can consult with the registered dietitian about appropriately thickened liquids. The UAP should have reported the incident immediately, but addressing that issue is not the immediate priority.

A client is receiving an infusion of alteplase (Activase) for an intra-arterial clot. The client begins to mumble and is disoriented. What action by the nurse takes priority?

a. Assess the clients neurologic status. b. Notify the Rapid Response Team. c. Prepare to administer vitamin K. d. Turn down the infusion rate. ANS: B Clients on fibrinolytic therapy are at high risk of bleeding. The sudden onset of neurologic signs may indicate the client is having a hemorrhagic stroke. The nurse does need to complete a thorough neurological examination, but should first call the Rapid Response Team based on the clients manifestations. The nurse notifies the Rapid Response Team first. Vitamin K is not the antidote for this drug. Turning down the infusion rate will not be helpful if the client is still receiving any of the drug.

A client is wearing a Venturi mask to deliver oxygen and the dinner tray has arrived. What action by the nurse is best?

a. Assess the clients oxygen saturation and, if normal, turn off the oxygen. b. Determine if the client can switch to a nasal cannula during the meal. c. Have the client lift the mask off the face when taking bites of food. d. Turn the oxygen off while the client eats the meal and then restart it. ANS: B Oxygen is a drug that needs to be delivered constantly. The nurse should determine if the provider has approved switching to a nasal cannula during meals. If not, the nurse should consult with the provider about this issue. The oxygen should not be turned off. Lifting the mask to eat will alter the FiO2 delivered.

At 4:45 p.m., a nurse assesses a client with diabetes mellitus who is recovering from an abdominal hysterectomy 2 days ago. The nurse notes that the client is confused and diaphoretic. The nurse reviews the assessment data provided in the chart below: Capillary Blood Glucose Testing (AC/HS) Dietary Intake At 0630: 95 At 1130: 70 At 1630: 47 Breakfast: 10% eaten client states she is not hungry Lunch: 5% eaten client is nauseous; vomits once After reviewing the clients assessment data, which action is appropriate at this time?

a. Assess the clients oxygen saturation level and administer oxygen. b.Reorient the client and apply a cool washcloth to the clients forehead. *c. Administer dextrose 50% intravenously and reassess the client*. d.Provide a glass of orange juice and encourage the client to eat dinner. ANS: C The clients symptoms are related to hypoglycemia. Since the client has not been tolerating food, the nurse should administer dextrose intravenously. The clients oxygen level could be checked, but based on the information provided, this is not the priority. The client will not be reoriented until the glucose level rises.

The nurse assesses the client using the device pictured below to deliver 50% O2: The nurse finds the mask fits snugly, the skin under the mask and straps is intact, and the flow rate of the oxygen is 3 L/min. What action by the nurse is best?

a. Assess the clients oxygen saturation. b. Document these findings in the chart. c. Immediately increase the flow rate. d. Turn the flow rate down to 2 L/min. ANS: C For the Venturi mask to deliver high flow of oxygen, the flow rate must be set correctly, usually between 4 and 10 L/min. The clients flow rate is too low and the nurse should increase it. After increasing the flow rate, the nurse assesses the oxygen saturation and documents the findings.

A client has a tracheostomy that is 3 days old. Upon assessment, the nurse notes the clients face is puffy and the eyelids are swollen. What action by the nurse takes priority?

a. Assess the clients oxygen saturation. b. Notify the Rapid Response Team. c. Oxygenate the client with a bag-valve-mask. d. Palpate the skin of the upper chest. ANS: A This client may have subcutaneous emphysema, which is air that leaks into the tissues surrounding the tracheostomy. The nurse should first assess the clients oxygen saturation and other indicators of oxygenation. If the client is stable, the nurse can palpate the skin of the upper chest to feel for the air. If the client is unstable, the nurse calls the Rapid Response Team. Using a bag-valve-mask device may or may not be appropriate for the unstable client.

A nurse cares for a client who has packing inserted for posterior nasal bleeding. Which action should the nurse take first?

a. Assess the clients pain level. b. Keep the clients head elevated. c. Teach the client about the causes of nasal bleeding. d. Make sure the string is taped to the clients cheek. ANS: D The string should be attached to the clients cheek to hold the packing in place. The nurse needs to make sure that this does not move because it can occlude the clients airway. The other options are good interventions, but ensuring that the airway is patent is the priority objective.

A client just returned to the surgical unit after a gastric bypass. What action by the nurse is the priority?

a. Assess the clients pain. b. Check the surgical incision. c. Ensure an adequate airway. d. Program the morphine pump. ANS: C All actions are appropriate care measures for this client; however, airway is always the priority. Bariatric clients tend to have short, thick necks that complicate airway management.

A morbidly obese client is admitted to a community hospital that does not typically care for bariatric-sized clients. What action by the nurse is most appropriate?

a. Assess the clients readiness to make lifestyle changes. b. Ensure adequate staff when moving the client. c. Leave siderails down to prevent pressure ulcers. d. Reinforce the need to be sensitive to the client. ANS: B Many hospitals that see bariatric-sized clients have appropriate equipment for this population. A hospital that does not typically see these clients is less likely to have appropriate equipment, putting staff and client safety at risk. The nurse ensures enough staffing is available to help with all aspects of mobility. It may or may not be appropriate to assess the clients willingness to make lifestyle changes. Leaving the siderails down may present a safety hazard. The staff should be sensitive to this clients situation, but safety takes priority.

A nurse is assessing a client with hypokalemia, and notes that the clients handgrip strength has diminished since the previous assessment 1 hour ago. Which action should the nurse take first?

a. Assess the clients respiratory rate, rhythm, and depth. b. Measure the clients pulse and blood pressure. c. Document findings and monitor the client. d. Call the health care provider. ANS: A In a client with hypokalemia, progressive skeletal muscle weakness is associated with increasing severity of hypokalemia. The most life-threatening complication of hypokalemia is respiratory insufficiency. It is imperative for the nurse to perform a respiratory assessment first to make sure that the client is not in immediate jeopardy. Cardiac dysrhythmias are also associated with hypokalemia. The clients pulse and blood pressure should be assessed after assessing respiratory status. Next, the nurse would call the health care provider to obtain orders for potassium replacement. Documenting findings and continuing to monitor the client should occur during and after potassium replacement therapy.

A nurse admits a client who is experiencing an exacerbation of heart failure. Which action should the nurse take first?

a. Assess the clients respiratory status. b. Draw blood to assess the clients serum electrolytes. c. Administer intravenous furosemide (Lasix). d. Ask the client about current medications. ANS: A Assessment of respiratory and oxygenation status is the priority nursing intervention for the prevention of complications. Monitoring electrolytes, administering diuretics, and asking about current medications are important but do not take priority over assessing respiratory status.

A client has hypertension and high risk factors for cardiovascular disease. The client is overwhelmed with the recommended lifestyle changes. What action by the nurse is best?

a. Assess the clients support system. b. Assist in finding one change the client can control. c. Determine what stressors the client faces in daily life. d. Inquire about delegating some of the clients obligations. ANS: B All options are appropriate when assessing stress and responses to stress. However, this client feels overwhelmed by the suggested lifestyle changes. Instead of looking at all the needed changes, the nurse should assist the client in choosing one the client feels optimistic about controlling. Once the client has mastered that change, he or she can move forward with another change. Determining support systems, daily stressors, and delegation opportunities does not directly impact the clients feelings of control.

A nurse is caring for a client after joint replacement surgery. What action by the nurse is most important to prevent wound infection?

a. Assess the clients white blood cell count. b. Culture any drainage from the wound. c. Monitor the clients temperature every 4 hours. *d. Use aseptic technique for dressing changes*. ANS: D Preventing surgical wound infection is a primary responsibility of the nurse, who must use aseptic technique to change dressings or empty drains. The other actions do not prevent infection but can lead to early detection of an infection that is already present

A nurse is assessing a dark-skinned client for pallor. What action is best?

a. Assess the conjunctiva of the eye. b. Have the client open the hand widely. c. Look at the roof of the clients mouth. d. Palpate for areas of mild swelling. ANS: A To assess pallor in dark-skinned people, assess the conjunctiva of the eye or the mucous membranes. Looking at the roof of the mouth can reveal jaundice. Opening the hand widely is not related to pallor, nor is palpating for mild swelling.

A client has a continuous passive motion (CPM) device after a total knee replacement. What action does the nurse delegate to the unlicensed assistive personnel (UAP) after the affected leg is placed in the machine while the client is in bed?

a. Assess the distal circulation in 30 minutes. b. Change the settings based on range of motion. *c. Raise the lower siderail on the affected side.* d. Remind the client to do quad-setting exercises. ANS: C Because the clients leg is strapped into the CPM, if it falls off the bed due to movement, the clients leg (and new joint) can be injured. The nurse should instruct the UAP to raise the siderail to prevent this from occurring. Assessment is a nursing responsibility. Only the surgeon, physical therapist, or specially trained technician adjusts the CPM settings. Quad-setting exercises are not related to the CPM machine.

A client has been diagnosed with a deep vein thrombosis and is to be discharged on warfarin (Coumadin). The client is adamant about refusing the drug because its dangerous. What action by the nurse is best?

a. Assess the reason behind the clients fear. b. Remind the client about laboratory monitoring. c. Tell the client drugs are safer today than before. d. Warn the client about consequences of noncompliance. ANS: A The first step is to assess the reason behind the clients fear, which may be related to the experience of someone the client knows who took warfarin. If the nurse cannot address the specific rationale, teaching will likely be unsuccessful. Laboratory monitoring once every few weeks may not make the client perceive the drug to be safe. General statements like drugs are safer today do not address the root cause of the problem. Warning the client about possible consequences of not taking the drug is not therapeutic and is likely to lead to an adversarial relationship.

A client with Mnires disease is in the hospital when the client has an attack of this disorder. What action by the nurse takes priority?

a. Assess vital signs every 15 minutes. b. Dim or turn off lights in the clients room. *c. Place the client in bed with the upper siderails up*. d. Provide a cool, wet cloth for the clients face. ANS: C Clients with Mnires disease can have vertigo so severe that they can fall. The nurse should assist the client into bed and put the siderails up to keep the client from falling out of bed due to the intense whirling feeling. The other actions are not warranted for clients with Mnires disease.

A nursing student is caring for a client with an abdominal aneurysm. What action by the student requires the registered nurse to intervene?

a. Assesses the client for back pain b. Auscultates over abdominal bruit c. Measures the abdominal girth d. Palpates the abdomen in four quadrants ANS: D Abdominal aneurysms should never be palpated as this increases the risk of rupture. The registered nurse should intervene when the student attempts to do this. The other actions are appropriate.

A client is receiving total parenteral nutrition (TPN). What action by the nurse is most important?

a. Assessing blood glucose as directed b. Changing the IV dressing each day c. Checking the TPN with another nurse d. Performing appropriate hand hygiene ANS: D Clients on TPN are at high risk for infection. The nurse performs appropriate hand hygiene as a priority intervention. Checking blood glucose is also an important measure, but preventing infection takes priority. The IV dressing is changed every 48 to 72 hours. TPN does not need to be double-checked with another nurse.

A student nurse is assessing the peripheral vascular system of an older adult. What action by the student would cause the faculty member to intervene?

a. Assessing blood pressure in both upper extremities b. Auscultating the carotid arteries for any bruits c. Classifying capillary refill of 4 seconds as normal d. Palpating both carotid arteries at the same time ANS: D The student should not compress both carotid arteries at the same time to avoid brain ischemia. Blood pressure should be taken and compared in both arms. Prolonged capillary refill is considered to be greater than 5 seconds in an older adult, so classifying refill of 4 seconds as normal would not require intervention. Bruits should be auscultated.

A nursing student is instructed to remove a clients ear packing and instill eardrops. What action by the student requires intervention by the registered nurse?

a. Assessing the eardrum with an otoscope b. Inserting a cotton ball in the ear after the drops c. Warming the eardrops in water for 5 minutes *d. Washing the hands and removing the packing* ANS: D The student should wash his or her hands, don gloves, and then remove the packing. The other actions are correct.

A nurse answers a clients call light and finds the client in the bathroom, vomiting large amounts of bright red blood. Which action should the nurse take first?

a. Assist the client back to bed. b. Notify the provider immediately. c. Put on a pair of gloves. d. Take a set of vital signs. ANS: C All of the actions are appropriate; however, the nurse should put on a pair of gloves first to avoid contamination with blood or body fluids.

A client with rheumatoid arthritis (RA) is on the postoperative nursing unit after having elective surgery. The client reports that one arm feels like pins and needles and that the neck is very painful since returning from surgery. What action by the nurse is best?

a. Assist the client to change positions. b. Document the findings in the clients chart. c. Encourage range of motion of the neck. *d. Notify the provider immediately.* ANS: D Clients with RA can have cervical joint involvement. This can lead to an emergent situation in which the phrenic nerve is compressed, causing respiratory insufficiency. The client can also suffer a permanent spinal cord injury. The nurse needs to notify the provider immediately. Changing positions and doing range of motion may actually worsen the situation. The nurse should document findings after notifying the provider.

A client has frequent hospitalizations for leukemia and is worried about functioning as a parent to four small children. What action by the nurse would be most helpful?

a. Assist the client to make sick day plans for household responsibilities. b. Determine if there are family members or friends who can help the client. c. Help the client inform friends and family that they will have to help out. d. Refer the client to a social worker in order to investigate respite child care. ANS: A While all options are reasonable choices, the best option is to help the client make sick day plans, as that is more comprehensive and inclusive than the other options, which focus on a single item.

A nurse plans care for a client who is experiencing dyspnea and must stop multiple times when climbing a flight of stairs. Which intervention should the nurse include in this clients plan of care?

a. Assistance with activities of daily living b. Physical therapy activities every day c. Oxygen therapy at 2 liters per nasal cannula d. Complete bedrest with frequent repositioning ANS: A A client with dyspnea and difficulty completing activities such as climbing a flight of stairs has class III dyspnea. The nurse should provide assistance with activities of daily living. These clients should be encouraged to participate in activities as tolerated. They should not be on complete bedrest, may not be able to tolerate daily physical therapy, and only need oxygen if hypoxia is present.

After teaching a young adult client who is newly diagnosed with type 1 diabetes mellitus, the nurse assesses the clients understanding. Which statement made by the client indicates a correct understanding of the need for eye examinations?

a. At my age, I should continue seeing the ophthalmologist as I usually do. b.I will see the eye doctor when I have a vision problem and yearly after age 40 c. My vision will change quickly. I should see the ophthalmologist twice a year. *d.Diabetes can cause blindness, so I should see the ophthalmologist yearly*. ANS: D Diabetic retinopathy is a leading cause of blindness in North America. All clients with diabetes, regardless of age, should be examined by an ophthalmologist (rather than an optometrist or optician) at diagnosis and at least yearly thereafter.

A nurse cares for a client who is experiencing status epilepticus. Which prescribed medication should the nurse prepare to administer?

a. Atenolol (Tenormin) *b. Lorazepam (Ativan*) c. Phenytoin (Dilantin) d. Lisinopril (Prinivil) ANS: B Initially, intravenous lorazepam is administered to stop motor movements. This is followed by the administration of phenytoin. Atenolol, a beta blocker, and lisinopril, an angiotensin-converting enzyme inhibitor, are not administered for seizure activity. These medications are typically administered for hypertension and heart failure

A nurse assesses a client with a history of epilepsy who experiences stiffening of the muscles of the arms and legs, followed by an immediate loss of consciousness and jerking of all extremities. How should the nurse document this activity?

a. Atonic seizure *b. Tonic-clonic seizure* c. Myoclonic seizure d. Absence seizure ANS: B Seizure activity that begins with stiffening of the arms and legs, followed by loss of consciousness and jerking of all extremities, is characteristic of a tonic-clonic seizure. An atonic seizure presents as a sudden loss of muscle tone followed by postictal confusion. A myoclonic seizure presents with a brief jerking or stiffening of extremities that may occur singly or in groups. Absence seizures present with automatisms, and the client is unaware of his or her environment.

A nurse is examining a client reporting right upper quadrant (RUQ) abdominal pain. What technique should the nurse use to assess this clients abdomen?

a. Auscultate after palpating. b. Avoid any palpation. c. Palpate the RUQ first. d. Palpate the RUQ last. ANS: D If pain is present in a certain area of the abdomen, that area should be palpated last to keep the client from tensing up, which could possibly affect the rest of the examination. Auscultation of the abdomen occurs prior to palpation.

A client presents to the emergency department reporting severe abdominal pain. On assessment, the nurse finds a bulging, pulsating mass in the abdomen. What action by the nurse is the priority?

a. Auscultate for bowel sounds. b. Notify the provider immediately. c. Order an abdominal flat-plate x-ray. d. Palpate the mass and measure its size. ANS: B This observation could indicate an abdominal aortic aneurysm, which could be life threatening and should never be palpated. The nurse notifies the provider at once. An x-ray may be indicated. Auscultation is part of assessment, but the nurses priority action is to notify the provider.

A client is receiving bolus feedings through a Dobhoff tube. What action by the nurse is most important?

a. Auscultate lung sounds after each feeding. b. Check tube placement before each feeding. c. Check tube placement every 8 hours. d. Weigh the client daily on the same scale. ANS: B For bolus feedings, the nurse checks placement of the tube per institutional policy prior to each feeding, which is more often than every 8 hours during the day. Auscultating lung sounds is also important, but this will indicate a complication that has already occurred. Weighing the client is important to determine if nutritional goals are being met.

A client with acquired immune deficiency syndrome has been hospitalized with suspected cryptosporidiosis. What physical assessment would be most consistent with this condition?

a. Auscultating the lungs *b. Assessing mucous membranes* c. Listening to bowel sounds d. Performing a neurologic examination ANS: B Cryptosporidiosis can cause extreme loss of fluids and electrolytes, up to 20 L/day. The nurse should assess signs of hydration/dehydration as the priority, including checking the clients mucous membranes for dryness. The nurse will perform the other assessments as part of a comprehensive assessment.

A nurse assesses a clients respiratory status. Which information is of highest priority for the nurse to obtain?

a. Average daily fluid intake b. Neck circumference c. Height and weight d. Occupation and hobbies ANS: D Many respiratory problems occur as a result of chronic exposure to inhalation irritants used in a clients occupation and hobbies. Although it will be important for the nurse to assess the clients fluid intake, height, and weight, these will not be as important as determining his occupation and hobbies. Determining the clients neck circumference will not be an important part of a respiratory assessment.

A client is going on a cruise but has had motion sickness in the past. What suggestion does the nurse make to this client?

a. Avoid alcohol on the cruise ship. b. Change positions slowly on the ship. c. Change your travel plans. *d. Try scopolamine (Transderm Scop*). ANS: D Scopolamine can successfully treat the vertigo and dizziness associated with motion sickness. Avoiding alcohol and changing positions slowly are not effective. Telling the client to change travel plans is not a caring suggestion.

A nurse teaches a client who is being discharged home with a peripherally inserted central catheter (PICC). Which statement should the nurse include in this clients teaching?

a. Avoid carrying your grandchild with the arm that has the central catheter. b.Be sure to place the arm with the central catheter in a sling during the day. c. Flush the peripherally inserted central catheter line with normal saline daily. d.You can use the arm with the central catheter for most activities of daily living. ANS: A A properly placed PICC (in the antecubital fossa or the basilic vein) allows the client considerable freedom of movement. Clients can participate in most activities of daily living; however, heavy lifting can dislodge the catheter or occlude the lumen. Although it is important to keep the insertion site and tubing dry, the client can shower. The device is flushed with heparin.

A nurse is working with a community group promoting healthy aging. What recommendation is best to help prevent osteoarthritis (OA)?

a. Avoid contact sports. b. Get plenty of calcium. *c. Lose weight if needed.* d. Engage in weight-bearing exercise. ANS: C Obesity can lead to OA, and if the client is overweight, losing weight can help prevent OA or reduce symptoms once it occurs. Arthritis can be caused by contact sports, but this is less common than obesity. Calcium and weight-bearing exercise are both important for osteoporosis.

A nurse teaches a client who has a history of heart failure. Which statement should the nurse include in this clients discharge teaching?

a. Avoid drinking more than 3 quarts of liquids each day. b. Eat six small meals daily instead of three larger meals. c. When you feel short of breath, take an additional diuretic. d. Weigh yourself daily while wearing the same amount of clothing. ANS: D Clients with heart failure are instructed to weigh themselves daily to detect worsening heart failure early, and thus avoid complications. Other signs of worsening heart failure include increasing dyspnea, exercise intolerance, cold symptoms, and nocturia. Fluid overload increases symptoms of heart failure. The client should be taught to eat a heart-healthy diet, balance intake and output to prevent dehydration and overload, and take medications as prescribed. The most important discharge teaching is daily weights as this provides the best data related to fluid retention.

A client has undergone a percutaneous stereotactic rhizotomy. What instruction by the nurse is most important on discharge from the ambulatory surgical center?

a. Avoid having teeth pulled for 1 year. b. Brush your teeth with a soft toothbrush. *c. Do not use harsh chemicals on your face*. d. Inform your dentist of this procedure. ANS: C The affected side is left without sensation after this procedure. The client should avoid putting harsh chemicals on the face because he or she will not feel burning or stinging on that side. This will help avoid injury. The other instructions are not necessary.

A nurse teaches a client who is prescribed digoxin (Lanoxin) therapy. Which statement should the nurse include in this clients teaching?

a. Avoid taking aspirin or aspirin-containing products. b. Increase your intake of foods that are high in potassium. c. Hold this medication if your pulse rate is below 80 beats/min. d. Do not take this medication within 1 hour of taking an antacid. ANS: D Gastrointestinal absorption of digoxin is erratic. Many medications, especially antacids, interfere with its absorption. Clients are taught to hold their digoxin for bradycardia; a heart rate of 80 beats/min is too high for this cutoff. Potassium and aspirin have no impact on digoxin absorption, nor do these statements decrease complications of digoxin therapy.

A nurse is teaching a client with heart failure who has been prescribed enalapril (Vasotec). Which statement should the nurse include in this clients teaching?

a. Avoid using salt substitutes. b. Take your medication with food. c. Avoid using aspirin-containing products. d. Check your pulse daily. ANS: A Angiotensin-converting enzyme (ACE) inhibitors such as enalapril inhibit the excretion of potassium. Hyperkalemia can be a life-threatening side effect, and clients should be taught to limit potassium intake. Salt substitutes are composed of potassium chloride. ACE inhibitors do not need to be taken with food and have no impact on the clients pulse rate. Aspirin is often prescribed in conjunction with ACE inhibitors and is not contraindicated

The nurse works with clients who have hearing problems. Which action by a client best indicates goals for an important diagnosis have been met?

a. Babysitting the grandchildren several times a week b. Having an adaptive hearing device for the television *c. Being active in community events and volunteer work* d. Responding agreeably to suggestions for adaptive devices ANS: C Clients with hearing problems can become frustrated and withdrawn. The client who is actively engaged in the community shows the best evidence of psychosocial adjustment to hearing loss. Babysitting the grandchildren is a positive sign but does not indicate involvement outside the home. Having an adaptive device is not the same as using it, and watching TV without evidence of other activities can also indicate social isolation. Responding agreeably does not indicate the client will actually follow through.

A nurse cares for a client who presents with an acute exacerbation of multiple sclerosis (MS). Which prescribed medication should the nurse prepare to administer?

a. Baclofen (Lioresal) b. Interferon beta-1b (Betaseron) c. Dantrolene sodium (Dantrium) *d. Methylprednisolone (Medrol)* ANS: D Methylprednisolone is the drug of choice for acute exacerbations of the disease. The other drugs are not used to treat acute exacerbations of MS. Interferon beta-1b is used to treat and control MS, decrease specific symptoms, and slow the progression of the disease. Baclofen and dantrolene sodium are prescribed to lessen muscle spasticity associated with MS.

A client has newly diagnosed systemic lupus erythematosus (SLE). What instruction by the nurse is most important?

a. Be sure you get enough sleep at night. b. Eat plenty of high-protein, high-iron foods. *c. Notify your provider at once if you get a fever*. d. Weigh yourself every day on the same scale. ANS: C Fever is the classic sign of a lupus flare and should be reported immediately. Rest and nutrition are important but do not take priority over teaching the client what to do if he or she develops an elevated temperature. Daily weights may or may not be important depending on renal involvement.

A nursing student is struggling to understand the process of graft-versus-host disease. What explanation by the nurse instructor is best?

a. Because of immunosuppression, the donor cells take over. b. Its like a transfusion reaction because no perfect matches exist. c. The clients cells are fighting donor cells for dominance. d. The donors cells are actually attacking the clients cells. ANS: D Graft versus host disease is an autoimmune-type process in which the donor cells recognize the clients cells as foreign and begin attacking them. The other answers are not accurate.

A nurse performs an admission assessment on a 75-year-old client with multiple chronic diseases. The clients blood pressure is 135/75 mm Hg and oxygen saturation is 94% on 2 liters per nasal cannula. The nurse assesses the clients rhythm on the cardiac monitor and observes the reading shown below: Which action should the nurse take first?

a. Begin external temporary pacing. b. Assess peripheral pulse strength. c. Ask the client what medications he or she takes. d. Administer 1 mg of atropine. ANS: C This client is stable and therefore does not require any intervention except to determine the cause of the bradycardia. Bradycardia is often caused by medications. Clients who have multiple chronic diseases are often on multiple medications that can interact with each other. The nurse should assess the clients current medications first.

A nurse is caring for a client who has just had a central venous access line inserted. Which action should the nurse take next?

a. Begin the prescribed infusion via the new access. b.Ensure an x-ray is completed to confirm placement. c. Check medication calculations with a second RN. d.Make sure the solution is appropriate for a central line. ANS: B A central venous access device, once placed, needs an x-ray confirmation of proper placement before it is used. The bedside nurse would be responsible for beginning the infusion once placement has been verified. Any IV solution can be given through a central line.

A client has been admitted after sustaining a humerus fracture that occurred when picking up the family cat. What test result would the nurse correlate to this condition?

a. Bence-Jones protein in urine b. Epstein-Barr virus: positive c. Hemoglobin: 18 mg/dL d. Red blood cell count: 8.2/mm3 ANS: A This client has possible multiple myeloma. A positive Bence-Jones protein finding would correlate with this condition. The Epstein-Barr virus is a herpesvirus that causes infectious mononucleosis and some cancers. A hemoglobin of 18 mg/dL is slightly high for a male and somewhat high for a female; this can be caused by several conditions, and further information would be needed to correlate this value with a specific medical condition. A red blood cell count of 8.2/mm3 is also high, but again, more information would be needed to correlate this finding with a specific medical condition.

A client has been taught about retinitis pigmentosa (RP). What statement by the client indicates a need for further teaching?

a. Beta carotene, lutein, and zeaxanthin are good supplements. b. I might qualify for a retinal transplant one day soon. *c. Since Im going blind, sunglasses are not needed anymore*. d. Vitamin A has been shown to slow progression of RP. ANS: C Sunglasses are needed to prevent the development of cataracts in addition to the RP. The other statements are accurate.

1. A client is admitted with Guillain-Barr syndrome (GBS). What assessment takes priority?

a. Bladder control b. Cognitive perception *c. Respiratory system* d. Sensory functions ANS: C Clients with GBS have muscle weakness, possibly to the point of paralysis. If respiratory muscles are paralyzed, the client may need mechanical ventilation, so the respiratory system is the priority. The nurse will complete urinary, cognitive, and sensory assessments as part of a thorough evaluation.

A nurse assesses a client who is prescribed a medication that inhibits angiotensin I from converting into angiotensin II (angiotensin-converting enzyme [ACE] inhibitor). For which expected therapeutic effect should the nurse assess?

a. Blood pressure decrease from 180/72 mm Hg to 144/50 mm Hg b. Daily weight increase from 55 kg to 57 kg c. Heart rate decrease from 100 beats/min to 82 beats/min d. Respiratory rate increase from 12 breaths/min to 15 breaths/min ANS: A ACE inhibitors will disrupt the reninangiotensin II pathway and prevent the kidneys from reabsorbing water and sodium. The kidneys will excrete more water and sodium, decreasing the clients blood pressure.

. A nurse assesses a client after administering a prescribed beta blocker. Which assessment should the nurse expect to find?

a. Blood pressure increased from 98/42 mm Hg to 132/60 mm Hg b. Respiratory rate decreased from 25 breaths/min to 14 breaths/min c. Oxygen saturation increased from 88% to 96% d. Pulse decreased from 100 beats/min to 80 beats/min ANS: D Beta blockers block the stimulation of beta1-adrenergic receptors. They block the sympathetic (fight-or-flight) response and decrease the heart rate (HR). The beta blocker will decrease HR and blood pressure, increasing ventricular filling time. It usually does not have effects on beta2-adrenergic receptor sites. Cardiac output will drop because of decreased HR.

A nurse suspects a client has serum sickness. What laboratory result would the nurse correlate with this condition?

a. Blood urea nitrogen: 12 mg/dL *b. Creatinine: 3.2 mg/dL* c. Hemoglobin: 8.2 mg/dL d. White blood cell count: 12,000/mm3 ANS: B The creatinine is high, possibly indicating the client has serum sickness nephritis. Blood urea nitrogen and white blood cell count are both normal. Hemoglobin is not related.

A client with multiple myeloma demonstrates worsening bone density on diagnostic scans. About what drug does the nurse plan to teach this client?

a. Bortezomib (Velcade) b. Dexamethasone (Decadron) c. Thalidomide (Thalomid) d. Zoledronic acid (Zometa) ANS: D All the options are drugs used to treat multiple myeloma, but the drug used specifically for bone manifestations is zoledronic acid (Zometa), which is a bisphosphonate. This drug class inhibits bone resorption and is used to treat osteoporosis as well.

A client admitted for sickle cell crisis is distraught after learning her child also has the disease. What response by the nurse is best?

a. Both you and the father are equally responsible for passing it on. b. I can see you are upset. I can stay here with you a while if you like. c. Its not your fault; there is no way to know who will have this disease. d. There are many good treatments for sickle cell disease these days. ANS: B The best response is for the nurse to offer self, a therapeutic communication technique that uses presence. Attempting to assign blame to both parents will not help the client feel better. There is genetic testing available, so it is inaccurate to state there is no way to know who will have the disease. Stating that good treatments exist belittles the clients feelings.

.A client admitted for pneumonia has been tachypneic for several days. When the nurse starts an IV to give fluids, the client questions this action, saying I have been drinking tons of water. How am I dehydrated? What response by the nurse is best?

a. Breathing so quickly can be dehydrating. b.Everyone with pneumonia is dehydrated. c. This is really just to administer your antibiotics. d.Why do you think you are so dehydrated? ANS: A Tachypnea and mouth breathing, both seen in pneumonia, increase insensible water loss and can lead to a degree of dehydration. The other options do not give the client useful information.

A nurse obtains a health history on a client prior to administering prescribed sumatriptan succinate (Imitrex) for migraine headaches. Which condition should alert the nurse to hold the medication and contact the health care provider?

a. Bronchial asthma *b. Prinzmetals angina* c. Diabetes mellitus d. Chronic kidney disease ANS: B Sumatriptan succinate effectively reduces pain and other associated symptoms of migraine headache by binding to serotonin receptors and triggering cranial vasoconstriction. Vasoconstrictive effects are not confined to the cranium and can cause coronary vasospasm in clients with Prinzmetals angina. The other conditions would not affect the clients treatment.

A nurse administers medications to a client who has asthma. Which medication classification is paired correctly with its physiologic response to the medication?

a. Bronchodilator Stabilizes the membranes of mast cells and prevents the release of inflammatory mediators b. Cholinergic antagonist Causes bronchodilation by inhibiting the parasympathetic nervous system c. Corticosteroid Relaxes bronchiolar smooth muscles by binding to and activating pulmonary beta2 receptors d. Cromone Disrupts the production of pathways of inflammatory mediators ANS: B Cholinergic antagonist drugs cause bronchodilation by inhibiting the parasympathetic nervous system. This allows the sympathetic nervous system to dominate and release norepinephrine that actives beta2 receptors. Bronchodilators relax bronchiolar smooth muscles by binding to and activating pulmonary beta2 receptors. Corticosteroids disrupt the production of pathways of inflammatory mediators. Cromones stabilize the membranes of mast cells and prevent the release of inflammatory mediators.

A nurse is assessing a client who has a tracheostomy. The nurse notes that the tracheostomy tube is pulsing with the heartbeat as the clients pulse is being taken. No other abnormal findings are noted. What action by the nurse is most appropriate?

a. Call the operating room to inform them of a pending emergency case. b. No action is needed at this time; this is a normal finding in some clients. c. Remove the tracheostomy tube; ventilate the client with a bag-valve-mask. d. Stay with the client and have someone else call the provider immediately. ANS: D This client may have a tracheainnominate artery fistula, which can be a life-threatening emergency if the artery is breached and the client begins to hemorrhage. Since no bleeding is yet present, the nurse stays with the client and asks someone else to notify the provider. If the client begins hemorrhaging, the nurse removes the tracheostomy and applies pressure at the bleeding site. The client will need to be prepared for surgery.

A nurse cares for a client who had a bronchoscopy 2 hours ago. The client asks for a drink of water. Which action should the nurse take next?

a. Call the physician and request a prescription for food and water. b. Provide the client with ice chips instead of a drink of water. c. Assess the clients gag reflex before giving any food or water. d. Let the client have a small sip to see whether he or she can swallow. ANS: C The topical anesthetic used during the procedure will have affected the clients gag reflex. Before allowing the client anything to eat or drink, the nurse must check for the return of this reflex.

A client has a platelet count of 9000/mm3. The nurse finds the client confused and mumbling. What action takes priority?

a. Calling the Rapid Response Team b. Delegating taking a set of vital signs c. Instituting bleeding precautions d. Placing the client on bedrest ANS: A With a platelet count this low, the client is at high risk of spontaneous bleeding. The most disastrous complication would be intracranial bleeding. The nurse needs to call the Rapid Response Team as this client has manifestations of a sudden neurologic change. The nurse should not delegate the vital signs as the client is no longer stable. Bleeding precautions will not address the immediate situation. Placing the client on bedrest or putting the client back into bed is important, but the critical action is to call for immediate medical attention.

A nurse develops a dietary plan for a client with diabetes mellitus and new-onset microalbuminuria. Which component of the clients diet should the nurse decrease?

a. Carbohydrates *b.Proteins* c. Fats d.Total calories ANS: B Restriction of dietary protein to 0.8 g/kg of body weight per day is recommended for clients with microalbuminuria to delay progression to renal failure. The clients diet does not need to be decreased in carbohydrates, fats, or total calories.

A nurse assesses a client who is experiencing an acid-base imbalance. The clients arterial blood gas values are pH 7.34, PaO2 88 mm Hg, PaCO2 38 mm Hg, and HCO3 19 mEq/L. Which assessment should the nurse perform first?

a. Cardiac rate and rhythm b. Skin and mucous membranes c. Musculoskeletal strength d. Level of orientation ANS: A Early cardiovascular changes for a client experiencing moderate acidosis include increased heart rate and cardiac output. As the acidosis worsens, the heart rate decreases and electrocardiographic changes will be present. Central nervous system and neuromuscular system changes do not occur with mild acidosis and should be monitored if the acidosis worsens. Skin and mucous membrane assessment is not a priority now, but will change as acidosis worsens.

A nurse cares for a client who is experiencing deteriorating neurologic functions. The client states, I am worried I will not be able to care for my young children. How should the nurse respond?

a. Caring for your children is a priority. You may not want to ask for help, but you have to. b. Our community has resources that may help you with some household tasks so you have energy to care for your children. c. You seem distressed. Would you like to talk to a psychologist about adjusting to your changing status? *d. Give me more information about what worries you, so we can see if we can do something to make adjustments*. ANS: D Investigate specific concerns about situational or role changes before providing additional information. The nurse should not tell the client what is or is not a priority for him or her. Although community resources may be available, they may not be appropriate for the client. Consulting a psychologist would not be appropriate without obtaining further information from the client related to current concerns.

.A nurse teaches a client with type 2 diabetes mellitus who is prescribed glipizide (Glucotrol). Which statement should the nurse include in this clients teaching?

a. Change positions slowly when you get out of bed. *b.Avoid taking nonsteroidal anti-inflammatory drugs (NSAIDs)*. c. If you miss a dose of this drug, you can double the next dose. d.Discontinue the medication if you develop a urinary infection. ANS: B NSAIDs potentiate the hypoglycemic effects of sulfonylurea agents. Glipizide is a sulfonylurea. The other statements are not applicable to glipizide

An older client has had an instance of drug toxicity and asks why this happens, since the client has been on this medication for years at the same dose. What response by the nurse is best?

a. Changes in your liver cause drugs to be metabolized differently. b. Perhaps you dont need as high a dose of the drug as before. c. Stomach muscles atrophy with age and you digest more slowly. d. Your body probably cant tolerate as much medication anymore. ANS: A Decreased liver enzyme activity depresses drug metabolism, which leads to accumulation of drugspossibly to toxic levels. The other options do not accurately explain this age-related change.

A nurse plans care for a client who has a hypoactive response to a test of deep tendon reflexes. Which intervention should the nurse include in this clients plan of care?

a. Check bath water temperature with a thermometer. *b. Provide the client with assistance when ambulating*. c. Place elastic support hose on the clients legs. d. Assess the clients feet for wounds each shift. ANS: B Hypoactive deep tendon reflexes and loss of vibration sense can impair balance and coordination, predisposing the client to falls. The nurse should plan to provide the client with ambulation assistance to prevent injury. The other interventions do not address the clients problem

.A nurse responds to an IV pump alarm related to increased pressure. Which action should the nurse take first?

a. Check for kinking of the catheter. b.Flush the catheter with a thrombolytic enzyme. c. Get a new infusion pump. d.Remove the IV catheter. ANS: A Fluid flow through the infusion system requires that pressure on the external side be greater than pressure at the catheter tip. Fluid flow can be slowed for many reasons. A common reason, and one that is easy to correct, is a kinked catheter. If this is not the cause of the pressure alarm, the nurse may have to ascertain whether a clot has formed inside the catheter lumen, or if the pump is no longer functional. Removal of the IV catheter and placement of a new IV catheter should be completed when no other option has resolved the problem.

An older female client has been prescribed esomeprazole (Nexium) for treatment of chronic gastric ulcers. What teaching is particularly important for this client?

a. Check with the pharmacist before taking other medications. b. Increase intake of calcium and vitamin D. c. Report any worsening of symptoms to the provider. d. Take the medication as prescribed by the provider. ANS: B All of this advice is appropriate for any client taking this medication. However, long-term use is associated with osteoporosis and osteoporosis-related fractures. This client is already at higher risk for this problem and should be instructed to increase calcium and vitamin D intake. The other options are appropriate for any client taking any medication and are not specific to the use of esomeprazole.

The nurse is caring for a client using a continuous passive motion (CPM) machine and has delegated some tasks to the unlicensed assistive personnel (UAP). What action by the UAP warrants intervention by the nurse?

a. Checking to see if the machine is working b. Keeping controls in a secure place on the bed c. Placing padding in the machine per request *d. Storing the CPM machine under the bed after removal* ANS: D For infection control (and to avoid tripping on it), the CPM machine is never placed on the floor. The other actions are appropriate

An older adult is brought to the emergency department by a family member, who reports a moderate change in mental status and mild cough. The client is afebrile. The health care provider orders a chest x-ray. The family member questions why this is needed since the manifestations seem so vague. What response by the nurse is best?

a. Chest x-rays are always ordered when we suspect pneumonia. b.Older people often have vague symptoms, so an x-ray is essential. c. The x-ray can be done and read before laboratory work is reported. d.We are testing for any possible source of infection in the client. ANS: B It is essential to obtain an early chest x-ray in older adults suspected of having pneumonia because symptoms are often vague. Waiting until definitive manifestations are present to obtain the x-ray leads to a costly delay in treatment. Stating that chest x-rays are always ordered does not give the family definitive information. The xray can be done while laboratory values are still pending, but this also does not provide specific information about the importance of a chest x-ray in this client. The client has manifestations of pneumonia, so the staff is not testing for any possible source of infection but rather is testing for a suspected disorder.

A nurse cares for a client who is recovering from a myocardial infarction. The client states, I will need to stop eating so much chili to keep that indigestion pain from returning. How should the nurse respond?

a. Chili is high in fat and calories; it would be a good idea to stop eating it. b. The provider has prescribed an antacid for you to take every morning. c. What do you understand about what happened to you? d. When did you start experiencing this indigestion? ANS: C Clients who experience myocardial infarction often respond with denial, which is a defense mechanism. The nurse should ask the client what he or she thinks happened, or what the illness means to him or her. The other responses do not address the clients misconception about recent pain and the cause of that pain.

A nurse working in a geriatric clinic sees clients with cold symptoms and rhinitis. Which drug would be appropriate to teach these clients to take for their symptoms?

a. Chlorpheniramine (ChlorTrimeton) b.Diphenhydramine (Benadryl) c. Fexofenadine (Allegra) d.Hydroxyzine (Vistaril) ANS: C First-generation antihistamines are not appropriate for use in the older population. These drugs include chlorpheniramine, diphenhydramine, and hydroxyzine. Fexofenadine is a second-generation antihistamine.

The nurse is reviewing the lipid panel of a male client who has atherosclerosis. Which finding is most concerning?

a. Cholesterol: 126 mg/dL b. High-density lipoprotein cholesterol (HDL-C): 48 mg/dL c. Low-density lipoprotein cholesterol (LDL-C): 122 mg/dL d. Triglycerides: 198 mg/dL ANS: D Triglycerides in men should be below 160 mg/dL. The other values are appropriate for adult males.

A client with HIV wasting syndrome has inadequate nutrition. What assessment finding by the nurse best indicates that goals have been met for this client problem?

a. Chooses high-protein food b. Has decreased oral discomfort c. Eats 90% of meals and snacks *d. Has a weight gain of 2 pounds/1 month* ANS: D The weight gain is the best indicator that goals for this client problem have been met because it demonstrates that the client not only is eating well but also is able to absorb the nutrients

A nurse supervises an unlicensed assistive personnel (UAP) applying electrocardiographic monitoring. Which statement should the nurse provide to the UAP related to this procedure?

a. Clean the skin and clip hairs if needed. b. Add gel to the electrodes prior to applying them. c. Place the electrodes on the posterior chest. d. Turn off oxygen prior to monitoring the client. ANS: A To ensure the best signal transmission, the skin should be clean and hairs clipped. Electrodes should be placed on the anterior chest, and no additional gel is needed. Oxygen has no impact on electrocardiographic monitoring.

A nurse assesses a client after an open lung biopsy. Which assessment finding is matched with the correct intervention?

a. Client states he is dizzy. Nurse applies oxygen and pulse oximetry. b. Clients heart rate is 55 beats/min. Nurse withholds pain medication. c. Client has reduced breath sounds. Nurse calls physician immediately. d. Clients respiratory rate is 18 breaths/min. Nurse decreases oxygen flow rate. ANS: C A potentially serious complication after biopsy is pneumothorax, which is indicated by decreased or absent breath sounds. The physician needs to be notified immediately. Dizziness after the procedure is not an expected finding. If the clients heart rate is 55 beats/min, no reason is known to withhold pain medication. A respiratory rate of 18 breaths/min is a normal finding and would not warrant changing the oxygen flow rate.

For which client would the nurse suggest the provider not prescribe misoprostol (Cytotec)?

a. Client taking antacids b. Client taking antibiotics c. Client who is pregnant d. Client over 65 years of age ANS: C Misoprostol can cause abortion, so pregnant women should not take this drug. The other clients have no contraindications to taking misoprostol

The nurse in the rheumatology clinic is assessing clients with rheumatoid arthritis (RA). Which client should the nurse see first?

a. Client taking celecoxib (Celebrex) and ranitidine (Zantac) b. Client taking etanercept (Enbrel) with a red injection site c. Client with a blood glucose of 190 mg/dL who is taking steroids *d. Client with a fever and cough who is taking tofacitinib (Xeljanz)* ANS: D Tofacitinib carries a Food and Drug Administration black box warning about opportunistic infections, tuberculosis, and cancer. Fever and cough may indicate tuberculosis. Ranitidine is often taken with celecoxib, which can cause gastrointestinal distress. Redness and itchy rashes are frequently seen with etanercept injections. Steroids are known to raise blood glucose levels

A nurse is assessing clients on a medical-surgical unit. Which adult client should the nurse identify as being at greatest risk for insensible water loss?

a. Client taking furosemide (Lasix) b. Anxious client who has tachypnea c. Client who is on fluid restrictions d. Client who is constipated with abdominal pain ANS: B Insensible water loss is water loss through the skin, lungs, and stool. Clients at risk for insensible water loss include those being mechanically ventilated, those with rapid respirations, and those undergoing continuous GI suctioning. Clients who have thyroid crisis, trauma, burns, states of extreme stress, and fever are also at increased risk. The client taking furosemide will have increased fluid loss, but not insensible water loss. The other two clients on a fluid restriction and with constipation are not at risk for fluid loss.

. A nurse is caring for four clients with leukemia. After hand-off report, which client should the nurse see first?

a. Client who had two bloody diarrhea stools this morning b. Client who has been premedicated for nausea prior to chemotherapy c. Client with a respiratory rate change from 18 to 22 breaths/min d. Client with an unchanged lesion to the lower right lateral malleolus ANS: A The client who had two bloody diarrhea stools that morning may be hemorrhaging in the gastrointestinal (GI) tract and should be assessed first. The client with the change in respiratory rate may have an infection or worsening anemia and should be seen next. The other two clients are not a priority at this time.

A nurse is caring for four clients. Which one should the nurse see first?

a. Client who needs a beta blocker, and has a blood pressure of 92/58 mm Hg b. Client who had a first dose of captopril (Capoten) and needs to use the bathroom c. Hypertensive client with a blood pressure of 188/92 mm Hg d. Client who needs pain medication prior to a dressing change of a surgical wound ANS: B Angiotensin-converting enzyme inhibitors such as captopril can cause hypotension, especially after the first dose. The nurse should see this client first to prevent falling if the client decides to get up without assistance. The two blood pressure readings are abnormal but not critical. The nurse should check on the client with higher blood pressure next to assess for problems related to the reading. The nurse can administer the beta blocker as standards state to hold it if the systolic blood pressure is below 90 mm Hg. The client who needs pain medication prior to the dressing change is not a priority over client safety and assisting the other client to the bathroom.

A nurse works in the rheumatology clinic and sees clients with rheumatoid arthritis (RA). Which client should the nurse see first?

a. Client who reports jaw pain when eating *b. Client with a red, hot, swollen right wrist* c. Client who has a puffy-looking area behind the knee d. Client with a worse joint deformity since the last visit ANS: B All of the options are possible manifestations of RA. However, the presence of one joint that is much redder, hotter, or more swollen that the other joints may indicate infection. The nurse needs to see this client first.

A nurse is caring for four clients receiving enteral tube feedings. Which client should the nurse see first?

a. Client with a blood glucose level of 138 mg/dL b. Client with foul-smelling diarrhea c. Client with a potassium level of 2.6 mEq/L d. Client with a sodium level of 138 mEq/L ANS: C The potassium is critically low, perhaps due to hyperglycemia-induced hyperosmolarity. The nurse should see this client first. The blood glucose reading is high, but not extreme. The sodium is normal. The client with the diarrhea should be seen last to avoid cross-contamination.

. A nurse in a hematology clinic is working with four clients who have polycythemia vera. Which client should the nurse see first?

a. Client with a blood pressure of 180/98 mm Hg b. Client who reports shortness of breath c. Client who reports calf tenderness and swelling d. Client with a swollen and painful left great toe ANS: B Clients with polycythemia vera often have clotting abnormalities due to the hyperviscous blood with sluggish flow. The client reporting shortness of breath may have a pulmonary embolism and should be seen first. The client with a swollen calf may have a deep vein thrombosis and should be seen next. High blood pressure and gout symptoms are common findings with this disorder.

A nurse is teaching a community group about noise-induced hearing loss. Which client who does not use ear protection should the nurse refer to an audiologist as the priority?

a. Client with an hour car commute on the freeway each day b. Client who rides a motorcycle to work 20 minutes each way c. Client who sat in the back row at a rock concert recently *d. Client who is a tree-trimmer and uses a chainsaw 6 to 7 hours a day* ANS: D A chainsaw becomes dangerous to hearing after 2 hours of exposure without hearing protection. This client needs to be referred as the priority. Normal car traffic is safe for more than 8 hours. Motorcycle noise is safe for about 8 hours. The safe exposure time for a front-row rock concert seat is 3 minutes, but this client was in the back, and so had less exposure. In addition, a one-time exposure is less damaging than chronic exposure

A nurse is caring for four clients. After reviewing todays laboratory results, which client should the nurse see first?

a. Client with an international normalized ratio of 2.8 b. Client with a platelet count of 128,000/mm3 c. Client with a prothrombin time (PT) of 28 seconds d. Client with a red blood cell count of 5.1 million/L ANS: C A normal PT is 11 to 12.5 seconds. This client is at high risk of bleeding. The other values are within normal limits.

A nurse and a registered dietitian are assessing clients for partial parenteral nutrition (PPN). For which client would the nurse suggest another route of providing nutrition?

a. Client with congestive heart failure b. Older client with dementia c. Client who has multiorgan failure d. Client who is post gastric resection ANS: A Clients receiving PPN typically get large amounts of fluid volume, making the client with heart failure a poor candidate. The other candidates are appropriate for this type of nutritional support.

A nurse is seeing clients in the ophthalmology clinic. Which client should the nurse see first?

a. Client with intraocular pressure reading of 24 mm Hg *b. Client who has had cataract surgery and has worsening vision* c. Client whose red reflex is absent on ophthalmologic examination d. Client with a tearing, reddened eye with exudate ANS: B After cataract surgery, worsening vision indicates an infection or other complication. The nurse should see this client first. The intraocular pressure is slightly elevated. An absent red reflex may indicate cataracts. The client who has the tearing eye may have an infection.

A nurse is assessing clients on a medical-surgical unit. Which client is at risk for hypokalemia?

a. Client with pancreatitis who has continuous nasogastric suctioning b. Client who is prescribed an angiotensin-converting enzyme (ACE) inhibitor c. Client in a motor vehicle crash who is receiving 6 units of packed red blood cells d. Client with uncontrolled diabetes and a serum pH level of 7.33 ANS: A A client with continuous nasogastric suctioning would be at risk for actual potassium loss leading to hypokalemia. The other clients are at risk for potassium excess or hyperkalemia.

A nurse assesses a client who is scheduled for a cardiac catheterization. Which assessment should the nurse complete prior to this procedure?

a. Clients level of anxiety b. Ability to turn self in bed c. Cardiac rhythm and heart rate d. Allergies to iodine-based agents ANS: D Before the procedure, the nurse should ascertain whether the client has an allergy to iodine-containing preparations, such as seafood or local anesthetics. The contrast medium used during the procedure is iodine based. This allergy can cause a life-threatening reaction, so it is a high priority. Second, it is important for the nurse to assess anxiety, mobility, and baseline cardiac status.

The health care provider tells the nurse that a client is to be started on a platelet inhibitor. About what drug does the nurse plan to teach the client?

a. Clopidogrel (Plavix) b. Enoxaparin (Lovenox) c. Reteplase (Retavase) d. Warfarin (Coumadin) ANS: A Clopidogrel is a platelet inhibitor. Enoxaparin is an indirect thrombin inhibitor. Reteplase is a fibrinolytic agent. Warfarin is a vitamin K antagonist.

A client seen in the emergency department reports fever, fatigue, and dry cough but no other upper respiratory symptoms. A chest x-ray reveals mediastinal widening. What action by the nurse is best?

a. Collect a sputum sample for culture by deep suctioning. b.Inform the client that antibiotics will be needed for 60 days. c. Place the client on Airborne Precautions immediately. d.Tell the client that directly observed therapy is needed. ANS: B This client has manifestations of early inhalation anthrax. For treatment, after IV antibiotics are finished, oral antibiotics are continued for at least 60 days. Sputum cultures are not needed. Anthrax is not transmissible from person to person, so Standard Precautions are adequate. Directly observed therapy is often used for tuberculosis.

A nurse assesses a client who has a nasal fracture. The client reports constant nasal drainage, a headache, and difficulty with vision. Which action should the nurse take next?

a. Collect the nasal drainage on a piece of filter paper. b. Encourage the client to blow his or her nose. c. Perform a test focused on a neurologic examination. d. Palpate the nose, face, and neck. ANS: A The client with nasal drainage after facial trauma could have a skull fracture that has resulted in leakage of cerebrospinal fluid (CSF). CSF can be differentiated from regular drainage by the fact that it forms a halo when dripped on filter paper. The other actions would be appropriate but are not as high a priority as assessing for CSF. A CSF leak would increase the clients risk for infection.

A client who has been taking antibiotics reports severe, watery diarrhea. About which test does the nurse teach the client?

a. Colonoscopy b. Enzyme-linked immunosorbent assay (ELISA) toxin A+B c. Ova and parasites d. Stool culture ANS: B Clients taking antibiotics are at risk for Clostridium difficile infection. The most common test for this disorder is a stool sample for ELISA toxin A+B. Colonoscopy, ova and parasites, and stool culture are not warranted at this time.

A client is being discharged on long-term therapy for tuberculosis (TB). What referral by the nurse is most appropriate?

a. Community social worker for Meals on Wheels b.Occupational therapy for job retraining c. Physical therapy for homebound therapy services d.Visiting Nurses for directly observed therapy ANS: D Directly observed therapy is often utilized for managing clients with TB in the community. Meals on Wheels, job retraining, and home therapy may or may not be appropriate.

A nurse assesses a client who is recovering from a myocardial infarction. The clients pulmonary artery pressure reading is 25/12 mm Hg. Which action should the nurse take first?

a. Compare the results with previous pulmonary artery pressure readings. b. Increase the intravenous fluid rate because these readings are low. c. Immediately notify the health care provider of the elevated pressures. d. Document the finding in the clients chart as the only action. ANS: A Normal pulmonary artery pressures range from 15 to 26 mm Hg for systolic and from 5 to 15 mm Hg for diastolic. Although this clients readings are within normal limits, the nurse needs to assess any trends that may indicate a need for medical treatment to prevent complications. There is no need to increase intravenous fluids or notify the provider.

A nurse is teaching a client with cerebellar function impairment. Which statement should the nurse include in this clients discharge teaching?

a. Connect a light to flash when your door bell rings. b. Label your faucet knobs with hot and cold signs. *c. Ask a friend to drive you to your follow-up appointments*. d. Use a natural gas detector with an audible alarm. ANS: C Cerebellar function enables the client to predict distance or gauge the speed with which one is approaching an object, control voluntary movement, maintain equilibrium, and shift from one skilled movement to another in an orderly sequence. A client who has cerebellar function impairment should not be driving. The client would not have difficulty hearing, distinguishing between hot and cold, or smelling.

A nurse is caring for a client with a nonhealing arterial lower leg ulcer. What action by the nurse is best?

a. Consult with the Wound Ostomy Care Nurse. b. Give pain medication prior to dressing changes. c. Maintain sterile technique for dressing changes. d. Prepare the client for eventual amputation. ANS: A A nonhealing wound needs the expertise of the Wound Ostomy Care Nurse (or Wound Ostomy Continence Nurse). Premedicating prior to painful procedures and maintaining sterile technique are helpful, but if the wound is not healing, more needs to be done. The client may need an amputation, but other options need to be tried first.

A nurse cares for a client who is prescribed mesalamine (Asacol) for ulcerative colitis. The client states, I am having trouble swallowing this pill. Which action should the nurse take?

a. Contact the clinical pharmacist and request the medication in suspension form. b. Empty the contents of the capsule into applesauce or pudding for administration. c. Ask the health care provider to prescribe the medication as an enema instead. d. Crush the pill carefully and administer it in applesauce or pudding. ANS: C Asacol is the oral formula for mesalamine and is produced as an enteric-coated pill that should not be crushed, chewed, or broken. Asacol is not available as a suspension or elixir. If the client is unable to swallow the Asacol pill, a mesalamine enema (Rowasa) may be administered instead, with a providers order.

While assessing a client who has facial trauma, the nurse auscultates stridor. The client is anxious and restless. Which action should the nurse take first?

a. Contact the provider and prepare for intubation. b. Administer prescribed albuterol nebulizer therapy. c. Place the client in high-Fowlers position. d. Ask the client to perform deep-breathing exercises. ANS: A Facial and neck tissue edema can occur in clients with facial trauma. Airway patency is the highest priority. Clients who experience stridor and hypoxia, manifested by anxiety and restlessness, should be immediately intubated to ensure airway patency. Albuterol decreases bronchi and bronchiole inflammation, not facial and neck edema. Although putting the client in high-Fowlers position and asking the client to perform breathing exercises may temporarily improve the clients comfort, these actions will not decrease the underlying problem or improve airway patency.

A nurse cares for a client newly diagnosed with colon cancer who has become withdrawn from family members. Which action should the nurse take?

a. Contact the provider and recommend a psychiatric consult for the client. b. Encourage the client to verbalize feelings about the diagnosis. c. Provide education about new treatment options with successful outcomes. d. Ask family and friends to visit the client and provide emotional support. ANS: B The nurse recognizes that the client may be expressing feelings of grief. The nurse should encourage the client to verbalize feelings and identify fears to move the client through the phases of the grief process. A psychiatric consult is not appropriate for the client. The nurse should not brush aside the clients feelings with discussions related to cancer prognosis and treatment. The nurse should not assume that the client desires family or friends to visit or provide emotional support.

A nurse assesses a client who reports waking up feeling very tired, even after 8 hours of good sleep. Which action should the nurse take first?

a. Contact the provider for a prescription for sleep medication. b. Tell the client not to drink beverages with caffeine before bed. c. Educate the client to sleep upright in a reclining chair. d. Ask the client if he or she has ever been evaluated for sleep apnea. ANS: D Clients are usually unaware that they have sleep apnea, but it should be suspected in people who have persistent daytime sleepiness and report waking up tired. Causes of the problem should be assessed before the client is offered suggestions for treatment.

A nurse is preparing to administer a blood transfusion. What action is most important?

a. Correctly identifying client using two identifiers b. Ensuring informed consent is obtained if required c. Hanging the blood product with Ringers lactate d. Staying with the client for the entire transfusion ANS: B If the facility requires informed consent for transfusions, this action is most important because it precedes the other actions taken during the transfusion. Correctly identifying the client and blood product is a National Patient Safety Goal, and is the most important action after obtaining informed consent. Ringers lactate is not used to transfuse blood. The nurse does not need to stay with the client for the duration of the transfusion.

A nurse is assessing a client with peripheral artery disease (PAD). The client states walking five blocks is possible without pain. What question asked next by the nurse will give the best information?

a. Could you walk further than that a few months ago? b. Do you walk mostly uphill, downhill, or on flat surfaces? c. Have you ever considered swimming instead of walking? d. How much pain medication do you take each day? ANS: A As PAD progresses, it takes less oxygen demand to cause pain. Needing to cut down on activity to be pain free indicates the clients disease is worsening. The other questions are useful, but not as important.

A nurse obtains a focused health history for a client who is scheduled for magnetic resonance imaging (MRI). Which condition should alert the nurse to contact the provider and cancel the procedure?

a. Creatine phosphokinase (CPK) of 100 IU/L b. Atrioventricular graft c. Blood urea nitrogen (BUN) of 50 mg/dL *d. Internal insulin pump* ANS: D Metal devices such as internal pumps, pacemakers, and prostheses interfere with the accuracy of the image and can become displaced by the magnetic force generated by an MRI procedure. An atrioventricular graft does not contain any metal. CPK and BUN levels have no impact on an MRI procedure.

A nurse caring for a client with sickle cell disease (SCD) reviews the clients laboratory work. Which finding should the nurse report to the provider?

a. Creatinine: 2.9 mg/dL b. Hematocrit: 30% c. Sodium: 147 mEq/L d. White blood cell count: 12,000/mm3 ANS: A An elevated creatinine indicates kidney damage, which occurs in SCD. A hematocrit level of 30% is an expected finding, as is a slightly elevated white blood cell count. A sodium level of 147 mEq/L, although slightly high, is not concerning.

A nurse assesses a client who is hospitalized for botulism. The clients vital signs are temperature: 99.8 F (37.6 C), heart rate: 100 beats/min, respiratory rate: 10 breaths/min, and blood pressure: 100/62 mm Hg. Which action should the nurse take?

a. Decrease stimulation and allow the client to rest. b. Stay with the client while another nurse calls the provider. c. Increase the clients intravenous fluid replacement rate. d. Check the clients blood glucose and administer orange juice. ANS: B A client with botulism is at risk for respiratory failure. This clients respiratory rate is slow, which could indicate impending respiratory distress or failure. The nurse should remain with the client while another nurse notifies the provider. The nurse should monitor and document the IV infusion per protocol, but this client does not require additional intravenous fluids. Allowing the client to rest or checking the clients blood glucose and administering orange juice are not appropriate actions.

A nurse administers prescribed adenosine (Adenocard) to a client. Which response should the nurse assess for as the expected therapeutic response?

a. Decreased intraocular pressure b. Increased heart rate c. Short period of asystole d. Hypertensive crisis ANS: C Clients usually respond to adenosine with a short period of asystole, bradycardia, hypotension, dyspnea, and chest pain. Adenosine has no conclusive impact on intraocular pressure.

A nursing student is studying nutritional problems and learns that kwashiorkor is distinguished from marasmus with which finding?

a. Deficit of calories b. Lack of all nutrients c. Specific lack of protein d. Unknown cause of malnutrition ANS: C Kwashiorkor is a lack of protein when total calories are adequate. Marasmus is a caloric malnutrition.

The nurse learns that the pathophysiology of Guillain-Barr syndrome includes segmental demyelination. The nurse should understand that this causes what?

a. Delayed afferent nerve impulses b. Paralysis of affected muscles c. Paresthesia in upper extremities *d. Slowed nerve impulse transmission* ANS: D Demyelination leads to slowed nerve impulse transmission. The other options are not correct.

A nurse is caring for a client who has the following laboratory results: potassium 3.4 mEq/L, magnesium 1.8 mEq/L, calcium 8.5 mEq/L, sodium 144 mEq/L. Which assessment should the nurse complete first?

a. Depth of respirations b. Bowel sounds c. Grip strength d. Electrocardiography ANS: A A client with a low serum potassium level may exhibit hypoactive bowel sounds, cardiac dysrhythmias, and muscle weakness resulting in shallow respirations and decreased handgrips. The nurse should assess the clients respiratory status first to ensure respirations are sufficient. The respiratory assessment should include rate and depth of respirations, respiratory effort, and oxygen saturation. The other assessments are important but are secondary to the clients respiratory status.

A nurse is caring for a morbidly obese client. What comfort measure is most important for the nurse to delegate to the unlicensed assistive personnel (UAP)?

a. Designating quiet time so the client can rest b. Ensuring siderails are not causing excess pressure c. Providing oral care before and after meals and snacks d. Relaying any reports of pain to the registered nurse ANS: B All actions are good for client comfort, but when dealing with an obese client, the staff should take extra precautions, such as ensuring the siderails are not putting pressure on the clients tissues. The other options are appropriate for any client, and are not specific to obese clients.

A nurse is caring for a client who has the following arterial blood values: pH 7.12, PaO2 56 mm Hg, PaCO2 65 mm Hg, and HCO3 22 mEq/L. Which clinical situation should the nurse correlate with these values?

a. Diabetic ketoacidosis in a person with emphysema b. Bronchial obstruction related to aspiration of a hot dog c. Anxiety-induced hyperventilation in an adolescent d. Diarrhea for 36 hours in an older, frail woman ANS: B Arterial blood gas values indicate that the client has acidosis with normal levels of bicarbonate, suggesting that the problem is not metabolic. Arterial concentrations of oxygen and carbon dioxide are abnormal, with low oxygen and high carbon dioxide levels. Thus, this client has respiratory acidosis from inadequate gas exchange. The fact that the bicarbonate level is normal indicates that this is an acute respiratory problem rather than a chronic problem, because no renal compensation has occurred.

A nurse evaluates the following arterial blood gas values in a client: pH 7.48, PaO2 98 mm Hg, PaCO2 28 mm Hg, and HCO3 22 mEq/L. Which client condition should the nurse correlate with these results?

a. Diarrhea and vomiting for 36 hours b. Anxiety-induced hyperventilation c. Chronic obstructive pulmonary disease (COPD) d. Diabetic ketoacidosis and emphysema ANS: B The elevated pH level indicates alkalosis. The bicarbonate level is normal, and so is the oxygen partial pressure. Loss of carbon dioxide is the cause of the alkalosis, which would occur in response to hyperventilation. Diarrhea and vomiting would cause metabolic alterations, COPD would lead to respiratory acidosis, and the client with emphysema most likely would have combined metabolic acidosis on top of a mild, chronic respiratory acidosis.

A nurse assesses a client with Crohns disease and colonic strictures. Which clinical manifestation should alert the nurse to urgently contact the health care provider?

a. Distended abdomen b. Temperature of 100.0 F (37.8 C) c. Loose and bloody stool d. Lower abdominal cramps ANS: A The presence of strictures predisposes the client to intestinal obstruction. Abdominal distention may indicate that the client has developed an obstruction of the large bowel, and the clients provider should be notified right away. Low-grade fever, bloody diarrhea, and abdominal cramps are common symptoms of Crohns disease.

A nurse assesses a client who presents with renal calculi. Which question should the nurse ask?

a. Do any of your family members have this problem? b. Do you drink any cranberry juice? c. Do you urinate after sexual intercourse? d. Do you experience burning with urination? ANS: A There is a strong association between family history and stone formation and recurrence. Nephrolithiasis is associated with many genetic variations; therefore, the nurse should ask whether other family members have also had renal stones. The other questions do not refer to renal calculi but instead are questions that should be asked of a client with a urinary tract infection.

The clients chart indicates a sensorineural hearing loss. What assessment question does the nurse ask to determine the possible cause?

a. Do you feel like something is in your ear? b. Do you have frequent ear infections? *c. Have you been exposed to loud noises?* d. Have you been told your ear bones dont move? ANS: C Sensorineural hearing loss can occur from damage to the cochlea, the eighth cranial nerve, or the brain. Exposure to loud music is one etiology. The other questions relate to conductive hearing loss.

A nurse cares for a client with chronic obstructive pulmonary disease (COPD) who appears thin and disheveled. Which question should the nurse ask first?

a. Do you have a strong support system? b. What do you understand about your disease? c. Do you experience shortness of breath with basic activities? d. What medications are you prescribed to take each day? ANS: C Clients with severe COPD may not be able to perform daily activities, including bathing and eating, because of excessive shortness of breath. The nurse should ask the client if shortness of breath is interfering with basic activities. Although the nurse should know about the clients support systems, current knowledge, and medications, these questions do not address the clients appearance.

A client has a recurrence of gastric cancer and is in the gastrointestinal clinic crying. What response by the nurse is most appropriate?

a. Do you have family or friends for support? b. Id like to know what you are feeling now. c. Well, we knew this would probably happen. d. Would you like me to refer you to hospice? ANS: B The nurse assesses the clients emotional state with open-ended questions and statements and shows a willingness to listen to the clients concerns. Asking about support people is very limited in nature, and yes-orno questions are not therapeutic. Stating that this was expected dismisses the clients concerns. The client may or may not be ready to hear about hospice, and this is another limited, yes-or-no question.

A client has been diagnosed with hypertension but does not take the antihypertensive medications because of a lack of symptoms. What response by the nurse is best?

a. Do you have trouble affording your medications? b. Most people with hypertension do not have symptoms. c. You are lucky; most people get severe morning headaches. d. You need to take your medicine or you will get kidney failure. ANS: B Most people with hypertension are asymptomatic, although a small percentage do have symptoms such as headache. The nurse should explain this to the client. Asking about paying for medications is not related because the client has already admitted nonadherence. Threatening the client with possible complications will not increase compliance.

A nurse assesses a client who has a history of heart failure. Which question should the nurse ask to assess the extent of the clients heart failure?

a. Do you have trouble breathing or chest pain? b. Are you able to walk upstairs without fatigue? c. Do you awake with breathlessness during the night? d. Do you have new-onset heaviness in your legs? ANS: B Clients with a history of heart failure generally have negative findings, such as shortness of breath. The nurse needs to determine whether the clients activity is the same or worse, or whether the client identifies a decrease in activity level. Trouble breathing, chest pain, breathlessness at night, and peripheral edema are symptoms of heart failure, but do not provide data that can determine the extent of the clients heart failure.

A nurse obtains the health history of a client with a suspected diagnosis of bladder cancer. Which question should the nurse ask when determining this clients risk factors?

a. Do you smoke cigarettes? b. Do you use any alcohol? c. Do you use recreational drugs? d. Do you take any prescription drugs? ANS: A Smoking is known to be a factor that greatly increases the risk of bladder cancer. Alcohol use, recreational drug use, and prescription drug use (except medications that contain phenacetin) are not known to increase the risk of developing bladder cancer.

A nurse is planning care for a client who is hyperventilating. The clients arterial blood gas values are pH 7.30, PaO2 94 mm Hg, PaCO2 31 mm Hg, and HCO3 26 mEq/L. Which question should the nurse ask when developing this clients plan of care?

a. Do you take any over-the-counter medications? b. You appear anxious. What is causing your distress? c. Do you have a history of anxiety attacks? d. You are breathing fast. Is this causing you to feel light-headed? ANS: B The nurse should assist the client who is experiencing anxiety-induced respiratory alkalosis to identify causes of the anxiety. The other questions will not identify the cause of the acid-base imbalance.

A confused client with pneumonia is admitted with an indwelling catheter in place. During interdisciplinary rounds the following day, which question should the nurse ask the primary health care provider?

a. Do you want daily weights on this client? b. Will the client be able to return home? c. Can we discontinue the indwelling catheter? d. Should we get another chest x-ray today? ANS: C An indwelling catheter dramatically increases the risks of urinary tract infection and urosepsis. Nursing staff should ensure that catheters are left in place only as long as they are medically needed. The nurse should inquire about removing the catheter. All other questions might be appropriate, but because of client safety, this question takes priority.

A nurse assesses an older adult client who has multiple chronic diseases. The clients heart rate is 48 beats/min. Which action should the nurse take first?

a. Document the finding in the chart. b. Initiate external pacing. c. Assess the clients medications. d. Administer 1 mg of atropine. ANS: C Pacemaker cells in the conduction system decrease in number as a person ages, resulting in bradycardia. The nurse should check the medication reconciliation for medications that might cause such a drop in heart rate, then should inform the health care provider. Documentation is important, but it is not the priority action. The heart rate is not low enough for atropine or an external pacemaker to be needed.

.A nurse assesses a client who has a 15-year history of diabetes and notes decreased tactile sensation in both feet. Which action should the nurse take first?

a. Document the finding in the clients chart. b.Assess tactile sensation in the clients hands. *c. Examine the clients feet for signs of injury*. d.Notify the health care provider. ANS: C Diabetic neuropathy is common when the disease is of long duration. The client is at great risk for injury in any area with decreased sensation because he or she is less able to feel injurious events. Feet are common locations for neuropathy and injury, so the nurse should inspect them for any signs of injury. After assessment, the nurse should document findings in the clients chart. Testing sensory perception in the hands may or may not be needed. The health care provider can be notified after assessment and documentation have been completed.

After a total knee replacement, a client is on the postoperative nursing unit with a continuous femoral nerve blockade. On assessment, the nurse notes the clients pulses are 2+/4+ bilaterally; the skin is pale pink, warm, and dry; and the client is unable to dorsiflex or plantarflex the affected foot. What action does the nurse perform next?

a. Document the findings and monitor as prescribed. b. Increase the frequency of monitoring the client. *c. Notify the surgeon or anesthesia provider immediately.* d. Palpate the clients bladder or perform a bladder scan. ANS: C With the femoral nerve block, the client should still be able to dorsiflex and plantarflex the affected foot. Since this client has an abnormal finding, the nurse should notify either the surgeon or the anesthesia provider immediately. Documentation is the last priority. Increasing the frequency of assessment may be a good idea, but first the nurse must notify the appropriate person. Palpating the bladder is not related

A client has a pyloric obstruction and reports sudden muscle weakness. What action by the nurse takes priority?

a. Document the findings in the chart. b. Request an electrocardiogram (ECG). c. Facilitate a serum potassium test. d. Place the client on bedrest. ANS: B Pyloric stenosis can lead to hypokalemia, which is manifested by muscle weakness. The nurse first obtains an ECG because potassium imbalances can lead to cardiac dysrhythmias. A potassium level is also warranted, as is placing the client on bedrest for safety. Documentation should be thorough, but none of these actions takes priority over the ECG.

A nurse auscultates a harsh hollow sound over a clients trachea and larynx. Which action should the nurse take first?

a. Document the findings. b. Administer oxygen therapy. c. Position the client in high-Fowlers position. d. Administer prescribed albuterol ANS: A Bronchial breath sounds, including harsh, hollow, tubular, and blowing sounds, are a normal finding over the trachea and larynx. The nurse should document this finding. There is no need to implement oxygen therapy, administer albuterol, or change the clients position because the finding is normal.

A client receiving a blood transfusion develops anxiety and low back pain. After stopping the transfusion, what action by the nurse is most important?

a. Documenting the events in the clients medical record b. Double-checking the client and blood product identification c. Placing the client on strict bedrest until the pain subsides d. Reviewing the clients medical record for known allergies ANS: B This client had a hemolytic transfusion reaction, most commonly caused by blood type or Rh incompatibility. The nurse should double-check all identifying information for both the client and blood type. Documentation occurs after the client is stable. Bedrest may or may not be needed. Allergies to medications or environmental items is not related.

A nurse is preparing to hang a blood transfusion. Which action is most important?

a. Documenting the transfusion b. Placing the client on NPO status c. Placing the client in isolation d. Putting on a pair of gloves ANS: D To prevent bloodborne illness, the nurse should don a pair of gloves prior to hanging the blood. Documentation is important but not the priority at this point. NPO status and isolation are not needed.

The nurse is caring for a client with leukemia who has the priority problem of fatigue. What action by the client best indicates that an important goal for this problem has been met?

a. Doing activities of daily living (ADLs) using rest periods b. Helping plan a daily activity schedule c. Requesting a sleeping pill at night d. Telling visitors to leave when fatigued ANS: A Fatigue is a common problem for clients with leukemia. This client is managing his or her own ADLs using rest periods, which indicates an understanding of fatigue and how to control it. Helping to plan an activity schedule is a lesser indicator. Requesting a sleeping pill does not help control fatigue during the day. Asking visitors to leave when tired is another lesser indicator. Managing ADLs using rest periods demonstrates the most comprehensive management strategy.

A nurse who is applying eyedrops to a client holds pressure against the corner of the eye nearest the nose after instilling the drops. The client asks what the nurse is doing. What response by the nurse is best?

a. Doing this allows time for absorption. b. I am keeping the drops in the eye. *c. This prevents systemic absorption*. d. I am stopping you from rubbing your eye. ANS: C This technique, called punctal occlusion, prevents eyedrops from being absorbed systemically. The other answers are inaccurate.

A nurse teaches a client who has viral gastroenteritis. Which dietary instruction should the nurse include in this clients teaching?

a. Drink plenty of fluids to prevent dehydration. b. You should only drink 1 liter of fluids daily. c. Increase your protein intake by drinking more milk. d. Sips of cola or tea may help to relieve your nausea. ANS: A The client should drink plenty of fluids to prevent dehydration. Milk products may not be tolerated. Caffeinated beverages increase intestinal motility and should be avoided.

.A nurse cares for a client who has type 1 diabetes mellitus. The client asks, Is it okay for me to have an occasional glass of wine? How should the nurse respond?

a. Drinking any wine or alcohol will increase your insulin requirements. b.Because of poor kidney function, people with diabetes should avoid alcohol. c. You should not drink alcohol because it will make you hungry and overeat. *d.One glass of wine is okay with a meal and is counted as two fat exchanges*. ANS: D Under normal circumstances, blood glucose levels will not be affected by moderate use of alcohol when diabetes is well controlled. Because alcohol can induce hypoglycemia, it should be ingested with or shortly after a meal. One alcoholic beverage is substituted for two fat exchanges when caloric intake is calculated. Kidney function is not impacted by alcohol intake. Alcohol is not associated with increased hunger or overeating.

A nurse assesses a client with a neurologic disorder. Which assessment finding should the nurse identify as a late manifestation of amyotrophic lateral sclerosis (ALS)?

a. Dysarthria b. Dysphagia c. Muscle weakness *d. Impairment of respiratory muscles* ANS: D In ALS, progressive muscle atrophy occurs until a flaccid quadriplegia develops. Eventually, the respiratory muscles are involved, which leads to respiratory compromise. Dysarthria, dysphagia, and muscle weakness are early clinical manifestations of ALS.

A nurse teaches a client who is at risk for colon cancer. Which dietary recommendation should the nurse teach this client?

a. Eat low-fiber and low-residual foods. b. White rice and bread are easier to digest. c. Add vegetables such as broccoli and cauliflower to your new diet. d. Foods high in animal fat help to protect the intestinal mucosa. ANS: C The client should be taught to modify his or her diet to decrease animal fat and refined carbohydrates. The client should also increase high-fiber foods and Brassica vegetables, including broccoli and cauliflower, which help to protect the intestinal mucosa from colon cancer.

A client tells the nurse about losing weight and regaining it multiple times. Besides eating and exercising habits, for what additional data should the nurse assess as the priority?

a. Economic ability to join a gym b. Food allergies and intolerances c. Psychosocial influences on weight d. Reasons for wanting to lose weight ANS: C While all topics might be important to assess, people who lose and gain weight in cycles often are depressed or have poor self-esteem, which has a negative effect on weight-loss efforts. The nurse assesses the clients psychosocial status as the priority.

A nurse is caring for a client with a history of renal insufficiency who is scheduled for a computed tomography scan of the head with contrast medium. Which priority intervention should the nurse implement?

a. Educate the client about strict bedrest after the procedure. b. Place an indwelling urinary catheter to closely monitor output. *c. Obtain a prescription for intravenous fluids*. d. Contact the provider to cancel the procedure. ANS: C If a contrast medium is used, intravenous fluid may be given to promote excretion of the contrast medium. Contrast medium also may act as a diuretic, resulting in the need for fluid replacement. The client will not require bedrest. Although urinary output should be monitored closely, there is no need for an indwelling urinary catheter. There is no need to cancel the procedure as long as actions are taken to protect the kidneys

A clients intraocular pressure (IOP) is 28 mm Hg. What action by the nurse is best?

a. Educate the client on corneal transplantation. b. Facilitate scheduling the eye surgery. *c. Plan to teach about drugs for glaucoma*. d. Refer the client to local Braille classes. ANS: C This increased IOP indicates glaucoma. The nurses main responsibility is teaching the client about drug therapy. Corneal transplantation is not used in glaucoma. Eye surgery is not indicated at this time. Braille classes are also not indicated at this time.

A client is in the family practice clinic reporting a severe cold that started 4 days ago. On examination, the nurse notes the client also has a severe headache and muscle aches. What action by the nurse is best?

a. Educate the client on oseltamivir (Tamiflu). b.Facilitate admission to the hospital. c. Instruct the client to have a flu vaccine. d.Teach the client to sneeze in the upper sleeve. ANS: D Sneezing and coughing into ones sleeve helps prevent the spread of upper respiratory infections. The client does have manifestations of the flu (influenza), but it is too late to start antiviral medications; to be effective, they must be started within 24 to 48 hours of symptom onset. The client does not need hospital admission. The client should be instructed to have a flu vaccination, but now that he or she has the flu, vaccination will have to wait until next year.

A client has been diagnosed with tuberculosis (TB). What action by the nurse takes highest priority?

a. Educating the client on adherence to the treatment regimen b.Encouraging the client to eat a well-balanced diet c. Informing the client about follow-up sputum cultures d.Teaching the client ways to balance rest with activity ANS: A The treatment regimen for TB ranges from 6 to 12 months, making adherence problematic for many people. The nurse should stress the absolute importance of following the treatment plan for the entire duration of prescribed therapy. The other options are appropriate topics to educate this client on but do not take priority.

A client has a tracheostomy tube in place. When the nurse suctions the client, food particles are noted. What action by the nurse is best?

a. Elevate the head of the clients bed. b. Measure and compare cuff pressures. c. Place the client on NPO status. d. Request that the client have a swallow study. ANS: B Constant pressure from the tracheostomy tube cuff can cause tracheomalacia, leading to dilation of the tracheal passage. This can be manifested by food particles seen in secretions or by noting that larger and larger amounts of pressure are needed to keep the tracheostomy cuff inflated. The nurse should measure the pressures and compare them to previous ones to detect a trend. Elevating the head of the bed, placing the client on NPO status, and requesting a swallow study will not correct this situation.

A nurse assesses a client 2 hours after a cardiac angiography via the left femoral artery. The nurse notes that the left pedal pulse is weak. Which action should the nurse take?

a. Elevate the leg and apply a sandbag to the entrance site. b. Increase the flow rate of intravenous fluids. c. Assess the color and temperature of the left leg. d. Document the finding as left pedal pulse of +1/4. ANS: C Loss of a pulse distal to an angiography entry site is serious, indicating a possible arterial obstruction. The pulse may be faint because of edema. The left pulse should be compared with the right, and pulses should be compared with previous assessments, especially before the procedure. Assessing color (pale, cyanosis) and temperature (cool, cold) will identify a decrease in circulation. Once all peripheral and vascular assessment data are acquired, the primary health care provider should be notified. Simply documenting the findings is inappropriate. The leg should be positioned below the level of the heart or dangling to increase blood flow to the distal portion of the leg. Increasing intravenous fluids will not address the clients problem.

20. A nurse cares for a client who has a new colostomy. Which action should the nurse take?

a. Empty the pouch frequently to remove excess gas collection. b. Change the ostomy pouch and wafer every morning. c. Allow the pouch to completely fill with stool prior to emptying it. d. Use surgical tape to secure the pouch and prevent leakage. ANS: A The nurse should empty the new ostomy pouch frequently because of excess gas collection, and empty the pouch when it is one-third to one-half full of stool. The ostomy pouch does not need to be changed every morning. Ostomy wafers with paste should be used to secure and seal the ostomy appliance; surgical tape should not be used.

A nurse is caring for several older clients in the hospital that the nurse identifies as being at high risk for healthcare-associated pneumonia. To reduce this risk, what activity should the nurse delegate to the unlicensed assistive personnel (UAP)?

a. Encourage between-meal snacks. b.Monitor temperature every 4 hours. c. Provide oral care every 4 hours. d.Report any new onset of cough. ANS: C Oral colonization by gram-negative bacteria is a risk factor for healthcare-associated pneumonia. Good, frequent oral care can help prevent this from developing and is a task that can be delegated to the UAP. Encouraging good nutrition is important, but this will not prevent pneumonia. Monitoring temperature and reporting new cough in clients is important to detect the onset of possible pneumonia but do not prevent it.

A hospitalized client has a platelet count of 58,000/mm3. What action by the nurse is best?

a. Encourage high-protein foods. b. Institute neutropenic precautions. c. Limit visitors to healthy adults. d. Place the client on safety precautions. ANS: D With a platelet count between 40,000 and 80,000/mm3, clients are at risk of prolonged bleeding even after minor trauma. The nurse should place the client on safety precautions. High-protein foods, while healthy, are not the priority. Neutropenic precautions are not needed as the clients white blood cell count is not low. Limiting visitors would also be more likely related to a low white blood cell count.

A client has a serum ferritin level of 8 ng/mL and microcytic red blood cells. What action by the nurse is best?

a. Encourage high-protein foods. b. Perform a Hemoccult test on the clients stools. c. Offer frequent oral care. d. Prepare to administer cobalamin (vitamin B12). ANS: B This client has laboratory findings indicative of iron deficiency anemia. The most common cause of this disorder is blood loss, often from the GI tract. The nurse should perform a Hemoccult test on the clients stools. High-protein foods may help the condition, but dietary interventions take time to work. That still does not determine the cause. Frequent oral care is not related. Cobalamin injections are for pernicious anemia.

A nurse is caring for a client who has a serum calcium level of 14 mg/dL. Which provider order should the nurse implement first?

a. Encourage oral fluid intake. b. Connect the client to a cardiac monitor. c. Assess urinary output. d. Administer oral calcitonin (Calcimar). ANS: B This client has hypercalcemia. Elevated serum calcium levels can decrease cardiac output and cause cardiac dysrhythmias. Connecting the client to a cardiac monitor is a priority to assess for lethal cardiac changes. Encouraging oral fluids, assessing urine output, and administering calcitonin are treatments for hypercalcemia, but are not the highest priority.

A nurse assesses a client who is prescribed fluticasone (Flovent) and notes oral lesions. Which action should the nurse take?

a. Encourage oral rinsing after fluticasone administration. b. Obtain an oral specimen for culture and sensitivity. c. Start the client on a broad-spectrum antibiotic. d. Document the finding as a known side effect. ANS: A The drug reduces local immunity and increases the risk for local infection, especially Candida albicans. Rinsing the mouth after using the inhaler will decrease the risk for developing this infection. Use of mouthwash and broad-spectrum antibiotics is not warranted in this situation. The nurse should document the finding, but the best action to take is to have the client start rinsing his or her mouth after using fluticasone. An oral specimen for culture and sensitivity will not provide information necessary to care for this client.

The nurse assesses a clients oral cavity and makes the discovery shown in the photo below: What action by the nurse is most appropriate?

a. Encourage the client to have genetic testing. b. Instruct the client on high-fiber foods. c. Place the client in protective precautions. d. Teach the client about cobalamin therapy. ANS: D This condition is known as glossitis, and is characteristic of B12 anemia. If the anemia is a pernicious anemia, it is treated with cobalamin. Genetic testing is not a priority for this condition. The client does not need highfiber foods or protective precautions.

A nurse is caring for an older adult client who has a pulmonary infection. Which action should the nurse take first?

a. Encourage the client to increase fluid intake. b. Assess the clients level of consciousness. c. Raise the head of the bed to at least 45 degrees. d. Provide the client with humidified oxygen. ANS: B Assessing the clients level of consciousness will be most important because it will show how the client is responding to the presence of the infection. Although it will be important for the nurse to encourage the client to turn, cough, and frequently breathe deeply; raise the head of the bed; increase oral fluid intake; and humidify the oxygen administered, none of these actions will be as important as assessing the level of consciousness. Also, the client who has a pulmonary infection may not be able to cough effectively if an area of abscess is present.

A nurse plans care for a client with lower back pain from a work-related injury. Which intervention should the nurse include in this clients plan of care?

a. Encourage the client to stretch the back by reaching toward the toes. b. Massage the affected area with ice twice a day. *c. Apply a heating pad for 20 minutes at least four times daily*. d. Advise the client to avoid warm baths or showers. ANS: C Heat increases blood flow to the affected area and promotes healing of injured nerves. Stretching and ice will not promote healing, and there is no need to avoid warm baths or showers.

A nurse assesses a client with diabetes mellitus 3 hours after a surgical procedure and notes the clients breath has a fruity odor. Which action should the nurse take?

a. Encourage the client to use an incentive spirometer. b.Increase the clients intravenous fluid flow rate. *c. Consult the provider to test for ketoacidosis*. d.Perform meticulous pulmonary hygiene care. ANS: C The stress of surgery increases the action of counterregulatory hormones and suppresses the action of insulin, predisposing the client to ketoacidosis and metabolic acidosis. One manifestation of ketoacidosis is a fruity odor to the breath. Documentation should occur after all assessments have been completed. Using an incentive spirometer, increasing IV fluids, and performing pulmonary hygiene will not address this clients problem.

A client has been treated for a deep vein thrombus and today presents to the clinic with petechiae. Laboratory results show a platelet count of 42,000/mm3. The nurse reviews the clients medication list to determine if the client is taking which drug?

a. Enoxaparin (Lovenox) b. Salicylates (aspirin) c. Unfractionated heparin d. Warfarin (Coumadin) ANS: C This client has manifestations of heparin-induced thrombocytopenia. Enoxaparin, salicylates, and warfarin do not cause this condition.

.A nurse teaches a client with diabetes mellitus who is experiencing numbness and reduced sensation. Which statement should the nurse include in this clients teaching to prevent injury?

a. Examine your feet using a mirror every day. b.Rotate your insulin injection sites every week. c. Check your blood glucose level before each meal. d.Use a bath thermometer to test the water temperature. ANS: D Clients with diminished sensory perception can easily experience a burn injury when bathwater is too hot. Instead of checking the temperature of the water by feeling it, they should use a thermometer. Examining the feet daily does not prevent injury, although daily foot examinations are important to find problems so they can be addressed. Rotating insulin and checking blood glucose levels will not prevent injury.

A nurse assesses an older adult client who is experiencing a myocardial infarction. Which clinical manifestation should the nurse expect?

a. Excruciating pain on inspiration b. Left lateral chest wall pain c. Disorientation and confusion d. Numbness and tingling of the arm ANS: C In older adults, disorientation or confusion may be the major manifestation of myocardial infarction caused by poor cardiac output. Pain manifestations and numbness and tingling of the arm could also be related to the myocardial infarction. However, the nurse should be more concerned about the new onset of disorientation or confusion caused by decreased perfusion.

A client with systemic lupus erythematosus (SLE) was recently discharged from the hospital after an acute exacerbation. The client is in the clinic for a follow-up visit and is distraught about the possibility of another hospitalization disrupting the family. What action by the nurse is best?

a. Explain to the client that SLE is an unpredictable disease. *b. Help the client create backup plans to minimize disruption.* c. Offer to talk to the family and educate them about SLE. d. Tell the client to remain compliant with treatment plans ANS: B SLE is an unpredictable disease and acute exacerbations can occur without warning, creating chaos in the family. Helping the client make backup plans for this event not only will decrease the disruption but will give the client a sense of having more control. Explaining facts about the disease is helpful as well but does not engage the client in problem solving. The family may need education, but again this does not help the client to problem-solve. Remaining compliant may help decrease exacerbations, but is not as powerful an intervention as helping the client plan for such events.

A nurse is assessing a client who has suffered a nasal fracture. Which assessment should the nurse perform first?

a. Facial pain b. Vital signs c. Bone displacement d. Airway patency ANS: D A patent airway is the priority. The nurse first should make sure that the airway is patent and then should determine whether the client is in pain and whether bone displacement or blood loss has occurred.

An older client has decided to give up driving due to cataracts. What assessment information is most important to collect?

a. Family history of visual problems *b. Feelings related to loss of driving* c. Knowledge about surgical options d. Presence of family support ANS: B Loss of driving is often associated with loss of independence, as is decreasing vision. The nurse should assess how the client feels about this decision and what its impact will be. Family history and knowledge about surgical options are not related as the client has made a decision to decline surgery. Family support is also useful information, but it is most important to get the clients perspective on this change.

A client has Crohns disease. What type of anemia is this client most at risk for developing?

a. Folic acid deficiency b. Fanconis anemia c. Hemolytic anemia d. Vitamin B12 anemia ANS: A Malabsorption syndromes such as Crohns disease leave a client prone to folic acid deficiency. Fanconis anemia, hemolytic anemia, and vitamin B12 anemia are not related to Crohns disease.

A nurse cares for a client who is diagnosed with acute rejection 2 months after receiving a simultaneous pancreas-kidney transplant. The client states, I was doing so well with my new organs, and the thought of having to go back to living on hemodialysis and taking insulin is so depressing. How should the nurse respond

a. Following the drug regimen more closely would have prevented this. *b.One acute rejection episode does not mean that you will lose the new organs*. c. Dialysis is a viable treatment option for you and may save your life. d.Since you are on the national registry, you can receive a second transplantation. ANS: B An episode of acute rejection does not automatically mean that the client will lose the transplant. Pharmacologic manipulation of host immune responses at this time can limit damage to the organ and allow the graft to be maintained. The other statements either belittle the client or downplay his or her concerns. The client may not be a candidate for additional organ transplantation.

A client with Sjgrens syndrome reports dry skin, eyes, mouth, and vagina. What nonpharmacologic comfort measure does the nurse suggest?

a. Frequent eyedrops *b. Home humidifier* c. Strong moisturizer d. Tear duct plugs ANS: B A humidifier will help relieve many of the clients Sjgrens syndrome symptoms. Eyedrops and tear duct plugs only affect the eyes, and moisturizer will only help the skin.

A nurse is caring for a client who is experiencing excessive diarrhea. The clients arterial blood gas values are pH 7.28, PaO2 98 mm Hg, PaCO2 45 mm Hg, and HCO3 16 mEq/L. Which provider order should the nurse expect to receive?

a. Furosemide (Lasix) 40 mg intravenous push b. Sodium bicarbonate 100 mEq diluted in 1 L of D5W c. Mechanical ventilation d. Indwelling urinary catheter ANS: B This clients arterial blood gas values represent metabolic acidosis related to a loss of bicarbonate ions from diarrhea. The bicarbonate should be replaced to help restore this clients acid-base balance. Furosemide would cause an increase in acid fluid and acid elimination via the urinary tract; although this may improve the clients pH, the client has excessive diarrhea and cannot afford to lose more fluid. Mechanical ventilation is used to treat respiratory acidosis for clients who cannot keep their oxygen saturation at 90%, or who have respirator muscle fatigue. Mechanical ventilation and an indwelling urinary catheter would not be prescribed for this client.

The nurse is caring for four hypertensive clients. Which druglaboratory value combination should the nurse report immediately to the health care provider?

a. Furosemide (Lasix)/potassium: 2.1 mEq/L b. Hydrochlorothiazide (Hydrodiuril)/potassium: 4.2 mEq/L c. Spironolactone (Aldactone)/potassium: 5.1 mEq/L d. Torsemide (Demadex)/sodium: 142 mEq/L ANS: A Lasix is a loop diuretic and can cause hypokalemia. A potassium level of 2.1 mEq/L is quite low and should be reported immediately. Spironolactone is a potassium-sparing diuretic that can cause hyperkalemia. A potassium level of 5.1 mEq/L is on the high side, but it is not as critical as the low potassium with furosemide. The other two laboratory values are normal.

A client with HIV/AIDS asks the nurse why gabapentin (Neurontin) is part of the drug regimen when the client does not have a history of seizures. What response by the nurse is best?

a. Gabapentin can be used as an antidepressant too. b. I have no idea why you should be taking this drug *c. This drug helps treat the pain from nerve irritation*. d. You are at risk for seizures due to fungal infections ANS: C Many classes of medications are used for neuropathic pain, including tricyclic antidepressants such as gabapentin. It is not being used as an antidepressant or to prevent seizures from fungal infections. If the nurse does not know the answer, he or she should find out for the client.

Which teaching point is most important for the client with bacterial pharyngitis?

a. Gargle with warm salt water. b.Take all antibiotics as directed. c. Use a humidifier in the bedroom. d.Wash hands frequently. ANS: B Any client on antibiotics must be instructed to complete the entire course of antibiotics. Not completing them can lead to complications or drug-resistant strains of bacteria. The other instructions are appropriate, just not the most important.

The nurse caring for clients with gastrointestinal disorders should understand that which category best describes the mechanism of action of sucralfate (Carafate)?

a. Gastric acid inhibitor b. Histamine receptor blocker c. Mucosal barrier fortifier d. Proton pump inhibitor ANS: C Sucralfate is a mucosal barrier fortifier (protector). It is not a gastric acid inhibitor, a histamine receptor blocker, or a proton pump inhibitor.

A nurse is caring for a young male client with lymphoma who is to begin treatment. What teaching topic is a priority?

a. Genetic testing b. Infection prevention c. Sperm banking d. Treatment options ANS: C All teaching topics are important to the client with lymphoma, but for a young male, sperm banking is of particular concern if the client is going to have radiation to the lower abdomen or pelvis.

A client hospitalized with sickle cell crisis frequently asks for opioid pain medications, often shortly after receiving a dose. The nurses on the unit believe the client is drug seeking. When the client requests pain medication, what action by the nurse is best?

a. Give the client pain medication if it is time for another dose. b. Instruct the client not to request pain medication too early. c. Request the provider leave a prescription for a placebo. d. Tell the client it is too early to have more pain medication. ANS: A Clients with sickle cell crisis often have severe pain that is managed with up to 48 hours of IV opioid analgesics. Even if the client is addicted and drug seeking, he or she is still in extreme pain. If the client can receive another dose of medication, the nurse should provide it. The other options are judgmental and do not address the clients pain. Giving placebos is unethical.

A client is receiving plasmapheresis. What action by the nurse best prevents infection in this client?

a. Giving antibiotics prior to treatments b. Monitoring the clients vital signs *c. Performing appropriate hand hygiene* d. Placing the client in protective isolation ANS: C Plasmapheresis is an invasive procedure, and the nurse uses good hand hygiene before and after client contact to prevent infection. Antibiotics are not necessary. Monitoring vital signs does not prevent infection but could alert the nurse to its possibility. The client does not need isolation.

A nurse is teaching a client with diabetes mellitus who asks, Why is it necessary to maintain my blood glucose levels no lower than about 60 mg/dL? How should the nurse respond?

a. Glucose is the only fuel used by the body to produce the energy that it needs. *b.Your brain needs a constant supply of glucose because it cannot store it*. c. Without a minimum level of glucose, your body does not make red blood cells. d.Glucose in the blood prevents the formation of lactic acid and prevents acidosis. ANS: B Because the brain cannot synthesize or store significant amounts of glucose, a continuous supply from the bodys circulation is needed to meet the fuel demands of the central nervous system. The nurse would want to educate the client to prevent hypoglycemia. The body can use other sources of fuel, including fat and protein, and glucose is not involved in the production of red blood cells. Glucose in the blood will encourage glucose metabolism but is not directly responsible for lactic acid formation.

While assessing a clients peripheral IV site, the nurse observes a streak of red along the vein path and palpates a 4-cm venous cord. How should the nurse document this finding?

a. Grade 3 phlebitis at IV site b.Infection at IV site c. Thrombosed area at IV site d.Infiltration at IV site ANS: A The presence of a red streak and palpable cord indicates grade 3 phlebitis. No information in the description indicates that infection, thrombosis, or infiltration is present.

After teaching a client with irritable bowel syndrome (IBS), a nurse assesses the clients understanding. Which menu selection indicates that the client correctly understands the dietary teaching?

a. Ham sandwich on white bread, cup of applesauce, glass of diet cola b. Broiled chicken with brown rice, steamed broccoli, glass of apple juice c. Grilled cheese sandwich, small banana, cup of hot tea with lemon d. Baked tilapia, fresh green beans, cup of coffee with low-fat milk ANS: B Clients with IBS are advised to eat a high-fiber diet (30 to 40 g/day), with 8 to 10 cups of liquid daily. Chicken with brown rice, broccoli, and apple juice has the highest fiber content. They should avoid alcohol, caffeine, and other gastric irritants.

A nurse is irrigating a clients ear when the client becomes nauseated. What action by the nurse is most appropriate for client comfort?

a. Have the client tilt the head back. b. Re-position the client on the other side. c. Slow the rate of the irrigation. *d. Stop the irrigation immediately*. ANS: D During ear irrigation, if the client becomes nauseated, stop the procedure. The other options are not helpful.

A nurse assesses a client who is prescribed alosetron (Lotronex). Which assessment question should the nurse ask this client?

a. Have you been experiencing any constipation? b. Are you eating a diet high in fiber and fluids? c. Do you have a history of high blood pressure? d. What vitamins and supplements are you taking? ANS: A Ischemic colitis is a life-threatening complication of alosetron. The nurse should assess the client for constipation. The other questions do not identify complications related to alosetron.

A nurse obtains a focused health history for a client who is scheduled for magnetic resonance angiography. Which priority question should the nurse ask before the test?

a. Have you had a recent blood transfusion? *b. Do you have allergies to iodine or shellfish?* c. Are you taking any cardiac medications? d. Do you currently use oral contraceptives? ANS: B Allergies to iodine and/or shellfish need to be explored because the client may have a similar reaction to the dye used in the procedure. In some cases, the client may need to be medicated with antihistamines or steroids before the test is given. A recent blood transfusion or current use of cardiac medications or oral contraceptives would not affect the angiography

A nurse cares for a client who has food poisoning resulting from a Clostridium botulinum infection. Which assessment should the nurse complete first?

a. Heart rate and rhythm b. Bowel sounds c. Urinary output d. Respiratory rate ANS: D Severe infection with C. botulinum can lead to respiratory failure, so assessments of oxygen saturation and respiratory rate are of high priority for clients with suspected C. botulinum infection. The other assessments may be completed after the respiratory system has been assessed.

A nurse assesses a client who had a myocardial infarction and is hypotensive. Which additional assessment finding should the nurse expect?

a. Heart rate of 120 beats/min b. Cool, clammy skin c. Oxygen saturation of 90% d. Respiratory rate of 8 breaths/min ANS: A When a client experiences hypotension, baroreceptors in the aortic arch sense a pressure decrease in the vessels. The parasympathetic system responds by lessening the inhibitory effect on the sinoatrial node. This results in an increase in heart rate and respiratory rate. This tachycardia is an early response and is seen even when blood pressure is not critically low. An increased heart rate and respiratory rate will compensate for the low blood pressure and maintain oxygen saturations and perfusion. The client may not be able to compensate for long, and decreased oxygenation and cool, clammy skin will occur later.

A nurse assesses a client with pericarditis. Which assessment finding should the nurse expect to find?

a. Heart rate that speeds up and slows down b. Friction rub at the left lower sternal border c. Presence of a regular gallop rhythm d. Coarse crackles in bilateral lung bases ANS: B The client with pericarditis may present with a pericardial friction rub at the left lower sternal border. This sound is the result of friction from inflamed pericardial layers when they rub together. The other assessments are not related.

A client with rheumatoid arthritis (RA) has an acutely swollen, red, and painful joint. What nonpharmacologic treatment does the nurse apply?

a. Heating pad *b. Ice packs* c. Splints d. Wax dip ANS: B Ice is best for acute inflammation. Heat often helps with joint stiffness. Splinting helps preserve joint function. A wax dip is used to provide warmth to the joint which is more appropriate for chronic pain and stiffness.

A nurse prepares to teach a client who has experienced damage to the left temporal lobe of the brain. Which action should the nurse take when providing education about newly prescribed medications to this client?

a. Help the client identify each medication by its color. b. Provide written materials with large print size. *c. Sit on the clients right side and speak into the right ear*. d. Allow the client to use a white board to ask questions. ANS: C The temporal lobe contains the auditory center for sound interpretation. The clients hearing will be impaired in the left ear. The nurse should sit on the clients right side and speak into the right ear. The other interventions do not address the clients left temporal lobe damage

A client with sickle cell disease (SCD) takes hydroxyurea (Droxia). The client presents to the clinic reporting an increase in fatigue. What laboratory result should the nurse report immediately?

a. Hematocrit: 25% b. Hemoglobin: 9.2 mg/dL c. Potassium: 3.2 mEq/L d. White blood cell count: 38,000/mm3 ANS: D Although individuals with SCD often have elevated white blood cell (WBC) counts, this extreme elevation could indicate leukemia, a complication of taking hydroxyurea. The nurse should report this finding immediately. Alternatively, it could indicate infection, a serious problem for clients with SCD. Hematocrit and hemoglobin levels are normally low in people with SCD. The potassium level, while slightly low, is not as worrisome as the WBCs.

After teaching the wife of a client who has Parkinson disease, the nurse assesses the wifes understanding. Which statement by the clients wife indicates she correctly understands changes associated with this disease?

a. His masklike face makes it difficult to communicate, so I will use a white board. b. He should not socialize outside of the house due to uncontrollable drooling. c. This disease is associated with anxiety causing increased perspiration. *d. He may have trouble chewing, so I will offer bite-sized portions*. ANS: D Because chewing and swallowing can be problematic, small frequent meals and a supplement are better for meeting the clients nutritional needs. A masklike face and drooling are common in clients with Parkinson disease. The client should be encouraged to continue to socialize and communicate as normally as possible. The wife should understand that the clients masklike face can be misinterpreted and additional time may be needed for the client to communicate with her or others. Excessive perspiration is also common in clients with Parkinson disease and is associated with the autonomic nervous systems response.

A student nurse is providing tracheostomy care. What action by the student requires intervention by the instructor?

a. Holding the device securely when changing ties b. Suctioning the client first if secretions are present c. Tying a square knot at the back of the neck d. Using half-strength peroxide for cleansing ANS: C To prevent pressure ulcers and for client safety, when ties are used that must be knotted, the knot should be placed at the side of the clients neck, not in back. The other actions are appropriate.

The student nurse is performing a Weber tuning fork test. What technique is most appropriate?

a. Holding the vibrating tuning fork 10 to 12 inches from the clients ear *b. Placing the vibrating fork in the middle of the clients head* c. Starting by placing the vibrating fork on the mastoid process d. Tapping the vibrating tuning fork against the bridge of the nose ANS: B The Weber tuning fork test includes placing the vibrating tuning fork in the middle of the clients head and asking in which ear the client hears the vibrations louder. The other techniques are incorrect.

A nurse is providing care after auscultating clients breath sounds. Which assessment finding is correctly matched to the nurses primary intervention?

a. Hollow sounds are heard over the trachea. The nurse increases the oxygen flow rate. b. Crackles are heard in bases. The nurse encourages the client to cough forcefully. c. Wheezes are heard in central areas. The nurse administers an inhaled bronchodilator. d. Vesicular sounds are heard over the periphery. The nurse has the client breathe deeply. ANS: C Wheezes are indicative of narrowed airways, and bronchodilators help to open the air passages. Hollow sounds are typically heard over the trachea, and no intervention is necessary. If crackles are heard, the client may need a diuretic. Crackles represent a deep interstitial process, and coughing forcefully will not help the client expectorate secretions. Vesicular sounds heard in the periphery are normal and require no intervention.

The student nurse studying the gastrointestinal system understands that chyme refers to what?

a. Hormones that reduce gastric acidity b. Liquefied food ready for digestion c. Nutrients after being absorbed d. Secretions that help digest food ANS: B Before being digested, food must be broken down into a liquid form. This liquid is called chyme. Secretin is the hormone that inhibits acid production and decreases gastric motility. Absorption is carried out as the nutrients produced by digestion move from the lumen of the GI tract into the bodys circulatory system for uptake by individual cells. The secretions that help digest food include hydrochloric acid, bile, and digestive enzymes.

A nurse assesses a client with early-onset multiple sclerosis (MS). Which clinical manifestation should the nurse expect to find?

a. Hyperresponsive reflexes b. Excessive somnolence *c. Nystagmus* d. Heat intolerance ANS: C Early signs and symptoms of MS include changes in motor skills, vision, and sensation. Hyperresponsive reflexes, excessive somnolence, and heat intolerance are later manifestations of MS.

A nurse asks a client to take deep breaths during an electroencephalography. The client asks, Why are you asking me to do this? How should the nurse respond?

a. Hyperventilation causes vascular dilation of cerebral arteries, which decreases electoral activity in the brain. b. Deep breathing helps you to relax and allows the electroencephalograph to obtain a better waveform. *c. Hyperventilation causes cerebral vasoconstriction and increases the likelihood of seizure activity.* d. Deep breathing will help you to blow off carbon dioxide and decreases intracranial pressures. ANS: C Hyperventilation produces cerebral vasoconstriction and alkalosis, which increases the likelihood of seizure activity. The client is asked to breathe deeply 20 to 30 times for 3 minutes. The other responses are not accurate.

A client has thrombocytopenia. What client statement indicates the client understands self-management of this condition?

a. I brush and use dental floss every day. b. I chew hard candy for my dry mouth. c. I usually put ice on bumps or bruises. d. Nonslip socks are best when I walk. ANS: C The client should be taught to apply ice to areas of minor trauma. Flossing is not recommended. Hard foods should be avoided. The client should wear well-fitting shoes when ambulating.

A nurse is discharging a client after a total hip replacement. What statement by the client indicates good potential for self-management?

a. I can bend down to pick something up. b. I no longer need to do my exercises. *c. I will not sit with my legs crossed*. d. I wont wash my incision to keep it dry. ANS: C There are many precautions clients need to take after hip replacement surgery, including not bending more than 90 degrees at the hips, continuing prescribed exercises, not crossing the legs, and washing the incision daily and patting it dry.

When teaching a client recently diagnosed with type 1 diabetes mellitus, the client states, I will never be able to stick myself with a needle. How should the nurse respond?

a. I can give your injections to you while you are here in the hospital. b.Everyone gets used to giving themselves injections. It really does not hurt. c. Your disease will not be managed properly if you refuse to administer the shots *d.Tell me what it is about the injections that are concerning you* ANS: D Devote as much teaching time as possible to insulin injection and blood glucose monitoring. Clients with newly diagnosed diabetes are often fearful of giving themselves injections. If the client is worried about giving the injections, it is best to try to find out what specifically is causing the concern, so it can be addressed. Giving the injections for the client does not promote self-care ability. Telling the client that others give themselves injections may cause the client to feel bad. Stating that you dont know another way to manage the disease is dismissive of the clients concerns. .

A client has peripheral arterial disease (PAD). What statement by the client indicates misunderstanding about self-management activities?

a. I can use a heating pad on my legs if its set on low. b. I should not cross my legs when sitting or lying down. c. I will go out and buy some warm, heavy socks to wear. d. Its going to be really hard but I will stop smoking. ANS: A Clients with PAD should never use heating pads as skin sensitivity is diminished and burns can result. The other statements show good understanding of self-management.

After providing discharge teaching, a nurse assesses the clients understanding regarding increased risk for metabolic alkalosis. Which statement indicates the client needs additional teaching?

a. I dont drink milk because it gives me gas and diarrhea. b. I have been taking digoxin every day for the last 15 years. c. I take sodium bicarbonate after every meal to prevent heartburn. d. In hot weather, I sweat so much that I drink six glasses of water each day. ANS: C Excessive oral ingestion of sodium bicarbonate and other bicarbonate-based antacids can cause metabolic alkalosis. Avoiding milk, taking digoxin, and sweating would not lead to increased risk of metabolic alkalosis.

An emergency room nurse obtains the health history of a client. Which statement by the client should alert the nurse to the occurrence of heart failure?

a. I get short of breath when I climb stairs. b. I see halos floating around my head. c. I have trouble remembering things. d. I have lost weight over the past month. ANS: A Dyspnea on exertion is an early manifestation of heart failure and is associated with an activity such as stair climbing. The other findings are not specific to early occurrence of heart failure.

A nurse cares for a client with colon cancer who has a new colostomy. The client states, I think it would be helpful to talk with someone who has had a similar experience. How should the nurse respond?

a. I have a good friend with a colostomy who would be willing to talk with you. b. The enterostomal therapist will be able to answer all of your questions. c. I will make a referral to the United Ostomy Associations of America. d. Youll find that most people with colostomies dont want to talk about them. ANS: C Nurses need to become familiar with community-based resources to better assist clients. The local chapter of the United Ostomy Associations of America has resources for clients and their families, including Ostomates (specially trained visitors who also have ostomies). The nurse should not suggest that the client speak with a personal contact of the nurse. Although the enterostomal therapist is an expert in ostomy care, talking with him or her is not the same as talking with someone who actually has had a colostomy. The nurse should not brush aside the clients request by saying that most people with colostomies do not want to talk about them. Many people are willing to share their ostomy experience in the hope of helping others.

A nurse assesses a client in an outpatient clinic. Which statement alerts the nurse to the possibility of leftsided heart failure?

a. I have been drinking more water than usual. b. I am awakened by the need to urinate at night. c. I must stop halfway up the stairs to catch my breath. d. I have experienced blurred vision on several occasions. ANS: C Clients with left-sided heart failure report weakness or fatigue while performing normal activities of daily living, as well as difficulty breathing, or catching their breath. This occurs as fluid moves into the alveoli. Nocturia is often seen with right-sided heart failure. Thirst and blurred vision are not related to heart failure.

.After teaching a client who has diabetes mellitus and proliferative retinopathy, nephropathy, and peripheral neuropathy, the nurse assesses the clients understanding. Which statement made by the client indicates a correct understanding of the teaching?

a. I have so many complications; exercising is not recommended. b.I will exercise more frequently because I have so many complications. c. I used to run for exercise; I will start training for a marathon. *d.I should look into swimming or water aerobics to get my exercise* ANS: D Exercise is not contraindicated for this client, although modifications based on existing pathology are necessary to prevent further injury. Swimming or water aerobics will give the client exercise without the worry of having the correct shoes or developing a foot injury. The client should not exercise too vigorously.

After teaching a client who is being treated for dehydration, a nurse assesses the clients understanding. Which statement indicates the client correctly understood the teaching?

a. I must drink a quart of water or other liquid each day. b. I will weigh myself each morning before I eat or drink. c. I will use a salt substitute when making and eating my meals. d. I will not drink liquids after 6 PM so I wont have to get up at night. ANS: B One liter of water weighs 1 kg; therefore, a change in body weight is a good measure of excess fluid loss or fluid retention. Weight loss greater than 0.5 lb daily is indicative of excessive fluid loss. The other statements are not indicative of practices that will prevent dehydration.

After teaching a client who is scheduled for magnetic resonance imaging (MRI), the nurse assesses the clients understanding. Which client statement indicates a correct understanding of the teaching?

a. I must increase my fluids because of the dye used for the MRI. b. My urine will be radioactive so I should not share a bathroom. *c. I can return to my usual activities immediately after the MRI*. d. My gag reflex will be tested before I can eat or drink anything. ANS: C No postprocedure restrictions are imposed after MRI. The client can return to normal activities after the test is complete. There are no dyes or radioactive materials used for the MRI; therefore, increased fluids are not needed and the clients urine would not be radioactive. The procedure does not impact the clients gag reflex.

An older client with peripheral vascular disease (PVD) is explaining the daily foot care regimen to the family practice clinic nurse. What statement by the client may indicate a barrier to proper foot care?

a. I nearly always wear comfy sweatpants and house shoes. b. Im glad I get energy assistance so my house isnt so cold. c. My daughter makes sure I have plenty of lotion for my feet. d. My hands shake when I try to do things requiring coordination. ANS: D Clients with PVD need to pay special attention to their feet. Toenails need to be kept short and cut straight across. The client whose hands shake may cause injury when trimming toenails. The nurse should refer this client to a podiatrist. Comfy sweatpants and house shoes are generally loose and not restrictive, which is important for clients with PVD. Keeping the house at a comfortable temperature makes it less likely the client will use alternative heat sources, such as heating pads, to stay warm. The client should keep the feet moist and soft with lotion.

.After teaching a client with type 2 diabetes mellitus, the nurse assesses the clients understanding. Which statement made by the client indicates a need for additional teaching?

a. I need to have an annual appointment even if my glucose levels are in good control. *b.Since my diabetes is controlled with diet and exercise, I must be seen only if I am sick*. c. I can still develop complications even though I do not have to take insulin at this time. d.If I have surgery or get very ill, I may have to receive insulin injections for a short time. ANS: B Clients with diabetes need to be seen at least annually to monitor for long-term complications, including visual changes, microalbuminuria, and lipid analysis. The client may develop complications and may need insulin in the future.

.A nurse has educated a client on isoniazid (INH). What statement by the client indicates teaching has been effective?

a. I need to take extra vitamin C while on INH. b.I should take this medicine with milk or juice. c. I will take this medication on an empty stomach. d.My contact lenses will be permanently stained. ANS: C INH needs to be taken on an empty stomach, either 1 hour before or 2 hours after meals. Extra vitamin B needs to be taken while on the drug. Staining of contact lenses commonly occurs while taking rifampin (Rifadin).

The nurse is teaching a client with chronic obstructive pulmonary disease who has been prescribed continuous oxygen therapy at home. Which statement indicates the client correctly understands the teaching?

a. I plan to wear my oxygen when I exercise and feel short of breath. b. I will use my portable oxygen when grilling burgers in the backyard. c. I plan to use cotton balls to cushion the oxygen tubing on my ears. d. I will only smoke while I am wearing my oxygen via nasal cannula. ANS: C Cotton balls can decrease pressure ulcers from the oxygen tubing. Continuous oxygen orders mean the client should wear the oxygen at all times. Oxygen fuels a fire. Wearing oxygen while grilling and smoking increases the risk for fire.

After teaching a client who has a new colostomy, the nurse provides feedback based on the clients ability to complete self-care activities. Which statement should the nurse include in this feedback?

a. I realize that you had a tough time today, but it will get easier with practice. b. You cleaned the stoma well. Now you need to practice putting on the appliance. c. You seem to understand what I taught you today. What else can I help you with? d. You seem uncomfortable. Do you want your daughter to care for your ostomy? ANS: B The nurse should provide both approval and room for improvement in feedback after a teaching session. Feedback should be objective and constructive, and not evaluative. Reassuring the client that things will improve does not offer anything concrete for the client to work on, nor does it let him or her know what was done well. The nurse should not make the client convey learning needs because the client may not know what else he or she needs to understand. The client needs to become the expert in self-management of the ostomy, and the nurse should not offer to teach the daughter instead of the client.

After teaching a client with a history of renal calculi, the nurse assesses the clients understanding. Which statement made by the client indicates a correct understanding of the teaching?

a. I should drink at least 3 liters of fluid every day. b. I will eliminate all dairy or sources of calcium from my diet. c. Aspirin and aspirin-containing products can lead to stones. d. The doctor can give me antibiotics at the first sign of a stone. ANS: A Dehydration contributes to the precipitation of minerals to form a stone. Although increased intake of calcium causes hypercalcemia and leads to excessive calcium filtered into the urine, if the client is well hydrated the calcium will be excreted without issues. Dehydration increases the risk for supersaturation of calcium in the urine, which contributes to stone formation. The nurse should encourage the client to drink more fluids, not decrease calcium intake. Ingestion of aspirin or aspirin-containing products does not cause a stone. Antibiotics neither prevent nor treat a stone.

After teaching a client who is newly diagnosed with type 2 diabetes mellitus, the nurse assesses the clients understanding. Which statement made by the client indicates a need for additional teaching?

a. I should increase my intake of vegetables with higher amounts of dietary fiber. b.My intake of saturated fats should be no more than 10% of my total calorie intake. *c. I should decrease my intake of protein and eliminate carbohydrates from my diet*. d.My intake of water is not restricted by my treatment plan or medication regimen. ANS: C The client should not completely eliminate carbohydrates from the diet, and should reduce protein if microalbuminuria is present. The client should increase dietary intake of complex carbohydrates, including vegetables, and decrease intake of fat. Water does not need to be restricted unless kidney failure is present.

After teaching a client who has an implantable cardioverter-defibrillator (ICD), a nurse assesses the clients understanding. Which statement by the client indicates a correct understanding of the teaching?

a. I should wear a snug-fitting shirt over the ICD. b. I will avoid sources of strong electromagnetic fields. c. I should participate in a strenuous exercise program. d. Now I can discontinue my antidysrhythmic medication. ANS: B The client being discharged with an ICD is instructed to avoid strong sources of electromagnetic fields. Clients should avoid tight clothing, which could cause irritation over the ICD generator. The client should be encouraged to exercise but should not engage in strenuous activities that cause the heart rate to meet or exceed the ICD cutoff point because the ICD can discharge inappropriately. The client should continue all prescribed medications.

A nurse assesses a client admitted to the cardiac unit. Which statement by the client alerts the nurse to the possibility of right-sided heart failure

a. I sleep with four pillows at night. b. My shoes fit really tight lately. c. I wake up coughing every night. d. I have trouble catching my breath. ANS: B Signs of systemic congestion occur with right-sided heart failure. Fluid is retained, pressure builds in the venous system, and peripheral edema develops. Left-sided heart failure symptoms include respiratory symptoms. Orthopnea, coughing, and difficulty breathing all could be results of left-sided heart failure

A nurse cares for a client with urinary incontinence. The client states, I am so embarrassed. My bladder leaks like a young childs bladder. How should the nurse respond?

a. I understand how you feel. I would be mortified. b. Incontinence pads will minimize leaks in public. c. I can teach you strategies to help control your incontinence. d. More women experience incontinence than you might think. ANS: C The nurse should accept and acknowledge the clients concerns, and assist the client to learn techniques that will allow control of urinary incontinence. The nurse should not diminish the clients concerns with the use of pads or stating statistics about the occurrence of incontinence.

A nurse obtains the health history of a client who is newly admitted to the medical unit. Which statement by the client should alert the nurse to the presence of edema?

a. I wake up to go to the bathroom at night. b. My shoes fit tighter by the end of the day. c. I seem to be feeling more anxious lately. d. I drink at least eight glasses of water a day. ANS: B Weight gain can result from fluid accumulation in the interstitial spaces. This is known as edema. The nurse should note whether the client feels that his or her shoes or rings are tight, and should observe, when present, an indentation around the leg where the socks end. The other answers do not describe edema.

The nurse is caring for a client with lung cancer who states, I dont want any pain medication because I am afraid Ill become addicted. How should the nurse respond?

a. I will ask the provider to change your medication to a drug that is less potent. b. Would you like me to use music therapy to distract you from your pain? c. It is unlikely you will become addicted when taking medicine for pain. d. Would you like me to give you acetaminophen (Tylenol) instead? ANS: C Clients should be encouraged to take their pain medications; addiction usually is not an issue with a client in pain. The nurse would not request that the pain medication be changed unless it was not effective. Other methods to decrease pain can be used, in addition to pain medication.

A nurse cares for a client who is scheduled for the surgical creation of an ileal conduit. The client states, I am anxious about having an ileal conduit. What is it like to have this drainage tube? How should the nurse respond?

a. I will ask the provider to prescribe you an antianxiety medication. b. Would you like to discuss the procedure with your doctor once more? c. I think it would be nice to not have to worry about finding a bathroom. d. Would you like to speak with someone who has an ileal conduit? ANS: D The goal for the client who is scheduled to undergo a procedure such as an ileal conduit is to have a positive self-image and a positive attitude about his or her body. Discussing the procedure candidly with someone who has undergone the same procedure will foster such feelings, especially when the current client has an opportunity to ask questions and voice concerns to someone with first-hand knowledge. Medications for anxiety will not promote a positive self-image and a positive attitude, nor will discussing the procedure once more with the physician or hearing the nurses opinion.

After teaching a client who is prescribed adalimumab (Humira) for severe ulcerative colitis, the nurse assesses the clients understanding. Which statement made by the client indicates a need for additional teaching?

a. I will avoid large crowds and people who are sick. b. I will take this medication with my breakfast each morning. c. Nausea and vomiting are common side effects of this drug. d. I must wash my hands after I play with my dog. ANS: B Adalimumab (Humira) is an immune modulator that must be given via subcutaneous injection. It does not need to be given with food or milk. Nausea and vomiting are two common side effects. Adalimumab can cause immune suppression, so clients receiving the medication should avoid large crowds and people who are sick, and should practice good handwashing.

After teaching a client who is prescribed salmeterol (Serevent), the nurse assesses the clients understanding. Which statement by the client indicates a need for additional teaching?

a. I will be certain to shake the inhaler well before I use it. b. It may take a while before I notice a change in my asthma. c. I will use the drug when I have an asthma attack. d. I will be careful not to let the drug escape out of my nose and mouth. ANS: C Salmeterol is designed to prevent an asthma attack; it does not relieve or reverse symptoms. Salmeterol has a slow onset of action; therefore, it should not be used as a rescue drug. The drug must be shaken well because it has a tendency to separate easily. Poor technique on the clients part allows the drug to escape through the nose and mouth.

After teaching a client who is prescribed a long-acting beta2 agonist medication, a nurse assesses the clients understanding. Which statement indicates the client comprehends the teaching?

a. I will carry this medication with me at all times in case I need it. b. I will take this medication when I start to experience an asthma attack. c. I will take this medication every morning to help prevent an acute attack. d. I will be weaned off this medication when I no longer need it. ANS: C Long-acting beta2 agonist medications will help prevent an acute asthma attack because they are long acting. The client will take this medication every day for best effect. The client does not have to always keep this medication with him or her because it is not used as a rescue medication. This is not the medication the client will use during an acute asthma attack because it does not have an immediate onset of action. The client will not be weaned off this medication because this is likely to be one of his or her daily medications.

A nurse cares for a client who has advanced cardiac disease and states, I am having trouble sleeping at night. How should the nurse respond?

a. I will consult the provider to prescribe a sleep study to determine the problem. b. You become hypoxic while sleeping; oxygen therapy via nasal cannula will help. c. A continuous positive airway pressure, or CPAP, breathing mask will help you breathe at night. d. Use pillows to elevate your head and chest while you are sleeping. ANS: D The client is experiencing orthopnea (shortness of breath while lying flat). The nurse should teach the client to elevate the head and chest with pillows or sleep in a recliner. A sleep study is not necessary to diagnose this client. Oxygen and CPAP will not help a client with orthopnea.

A nurse cares for a client who had a partial laryngectomy 10 days ago. The client states that all food tastes bland. How should the nurse respond?

a. I will consult the speech therapist to ensure you are swallowing properly. b. This is normal after surgery. What types of food do you like to eat? c. I will ask the dietitian to change the consistency of the food in your diet. d. Replacement of protein, calories, and water is very important after surgery. ANS: B Many clients experience changes in taste after surgery. The nurse should identify foods that the client wants to eat to ensure the client maintains necessary nutrition. Although the nurse should collaborate with the speech therapist and dietitian to ensure appropriate replacement of protein, calories, and water, the other responses do not address the clients concerns.

After teaching a client who was malnourished and is being discharged, a nurse assesses the clients understanding. Which statement indicates the client correctly understood teaching to decrease risk for the development of metabolic acidosis?

a. I will drink at least three glasses of milk each day. b. I will eat three well-balanced meals and a snack daily. c. I will not take pain medication and antihistamines together. d. I will avoid salting my food when cooking or during meals. ANS: B Starvation or a diet with too few carbohydrates can lead to metabolic acidosis by forcing cells to switch to using fats for fuel and by creating ketoacids as a by-product of excessive fat metabolism. Eating sufficient calories from all food groups helps reduce this risk.

After teaching a client who was hospitalized for Salmonella food poisoning, a nurse assesses the clients understanding. Which statement made by the client indicates a need for additional teaching?

a. I will let my husband do all of the cooking for my family. b. Ill take the ciprofloxacin until the diarrhea has resolved. c. I should wash my hands with antibacterial soap before each meal. d. I must place my dishes into the dishwasher after each meal. ANS: B Ciprofloxacin should be taken for 10 to 14 days to treat Salmonella infection, and should not be stopped once the diarrhea has cleared. Clients should be advised to take the entire course of medication. People with Salmonella should not prepare foods for others because the infection may be spread in this way. Hands should be washed with antibacterial soap before and after eating to prevent spread of the bacteria. Dishes and eating utensils should not be shared and should be cleaned thoroughly. Clients can be carriers for up to 1 year.

After teaching a client who has stress incontinence, the nurse assesses the clients understanding. Which statement made by the client indicates a need for additional teaching?

a. I will limit my total intake of fluids. b. I must avoid drinking alcoholic beverages. c. I must avoid drinking caffeinated beverages. d. I shall try to lose about 10% of my body weight. ANS: A Limiting fluids concentrates urine and can irritate tissues, leading to increased incontinence. Many people try to manage incontinence by limiting fluids. Alcoholic and caffeinated beverages are bladder stimulants. Obesity increases intra-abdominal pressure, causing incontinence.

After teaching a client with bacterial cystitis who is prescribed phenazopyridine (Pyridium), the nurse assesses the clients understanding. Which statement made by the client indicates a correct understanding of the teaching?

a. I will not take this drug with food or milk. b. If I think I am pregnant, I will stop the drug. c. An orange color in my urine should not alarm me. d. I will drink two glasses of cranberry juice daily. ANS: C Phenazopyridine discolors urine, most commonly to a deep reddish orange. Many clients think they have blood in their urine when they see this. In addition, the urine can permanently stain clothing. Phenazopyridine is safe to take if the client is pregnant. There are no dietary restrictions or needs while taking this medication.

After teaching a client who has a femoral hernia, the nurse assesses the clients understanding. Which statement indicates the client needs additional teaching related to the proper use of a truss?

a. I will put on the truss before I go to bed each night. b. Ill put some powder under the truss to avoid skin irritation. c. The truss will help my hernia because I cant have surgery. d. If I have abdominal pain, Ill let my health care provider know right away. ANS: A The client should be instructed to apply the truss before arising, not before going to bed at night. The other statements show an accurate understanding of using a truss.

After teaching a client who is prescribed voice rest therapy for vocal cord polyps, a nurse assesses the clients understanding. Which statement indicates the client needs further teaching?

a. I will stay away from smokers to minimize inhalation of secondhand smoke. b. When I speak, I will whisper rather than use a normal tone of voice. c. For the next several weeks, I will not lift more than 10 pounds. d. I will drink at least three quarts of water each day to stay hydrated. ANS: B Treatment for vocal cord polyps includes no speaking, no lifting, and no smoking. The client has to be educated not to even whisper when resting the voice. It is also appropriate for the client to stay out of rooms where people are smoking, to stay hydrated, and to use stool softeners.

After teaching a client newly diagnosed with epilepsy, the nurse assesses the clients understanding. Which statement by the client indicates a need for additional teaching?

a. I will wear my medical alert bracelet at all times. b. While taking my epilepsy medications, I will not drink any alcoholic beverages. c. I will tell my doctor about my prescription and over-the-counter medications. *d. If I am nauseated, I will not take my epilepsy medication*. ANS: D The nurse must emphasize that antiepileptic drugs must be taken even if the client is nauseous. Discontinuing the medication can predispose the client to seizure activity and status epilepticus. The client should not drink alcohol while taking seizure medications. The client should wear a medical alert bracelet and should make the doctor aware of all medications to prevent complications of polypharmacy.

.A nurse in a family practice clinic is preparing discharge instructions for a client reporting facial pain that is worse when bending over, tenderness across the cheeks, and postnasal discharge. What instruction will be most helpful?

a. Ice packs may help with the facial pain. b.Limit fluids to dry out your sinuses. c. Try warm, moist heat packs on your face. d.We will schedule you for a computed tomography scan this week. ANS: C This client has rhinosinusitis. Comfort measures for this condition include breathing in warm steam, hot packs, nasal saline irrigations, sleeping with the head elevated, increased fluids, and avoiding cigarette smoke. The client does not need a CT scan.

A nurse delegates care for a client with early-stage Alzheimers disease to an unlicensed assistive personnel (UAP). Which statement should the nurse include when delegating this clients care?

a. If she is confused, play along and pretend that everything is okay. b. Remove the clock from her room so that she doesnt get confused. *c. Reorient the client to the day, time, and environment with each contact*. d. Use validation therapy to recognize and acknowledge the clients concerns. ANS: C Clients who have early-stage Alzheimers disease should be reoriented frequently to person, place, and time. The UAP should reorient the client and not encourage the clients delusions. The room should have a clock and white board with the current date written on it. Validation therapy is used with late-stage Alzheimers disease.

A nursing student is caring for a client with leukemia. The student asks why the client is still at risk for infection when the clients white blood cell count (WBC) is high. What response by the registered nurse is best?

a. If the WBCs are high, there already is an infection present. b. The client is in a blast crisis and has too many WBCs. c. There must be a mistake; the WBCs should be very low. d. Those WBCs are abnormal and dont provide protection. ANS: D In leukemia, the WBCs are abnormal and do not provide protection to the client against infection. The other statements are not accurate.

A nurse is caring for a client with paraplegia who is scheduled to participate in a rehabilitation program. The client states, I do not understand the need for rehabilitation; the paralysis will not go away and it will not get better. How should the nurse respond?

a. If you dont want to participate in the rehabilitation program, Ill let the provider know. b. Rehabilitation programs have helped many clients with your injury. You should give it a chance. *c. The rehabilitation program will teach you how to maintain the functional ability you have and prevent further disability* d. When new discoveries are made regarding paraplegia, people in rehabilitation programs will benefit first. ANS: C Participation in rehabilitation programs has many purposes, including prevention of disability, maintenance of functional ability, and restoration of function. The other responses do not meet this clients needs.

A nurse cares for a client who has a family history of colon cancer. The client states, My father and my brother had colon cancer. What is the chance that I will get cancer? How should the nurse respond?

a. If you eat a low-fat and low-fiber diet, your chances decrease significantly. b. You are safe. This is an autosomal dominant disorder that skips generations. c. Preemptive surgery and chemotherapy will remove cancer cells and prevent cancer. d. You should have a colonoscopy more frequently to identify abnormal polyps early. ANS: D The nurse should encourage the client to have frequent colonoscopies to identify abnormal polyps and cancerous cells early. The abnormal gene associated with colon cancer is an autosomal dominant gene mutation that does not skip a generation and places the client at high risk for cancer. Changing the clients diet, preemptive chemotherapy, and removal of polyps will decrease the clients risk but will not prevent cancer. However, a client at risk for colon cancer should eat a low-fat and high-fiber diet.

After teaching a client who is being discharged home after mitral valve replacement surgery, the nurse assesses the clients understanding. Which client statement indicates a need for additional teaching?

a. Ill be able to carry heavy loads after 6 months of rest. b. I will have my teeth cleaned by my dentist in 2 weeks. c. I must avoid eating foods high in vitamin K, like spinach. d. I must use an electric razor instead of a straight razor to shave. ANS: B Clients who have defective or repaired valves are at high risk for endocarditis. The client who has had valve surgery should avoid dental procedures for 6 months because of the risk for endocarditis. When undergoing a mitral valve replacement surgery, the client needs to be placed on anticoagulant therapy to prevent vegetation forming on the new valve. Clients on anticoagulant therapy should be instructed on bleeding precautions, including using an electric razor. If the client is prescribed warfarin, the client should avoid foods high in vitamin K. Clients recovering from open heart valve replacements should not carry anything heavy for 6 months while the chest incision and muscle heal.

After teaching a client who has diverticulitis, a nurse assesses the clients understanding. Which statement made by the client indicates a need for additional teaching?

a. Ill ride my bike or take a long walk at least three times a week. b. I must try to include at least 25 grams of fiber in my diet every day. c. I will take a laxative nightly at bedtime to avoid becoming constipated. d. I should use my legs rather than my back muscles when I lift heavy objects. ANS: C Laxatives are not recommended for clients with diverticulitis because they can increase pressure in the bowel, causing additional outpouching of the lumen. Exercise and a high-fiber diet are recommended for clients with diverticulitis because they promote regular bowel function. Using the leg muscles rather than the back for lifting prevents abdominal straining.

After teaching a client with perineal excoriation caused by diarrhea from acute gastroenteritis, a nurse assesses the clients understanding. Which statement by the client indicates a need for additional teaching?

a. Ill rinse my rectal area with warm water after each stool and apply zinc oxide ointment. b. I will clean my rectal area thoroughly with toilet paper after each stool and then apply aloe vera gel. c. I must take a sitz bath three times a day and then pat my rectal area gently but thoroughly to make sure I am dry. d. I shall clean my rectal area with a soft cotton washcloth and then apply vitamin A and D ointment. ANS: B Toilet paper can irritate the sensitive perineal skin, so warm water rinses or soft cotton washcloths should be used instead. Although aloe vera may facilitate healing of superficial abrasions, it is not an effective skin barrier for diarrhea. Skin barriers such as zinc oxide and vitamin A and D ointment help protect the rectal area from the excoriating effects of liquid stools. Patting the skin is recommended instead of rubbing the skin dry.

After teaching a client with type 2 diabetes mellitus who is prescribed nateglinide (Starlix), the nurse assesses the clients understanding. Which statement made by the client indicates a correct understanding of the prescribed therapy?

a. Ill take this medicine during each of my meals. b.I must take this medicine in the morning when I wake. c. I will take this medicine before I go to bed. *d.I will take this medicine immediately before I eat*. ANS: D Nateglinide is an insulin secretagogue that is designed to increase meal-related insulin secretion. It should be taken immediately before each meal. The medication should not be taken without eating as it will decrease the clients blood glucose levels. The medication should be taken before meals instead of during meals.

A nurse prepares a client for prescribed magnetic resonance imaging (MRI). Which action should the nurse implement prior to the test?

a. Implement nothing by mouth (NPO) status for 8 hours. b. Withhold all daily medications until after the examination. c. Administer morphine sulfate to prevent claustrophobia during the test. *d. Place the client in a gown that has cloth ties instead of metal snaps.* ANS: D Metal objects are a hazard because of the magnetic field used in the MRI procedure. Morphine sulfate is not administered to prevent claustrophobia; lorazepam (Ativan) or diazepam (Valium) may be used instead. The client does not need to be NPO, and daily medications do not need to be withheld prior to MRI.

A client has been prescribed lorcaserin (Belviq). What teaching is most appropriate?

a. Increase the fiber and water in your diet. b. Reduce fat to less than 30% each day. c. Report dry mouth and decreased sweating. d. Lorcaserin may cause loose stools for a few days. ANS: A This drug can cause constipation, so the client should increase fiber and water in the diet to prevent this from occurring. Reducing fat in the diet is important with orlistat. Lorcaserin can cause dry mouth but not decreased sweating. Loose stools are common with orlistat.

A nurse cares for a client with a 40-year smoking history who is experiencing distended neck veins and dependent edema. Which physiologic process should the nurse correlate with this clients history and clinical manifestations?

a. Increased pulmonary pressure creating a higher workload on the right side of the heart b. Exposure to irritants resulting in increased inflammation of the bronchi and bronchioles c. Increased number and size of mucus glands producing large amounts of thick mucus d. Left ventricular hypertrophy creating a decrease in cardiac output ANS: A Smoking increases pulmonary hypertension, resulting in cor pulmonale, or right-sided heart failure. Increased pressures in the lungs make it more difficult for blood to flow through the lungs. Blood backs up into the right side of the heart and then into the peripheral venous system, creating distended neck veins and dependent edema. Inflammation in bronchi and bronchioles creates an airway obstruction which manifests as wheezes. Thick mucus in the lungs has no impact on distended neck veins and edema. Left ventricular hypertrophy is associated with left heart failure and is not caused by a 40-year smoking history.

A nurse assesses a client with diabetes mellitus who is admitted with an acid-base imbalance. The clients arterial blood gas values are pH 7.36, PaO2 98 mm Hg, PaCO2 33 mm Hg, and HCO3 18 mEq/L. Which manifestation should the nurse identify as an example of the clients compensation mechanism?

a. Increased rate and depth of respirations b. Increased urinary output c. Increased thirst and hunger d. Increased release of acids from the kidneys ANS: A This client has metabolic acidosis. The respiratory system compensates by increasing its activity and blowing off excess carbon dioxide. Increased urinary output, thirst, and hunger are manifestations of hyperglycemia but are not compensatory mechanisms for acid-base imbalances. The kidneys do not release acids.

A nurse is evaluating a client who is being treated for dehydration. Which assessment result should the nurse correlate with a therapeutic response to the treatment plan?

a. Increased respiratory rate from 12 breaths/min to 22 breaths/min b. Decreased skin turgor on the clients posterior hand and forehead c. Increased urine specific gravity from 1.012 to 1.030 g/mL d. Decreased orthostatic light-headedness and dizziness ANS: D The focus of management for clients with dehydration is to increase fluid volumes to normal. When fluid volumes return to normal, clients should perfuse the brain more effectively, therefore improving confusion and decreasing orthostatic light-headedness or dizziness. Increased respiratory rate, decreased skin turgor, and increased specific gravity are all manifestations of dehydration.

A nurse reviews laboratory results for a client with diabetes mellitus who is prescribed an intensified insulin regimen: Fasting blood glucose: 75 mg/dL Postprandial blood glucose: 200 mg/dL Hemoglobin A1c level: 5.5% How should the nurse interpret these laboratory findings?

a. Increased risk for developing ketoacidosis *b.Good control of blood glucose* c. Increased risk for developing hyperglycemia d.Signs of insulin resistance ANS: B The client is maintaining blood glucose levels within the defined ranges for goals in an intensified regimen. Because the clients glycemic control is good, he or she is not at higher risk for ketoacidosis or hyperglycemia and is not showing signs of insulin resistance

A nurse is assessing a client who is recovering from a lung biopsy. Which assessment finding requires immediate action?

a. Increased temperature b. Absent breath sounds c. Productive cough d. Incisional discomfort ANS: B Absent breath sounds may indicate that the client has a pneumothorax, a serious complication after a needle biopsy or open lung biopsy. The other manifestations are not life threatening.

A nurse auscultates a clients lung fields. Which pathophysiologic process should the nurse associate with this breath sound?

a. Inflammation of the pleura b. Constriction of the bronchioles c. Upper airway obstruction d. Pulmonary vascular edema ANS: A A pleural friction rub can be heard when the pleura is inflamed and rubbing against the lung wall. The other pathophysiologic processes would not cause a pleural friction rub. Constriction of the bronchioles may be heard as a wheeze, upper airway obstruction may be heard as stridor, and pulmonary vascular edema may be heard as crackles.

The nurse is administering eyedrops to a client with an infection in the right eye. The drops go in both eyes, and two different bottles are used to administer the drops. The nurse accidentally uses the left eye bottle for the right eye. What action by the nurse is best?

a. Inform the provider of the issue. *b. Obtain a new bottle of eyedrops*. c. Rinse the clients right eye thoroughly. d. Wipe the left eye bottle with alcohol. ANS: B The nurse has contaminated the clean bottle by using it on the infected eye. The nurse needs to obtain a new bottle of solution to use on the left eye. The other actions are not appropriate.

A client has a primary selective immunoglobulin A deficiency. The nurse should prepare the client for selfmanagement by teaching what principle of medical management?

a. Infusions will be scheduled every 3 to 4 weeks. *b. Treatment is aimed at treating specific infections*. c. Unfortunately, there is no effective treatment. d. You will need many immunoglobulin A infusions. ANS: B Treatment for this disorder is vigorous management of infection, not infusion of exogenous immunoglobulins. The other responses are inaccurate.

An HIV-positive client is admitted to the hospital with Toxoplasma gondii infection. Which action by the nurse is most appropriate?

a. Initiate Contact Precautions. b. Place the client on Airborne Precautions. c. Place the client on Droplet Precautions. *d. Use Standard Precautions consistently.* ANS: D Toxoplasma gondii infection is an opportunistic infection that poses no threat to immunocompetent health care workers. Use of Standard Precautions is sufficient to care for this client.

A client with human immune deficiency virus is admitted to the hospital with fever, night sweats, and severe cough. Laboratory results include a CD4+ cell count of 180/mm3 and a negative tuberculosis (TB) skin test 4 days ago. What action should the nurse take first?

a. Initiate Droplet Precautions for the client. b. Notify the provider about the CD4+ results. *c. Place the client under Airborne Precautions*. d. Use Standard Precautions to provide care. ANS: C Since this clients CD4+ cell count is low, he or she may have anergy, or the inability to mount an immune response to the TB test. The nurse should first place the client on Airborne Precautions to prevent the spread of TB if it is present. Next the nurse notifies the provider about the low CD4+ count and requests alterative testing for TB. Droplet Precautions are not used for TB. Standard Precautions are not adequate in this case.

A nurse cares for a client who is experiencing epistaxis. Which action should the nurse take first?

a. Initiate Standard Precautions. b. Apply direct pressure. c. Sit the client upright. d. Loosely pack the nares with gauze. ANS: A The nurse should implement Standard Precautions and don gloves prior to completing the other actions.

.A medical-surgical nurse is concerned about the incidence of complications related to IV therapy, including bloodstream infection. Which intervention should the nurse suggest to the management team to make the biggest impact on decreasing complications?

a. Initiate a dedicated team to insert access devices. b.Require additional education for all nurses. c. Limit the use of peripheral venous access devices. d.Perform quality control testing on skin preparation products. ANS: A The Centers for Disease Control and Prevention recommends having a dedicated IV team to reduce complications, save money, and improve client satisfaction and outcomes. In-service education would always be helpful, but it would not have the same outcomes as an IV team. Limiting IV starts to the most experienced nurses does not allow newer nurses to gain this expertise. The quality of skin preparation products is only one aspect of IV insertion that could contribute to infection.

A nurse assesses a client after administering isosorbide mononitrate (Imdur). The client reports a headache. Which action should the nurse take?

a. Initiate oxygen therapy. b. Hold the next dose of Imdur. c. Instruct the client to drink water. d. Administer PRN acetaminophen. ANS: D The vasodilating effects of isosorbide mononitrate frequently cause clients to have headaches during the initial period of therapy. Clients should be told about this side effect and encouraged to take the medication with food. Some clients obtain relief with mild analgesics, such as acetaminophen. The clients headache is not related to hypoxia or dehydration; therefore, these interventions would not help. The client needs to take the medication as prescribed to prevent angina; the medication should not be held.

A nurse assesses a client with a spinal cord injury at level T5. The clients blood pressure is 184/95 mm Hg, and the client presents with a flushed face and blurred vision. Which action should the nurse take first?

a. Initiate oxygen via a nasal cannula. b. Place the client in a supine position. *c. Palpate the bladder for distention*. d. Administer a prescribed beta blocker ANS: C The client is manifesting symptoms of autonomic dysreflexia. Common causes include bladder distention, tight clothing, increased room temperature, and fecal impaction. If persistent, the client could experience neurologic injury. Precipitating conditions should be eliminated and the physician notified. The other actions would not be appropriate.

A nurse assesses a client who has ulcerative colitis and severe diarrhea. Which assessment should the nurse complete first?

a. Inspection of oral mucosa b. Recent dietary intake c. Heart rate and rhythm d. Percussion of abdomen ANS: C Although the client with severe diarrhea may experience skin irritation and hypovolemia, the client is most at risk for cardiac dysrhythmias secondary to potassium and magnesium loss from severe diarrhea. The client should have her or his electrolyte levels monitored, and electrolyte replacement may be necessary. Oral mucosa inspection, recent dietary intake, and abdominal percussion are important parts of physical assessment but are lower priority for this client than heart rate and rhythm.

An older client is scheduled to have hip replacement in 2 months and has the following laboratory values: white blood cell count: 8900/mm3, red blood cell count: 3.2/mm3, hemoglobin: 9 g/dL, hematocrit: 32%. What intervention by the nurse is most appropriate?

a. Instruct the client to avoid large crowds. *b. Prepare to administer epoetin alfa (Epogen).* c. Teach the client about foods high in iron. d. Tell the client that all laboratory results are normal. ANS: B This client is anemic, which needs correction prior to surgery. While eating iron-rich foods is helpful, to increase the clients red blood cells, hemoglobin, and hematocrit within 2 months, epoetin alfa is needed. This colony-stimulating factor will encourage the production of red cells. The clients white blood cell count is normal, so avoiding infection is not the priority.

A nurse cares for a client who is infected with Burkholderia cepacia. Which action should the nurse take first when admitting this client to a pulmonary care unit?

a. Instruct the client to wash his or her hands after contact with other people. b. Implement Droplet Precautions and don a surgical mask. c. Keep the client isolated from other clients with cystic fibrosis. d. Obtain blood, sputum, and urine culture specimens. ANS: C Burkholderia cepacia infection is spread through casual contact between cystic fibrosis clients, thus the need for these clients to be separated from one another. Strict isolation measures will not be necessary. Although the client should wash his or her hands frequently, the most important measure that can be implemented on the unit is isolation of the client from other clients with cystic fibrosis. There is no need to implement Droplet Precautions or don a surgical mask when caring for this client. Obtaining blood, sputum, and urine culture specimens will not provide information necessary to care for a client with Burkholderia cepacia infection.

An emergency department nurse cares for a client who experienced a spinal cord injury 1 hour ago. Which prescribed medication should the nurse prepare to administer?

a. Intrathecal baclofen (Lioresal) *b. Methylprednisolone (Medrol)* c. Atropine sulfate d. Epinephrine (Adrenalin ANS: B Methylprednisolone (Medrol) should be given within 8 hours of the injury. Clients who receive this therapy usually show improvement in motor and sensory function. The other medications are inappropriate for this client.

A client has the diagnosis of valley fever accompanied by myalgias and arthralgias. What treatment should the nurse educate the client on?

a. Intravenous amphotericin B b.Long-term antiinflammatories c. No specific treatment d.Oral fluconazole (Diflucan) ANS: D Valley fever, or coccidioidomycosis, is a fungal infection. Many people do not need treatment and the disease resolves on its own. However, the presence of joint and muscle pain indicates a moderate infection that needs treatment with antifungal medications. IV amphotericin is reserved for pregnant women and those with severe infection. Anti-inflammatory medications may be used to treat muscle aches and pain but are not used long term.

A client is receiving rivaroxaban (Xarelto) and asks the nurse to explain how it works. What response by the nurse is best?

a. It inhibits thrombin. b. It inhibits fibrinogen. c. It thins your blood. d. It works against vitamin K. ANS: A Rivaroxaban is a direct thrombin inhibitor. It does not work on fibrinogen or vitamin K. It is not a blood thinner, although many clients call anticoagulants by this name.

A nursing student wants to know why clients with chronic obstructive pulmonary disease tend to be polycythemic. What response by the nurse instructor is best?

a. It is due to side effects of medications for bronchodilation. b. It is from overactive bone marrow in response to chronic disease. c. It combats the anemia caused by an increased metabolic rate. d. It compensates for tissue hypoxia caused by lung disease. ANS: D In response to hypoxia, more red blood cells are made so more oxygen can be carried and delivered to tissues. This is a physiologic process in response to the disease; it is not a medication side effect, the result of overactive bone marrow, or a response to anemia.

A student nurse asks what essential hypertension is. What response by the registered nurse is best?

a. It means it is caused by another disease. b. It means it is essential that it be treated. c. It is hypertension with no specific cause. d. It refers to severe and life-threatening hypertension. ANS: C Essential hypertension is the most common type of hypertension and has no specific cause such as an underlying disease process. Hypertension that is due to another disease process is called secondary hypertension. A severe, life-threatening form of hypertension is malignant hypertension.

A nurse is teaching the daughter of a client who has Alzheimers disease. The daughter asks, Will the medication my mother is taking improve her dementia? How should the nurse respond?

a. It will allow your mother to live independently for several more years. b. It is used to halt the advancement of Alzheimers disease but will not cure it. *c. It will not improve her dementia but can help control emotional responses*. d. It is used to improve short-term memory but will not improve problem solving. ANS: C Drug therapy is not effective for treating dementia or halting the advancement of Alzheimers disease. However, certain drugs may help suppress emotional disturbances and psychiatric manifestations. Medication therapy may not allow the client to safely live independently.

A client is scheduled for a colonoscopy and the nurse has provided instructions on the bowel cleansing regimen. What statement by the client indicates a need for further teaching?

a. Its a good thing I love orange and cherry gelatin. b. My spouse will be here to drive me home. c. I should refrigerate the GoLYTELY before use. d. I will buy a case of Gatorade before the prep. ANS: A The client should be advised to avoid beverages and gelatin that are red, orange, or purple in color as their residue can appear to be blood. The other statements show a good understanding of the preparation for the procedure.

The nurse knows that a client with prolonged prothrombin time (PT) values (not related to medication) probably has dysfunction in which organ?

a. Kidneys b. Liver c. Spleen d. Stomach ANS: B Severe acute or chronic liver damage leads to a prolonged PT secondary to impaired synthesis of clotting proteins. The other organs are not related to this issue.

A client had an upper gastrointestinal hemorrhage and now has a nasogastric (NG) tube. What comfort measure may the nurse delegate to the unlicensed assistive personnel (UAP)?

a. Lavaging the tube with ice water b. Performing frequent oral care c. Re-positioning the tube every 4 hours d. Taking and recording vital signs ANS: B Clients with NG tubes need frequent oral care both for comfort and to prevent infection. Lavaging the tube is done by the nurse. Re-positioning the tube, if needed, is also done by the nurse. The UAP can take vital signs, but this is not a comfort measure.

To promote comfort after a colonoscopy, in what position does the nurse place the client?

a. Left lateral b. Prone c. Right lateral d. Supine ANS: A After colonoscopy, clients have less discomfort and quicker passage of flatus when placed in the left lateral position.

A client has rheumatoid arthritis that especially affects the hands. The client wants to finish quilting a baby blanket before the birth of her grandchild. What response by the nurse is best?

a. Lets ask the provider about increasing your pain pills. b. Hold ice bags against your hands before quilting. *c. Try a paraffin wax dip 20 minutes before you quilt*. d. You need to stop quilting before it destroys your fingers. ANS: C Paraffin wax dips are beneficial for decreasing pain in arthritic hands and lead to increased mobility. The nurse can suggest this comfort measure. Increasing pain pills will not help with movement. Ice has limited use unless the client has a hot or exacerbated joint. The client wants to finish her project, so the nurse should not negate its importance by telling the client it is destroying her joints.

A nurse cares for a client with ulcerative colitis. The client states, I feel like I am tied to the toilet. This disease is controlling my life. How should the nurse respond?

a. Lets discuss potential factors that increase your symptoms. b. If you take the prescribed medications, you will no longer have diarrhea. c. To decrease distress, do not eat anything before you go out. d. You must retake control of your life. I will consult a therapist to help. ANS: A Clients with ulcerative colitis often express that the disorder is disruptive to their lives. Stress factors can increase symptoms. These factors should be identified so that the client will have more control over his or her condition. Prescription medications and anorexia will not eliminate exacerbations. Although a therapist may assist the client, this is not an appropriate response.

A nurse cares for a client who states, My husband is repulsed by my colostomy and refuses to be intimate with me. How should the nurse respond?

a. Lets talk to the ostomy nurse to help you and your husband work through this. b. You could try to wear longer lingerie that will better hide the ostomy appliance. c. You should empty the pouch first so it will be less noticeable for your husband. d. If you are not careful, you can hurt the stoma if you engage in sexual activity. ANS: A The nurse should collaborate with the ostomy nurse to help the client and her husband work through intimacy issues. The nurse should not minimize the clients concern about her husband with ways to hide the ostomy. The client will not hurt the stoma by engaging in sexual activity.

.A nurse teaches a client with type 1 diabetes mellitus. Which statement should the nurse include in this clients teaching to decrease the clients insulin needs?

a. Limit your fluid intake to 2 liters a day. b.Animal organ meat is high in insulin. c. Limit your carbohydrate intake to 80 grams a day. *d.Walk at a moderate pace for 1 mile daily*. ANS: D Moderate exercise such as walking helps regulate blood glucose levels on a daily basis and results in lowered insulin requirements for clients with type 1 diabetes mellitus. Restricting fluids and eating organ meats will not reduce insulin needs. People with diabetes need at least 130 grams of carbohydrates each day.

A nurse assesses a client who has aortic regurgitation. In which location in the illustration shown below should the nurse auscultate to best hear a cardiac murmur related to aortic regurgitation?

a. Location A b. Location B c. Location C d. Location D ANS: A The aortic valve is auscultated in the second intercostal space just to the right of the sternum.

A nurse plans care for a client with Crohns disease who has a heavily draining fistula. Which intervention should the nurse indicate as the priority action in this clients plan of care?

a. Low-fiber diet b. Skin protection c. Antibiotic administration d. Intravenous glucocorticoids ANS: B Protecting the clients skin is the priority action for a client who has a heavily draining fistula. Intestinal fluid enzymes are caustic and can cause skin breakdown or fungal infections if the skin is not protected. The plan of care for a client who has Crohns disease includes adequate nutrition focused on high-calorie, high-protein, high-vitamin, and low-fiber meals, antibiotic administration, and glucocorticoids.

.A nurse prepares to insert a peripheral venous catheter in an older adult client. Which action should the nurse take to protect the clients skin during this procedure?

a. Lower the extremity below the level of the heart. b.Apply warm compresses to the extremity. c. Tap the skin lightly and avoid slapping. d.Place a washcloth between the skin and tourniquet. ANS: D To protect the clients skin, the nurse should place a washcloth or the clients gown between the skin and tourniquet. The other interventions are methods to distend the vein but will not protect the clients skin.

A nurse teaches a client who is prescribed nicotine replacement therapy. Which statement should the nurse include in this clients teaching?

a. Make a list of reasons why smoking is a bad habit. b. Rise slowly when getting out of bed in the morning. c. Smoking while taking this medication will increase your risk of a stroke. d. Stopping this medication suddenly increases your risk for a heart attack. ANS: C Clients who smoke while using drugs for nicotine replacement therapy increase the risk of stroke and heart attack. Nurses should teach clients not to smoke while taking this drug. The other responses are inappropriate

Several nurses have just helped a morbidly obese client get out of bed. One nurse accesses the clients record because I just have to know how much she weighs! What action by the clients nurse is most appropriate?

a. Make an anonymous report to the charge nurse. b. State That is a violation of client confidentiality. c. Tell the nurse Dont look; Ill tell you her weight. d. Walk away and ignore the other nurses behavior. ANS: B Ethical practice requires the nurse to speak up and tell the other nurse that he or she is violating client confidentiality rules. The other responses do not address this concern.

A nurse cares for a client who has a heart rate averaging 56 beats/min with no adverse symptoms. Which activity modification should the nurse suggest to avoid further slowing of the heart rate?

a. Make certain that your bath water is warm. b. Avoid straining while having a bowel movement. c. Limit your intake of caffeinated drinks to one a day. d. Avoid strenuous exercise such as running. ANS: B Bearing down strenuously during a bowel movement is one type of Valsalva maneuver, which stimulates the vagus nerve and results in slowing of the heart rate. Such a response is not desirable in a person who has bradycardia. The other instructions are not appropriate for this condition.

A nurse prepares to defibrillate a client who is in ventricular fibrillation. Which priority intervention should the nurse perform prior to defibrillating this client?

a. Make sure the defibrillator is set to the synchronous mode. b. Administer 1 mg of intravenous epinephrine. c. Test the equipment by delivering a smaller shock at 100 joules. d. Ensure that everyone is clear of contact with the client and the bed. ANS: D To avoid injury, the rescuer commands that all personnel clear contact with the client or the bed and ensures their compliance before delivery of the shock. A precordial thump can be delivered when no defibrillator is available. Defibrillation is done in asynchronous mode. Equipment should not be tested before a client is defibrillated because this is an emergency procedure; equipment should be checked on a routine basis. Epinephrine should be administered after defibrillation.

A nurse reviews the medication list of a client with a 20-year history of diabetes mellitus. The client holds up the bottle of prescribed duloxetine (Cymbalta) and states, My cousin has depression and is taking this drug. Do you think Im depressed? How should the nurse respond?

a. Many people with long-term diabetes become depressed after a while. *b.Its for peripheral neuropathy. Do you have burning pain in your feet or hands*? c. This antidepressant also has anti-inflammatory properties for diabetic pain. d.No. Many medications can be used for several different disorders. ANS: B Damage along nerves causes peripheral neuropathy and leads to burning pain along the nerves. Many drugs, including duloxetine (Cymbalta), can be used to treat peripheral neuropathy. The nurse should assess the client for this condition and then should provide an explanation of why this drug is being used. This medication, although it is used for depression, is not being used for that reason in this case. Duloxetine does not have antiinflammatory properties. Telling the client that many medications are used for different disorders does not provide the client with enough information to be useful

A nurse is caring for a client who exhibits dehydration-induced confusion. Which intervention should the nurse implement first?

a. Measure intake and output every 4 hours. b. Apply oxygen by mask or nasal cannula. c. Increase the IV flow rate to 250 mL/hr. d. Place the client in a high-Fowlers position. ANS: B Dehydration most frequently leads to poor cerebral perfusion and cerebral hypoxia, causing confusion. Applying oxygen can reduce confusion, even if perfusion is still less than optimal. Increasing the IV flow rate would increase perfusion. However, depending on the degree of dehydration, rehydrating the client too rapidly with IV fluids can lead to cerebral edema. Measuring intake and output and placing the client in a high-Fowlers position will not address the clients problem.

A nurse is caring for a client who is scheduled to undergo a thoracentesis. Which intervention should the nurse complete prior to the procedure?

a. Measure oxygen saturation before and after a 12-minute walk. b. Verify that the client understands all possible complications. c. Explain the procedure in detail to the client and the family. d. Validate that informed consent has been given by the client. ANS: D A thoracentesis is an invasive procedure with many potentially serious complications. Verifying that the client understands complications and explaining the procedure to be performed will be done by the physician or nurse practitioner, not the nurse. Measurement of oxygen saturation before and after a 12-minute walk is not a procedure unique to a thoracentesis.

An emergency room nurse assesses a client after a motor vehicle crash and notes ecchymotic areas across the clients lower abdomen. Which action should the nurse take first?

a. Measure the clients abdominal girth. b. Assess for abdominal guarding or rigidity. c. Check the clients hemoglobin and hematocrit. d. Obtain the clients complete health history. ANS: B On noticing the ecchymotic areas, the nurse should check to see if abdominal guarding or rigidity is present, because this could indicate major organ injury. The nurse should then notify the provider. Measuring abdominal girth or obtaining a complete health history is not appropriate at this time. Laboratory test results can be checked after assessment for abdominal guarding or rigidity.

A nurse prepares to discharge a client with cardiac dysrhythmia who is prescribed home health care services. Which priority information should be communicated to the home health nurse upon discharge?

a. Medication reconciliation b. Immunization history c. Religious beliefs d. Nutrition preferences ANS: A The home health nurse needs to know current medications the client is taking to ensure assessment, evaluation, and further education related to these medications. The other information will not assist the nurse to develop a plan of care for the client.

A nurse cares for a client who has a Giardia infection. Which medication should the nurse anticipate being prescribed for this client?

a. Metronidazole (Flagyl) b. Ciprofloxacin (Cipro) c. Sulfasalazine (Azulfidine) d. Ceftriaxone (Rocephin) ANS: A Metronidazole is the drug of choice for a Giardia infection. Ciprofloxacin and ceftriaxone are antibiotics used for bacterial infections. Sulfasalazine is used for ulcerative colitis and Crohns disease.

A nurse assesses a client with tachycardia. Which clinical manifestation requires immediate intervention by the nurse?

a. Mid-sternal chest pain b. Increased urine output c. Mild orthostatic hypotension d. P wave touching the T wave ANS: A Chest pain, possibly angina, indicates that tachycardia may be increasing the clients myocardial workload and oxygen demand to such an extent that normal oxygen delivery cannot keep pace. This results in myocardial hypoxia and pain. Increased urinary output and mild orthostatic hypotension are not life-threatening conditions and therefore do not require immediate intervention. The P wave touching the T wave indicates significant tachycardia and should be assessed to determine the underlying rhythm and cause; this is an important assessment but is not as critical as chest pain, which indicates cardiac cell death.

A nurse teaches a client who experiences occasional premature atrial contractions (PACs) accompanied by palpitations that resolve spontaneously without treatment. Which statement should the nurse include in this clients teaching?

a. Minimize or abstain from caffeine. b. Lie on your side until the attack subsides. c. Use your oxygen when you experience PACs. d. Take amiodarone (Cordarone) daily to prevent PACs. ANS: A PACs usually have no hemodynamic consequences. For a client experiencing infrequent PACs, the nurse should explore possible lifestyle causes, such as excessive caffeine intake and stress. Lying on the side will not prevent or resolve PACs. Oxygen is not necessary. Although medications may be needed to control symptomatic dysrhythmias, for infrequent PACs, the client first should try lifestyle changes to control them.

A nurse is caring for a client who is experiencing moderate metabolic alkalosis. Which action should the nurse take?

a. Monitor daily hemoglobin and hematocrit values. b. Administer furosemide (Lasix) intravenously. c. Encourage the client to take deep breaths. d. Teach the client fall prevention measures. ANS: D The priority nursing care for a client who is experiencing moderate metabolic alkalosis is providing client safety. Clients with metabolic alkalosis have muscle weakness and are at risk for falling. The other nursing interventions are not appropriate for metabolic alkalosis.

A nurse cares for a client who has been diagnosed with the Huntington gene but has no symptoms. The client asks for options related to family planning. What is the nurses best response?

a. Most clients with the Huntington gene do not pass on Huntington disease to their children. b. I understand that they can diagnose this disease in embryos. Therefore, you could select a healthy embryo from your fertilized eggs for implantation to avoid passing on Huntington disease. c. The need for family planning is limited because one of the hallmarks of Huntington disease is infertility. *d. Tell me more specifically what information you need about family planning so that I can direct you to the right information or health care provider*. ANS: D The presence of the Huntington gene means that the trait will be passed on to all offspring of the affected person. Understanding options for contraception and conception (e.g., surrogacy options) and implications for children may require the expertise of a genetic counselor or a reproductive specialist. The other statements are not accurate.

The nurse working in the orthopedic clinic knows that a client with which factor has an absolute contraindication for having a total joint replacement?

a. Needs multiple dental fillings b. Over age 85 *c. Severe osteoporosis* d. Urinary tract infection ANS: C Osteoporosis is a contraindication to joint replacement because the bones have a high risk of shattering as the new prosthesis is implanted. The client who needs fillings should have them done prior to the surgery. Age greater than 85 is not an absolute contraindication. A urinary tract infection can be treated prior to surgery.

A client is taking warfarin (Coumadin) and asks the nurse if taking St. Johns wort is acceptable. What response by the nurse is best?

a. No, it may interfere with the warfarin. b. There isnt any information about that. c. Why would you want to take that? d. Yes, it is a good supplement for you. ANS: A Many foods and drugs interfere with warfarin, St. Johns wort being one of them. The nurse should advise the client against taking it. The other answers are not accurate.

13. A nurse is teaching a larger female client about alcohol intake and how it affects hypertension. The client asks if drinking two beers a night is an acceptable intake. What answer by the nurse is best?

a. No, women should only have one beer a day as a general rule. b. No, you should not drink any alcohol with hypertension. c. Yes, since you are larger, you can have more alcohol. d. Yes, two beers per day is an acceptable amount of alcohol. ANS: A Alcohol intake should be limited to two drinks a day for men and one drink a day for women. A drink is classified as one beer, 1.5 ounces of hard liquor, or 5 ounces of wine. Limited alcohol intake is acceptable with hypertension. The womans size does not matter.

A nurse is caring for a client who has chronic emphysema and is receiving oxygen therapy at 6 L/min via nasal cannula. The following clinical data are available: Arterial Blood Gases Vital Signs pH = 7.28 Pulse rate = 96 beats/min PaO2 = 85 mm Hg Blood pressure = 135/45 PaCO2 = 55 mm Hg Respiratory rate = 6 breaths/min HCO3 = 26 mEq/L O2 saturation = 88% Which action should the nurse take first?

a. Notify the Rapid Response Team and provide ventilation support. b. Change the nasal cannula to a mask and reassess in 10 minutes. c. Place the client in Fowlers position if he or she is able to tolerate it. d. Decrease the flow rate of oxygen to 2 to 4 L/min, and reassess. ANS: A The primary trigger for respiration in a client with chronic respiratory acidosis is a decreased arterial oxygen level (hypoxic drive). Oxygen therapy can inhibit respiratory efforts in this case, eventually causing respiratory arrest and death. The nurse could decrease the oxygen flow rate; eventually, this might improve the clients respiratory rate, but the priority action would be to call the Rapid Response Team whenever a client with chronic carbon dioxide retention has a respiratory rate less than 10 breaths/min. Changing the cannula to a mask does nothing to improve the clients hypoxic drive, nor would it address the clients most pressing need. Positioning will not help the client breathe at a normal rate or maintain client safety.

A home care nurse prepares to administer intravenous medication to a client. The nurse assesses the site and reviews the clients chart prior to administering the medication: Client: Thomas Jackson DOB: 5/3/1936 Gender: Male January 23 (Today): Right upper extremity PICC is intact, patent, and has a good blood return. Site clean and free from manifestations of infiltration, irritation, and infection. Sue Franks, RN January 20: Purulent drainage from sacral wound. Wound cleansed and dressing changed. Dr. Smith notified and updated on client status. New orders received for intravenous antibiotics. Sue Franks, RN January 13: Client alert and oriented. Sacral wound dressing changed. Sue Franks, RN January 6: Right upper extremity PICC inserted. No complications. Discharged with home health care. Dr. Smith Based on the information provided, which action should the nurse take?

a. Notify the health care provider. b.Administer the prescribed medication. c. Discontinue the PICC. d.Switch the medication to the oral route. ANS: B A PICC that is functioning well without inflammation or infection may remain in place for months or even years. Because the line shows no signs of complications, it is permissible to administer the IV antibiotic. There is no need to call the physician to have the IV route changed to an oral route.

A nurse cares for a client who is recovering from a hemorrhoidectomy. The client states, I need to have a bowel movement. Which action should the nurse take?

a. Obtain a bedside commode for the client to use. b. Stay with the client while providing privacy. c. Make sure the call light is in reach to signal completion. d. Gather supplies to collect a stool sample for the laboratory. ANS: B The first bowel movement after hemorrhoidectomy can be painful enough to induce syncope. The nurse should stay with the client. The nurse should instruct clients who are discharged the same day to have someone nearby when they have their first postoperative bowel movement. Making sure the call light is within reach is an important nursing action too, but it does not take priority over client safety. Obtaining a bedside commode and taking a stool sample are not needed in this situation.

A nurse cares for a client admitted from a nursing home after several recent falls. What prescription should the nurse complete first?

a. Obtain urine sample for culture and sensitivity. b. Administer intravenous antibiotics. c. Encourage protein intake and additional fluids. d. Consult physical therapy for gait training. ANS: A Although all interventions are or might be important, obtaining a urine sample for urinalysis takes priority. Often urinary tract infection (UTI) symptoms in older adults are atypical, and a UTI may present with new onset of confusion or falling. The urine sample should be obtained before starting antibiotics. Dietary requirements and gait training should be implemented after obtaining the urine sample.

A nurse teaches a client who is recovering from a spinal fusion. Which statement should the nurse include in this clients postoperative instructions?

a. Only lift items that are 10 pounds or less. *b. Wear your brace whenever you are out of bed*. c. You must remain in bed for 3 weeks after surgery. d. You are prescribed medications to prevent rejection. ANS: B Clients who undergo spinal fusion are fitted with a brace that they must wear throughout the healing process (usually 3 to 6 months) whenever they are out of bed. The client should not lift anything. The client does not need to remain in bed. Medications for rejection prevention are not necessary for this procedure.

A nurse assesses a client with mitral valve stenosis. What clinical manifestation should alert the nurse to the possibility that the clients stenosis has progressed?

a. Oxygen saturation of 92% b. Dyspnea on exertion c. Muted systolic murmur d. Upper extremity weakness ANS: B Dyspnea on exertion develops as the mitral valvular orifice narrows and pressure in the lungs increases. The other manifestations do not relate to the progression of mitral valve stenosis.

A client had a percutaneous transluminal coronary angioplasty for peripheral arterial disease. What assessment finding by the nurse indicates a priority outcome for this client has been met?

a. Pain rated as 2/10 after medication b. Distal pulse on affected extremity 2+/4+ c. Remains on bedrest as directed d. Verbalizes understanding of procedure ANS: B Assessing circulation distal to the puncture site is a critical nursing action. A pulse of 2+/4+ indicates good perfusion. Pain control, remaining on bedrest as directed after the procedure, and understanding are all important, but do not take priority over perfusion.

A nurse assesses a client who is recovering from an ileostomy placement. Which clinical manifestation should alert the nurse to urgently contact the health care provider?

a. Pale and bluish stoma b. Liquid stool c. Ostomy pouch intact d. Blood-smeared output ANS: A The nurse should assess the stoma for color and contact the health care provider if the stoma is pale, bluish, or dark. The nurse should expect the client to have an intact ostomy pouch with dark green liquid stool that may contain some blood.

A nurse assesses a client recovering from a cerebral angiography via the clients right femoral artery. Which assessment should the nurse complete?

a. Palpate bilateral lower extremity pulses. b. Obtain orthostatic blood pressure readings. c. Perform a funduscopic examination. d. Assess the gag reflex prior to eating. ANS: A Cerebral angiography is performed by threading a catheter through the femoral or brachial artery. The extremity is kept immobilized after the procedure. The nurse checks the extremity for adequate circulation by noting skin color and temperature, presence and quality of pulses distal to the injection site, and capillary refill. Clients usually are on bedrest; therefore, orthostatic blood pressure readings cannot be performed. The funduscopic examination would not be affected by cerebral angiography. The client is given analgesics but not conscious sedation; therefore, the clients gag reflex would not be compromised.

The nurse is caring for a client on the medical-surgical unit who suddenly becomes unresponsive and has no pulse. The cardiac monitor shows the rhythm below: After calling for assistance and a defibrillator, which action should the nurse take next?

a. Perform a pericardial thump. b. Initiate cardiopulmonary resuscitation (CPR). c. Start an 18-gauge intravenous line. d. Ask the clients family about code status. ANS: B The clients rhythm is ventricular fibrillation. This is a lethal rhythm that is best treated with immediate defibrillation. While the nurse is waiting for the defibrillator to arrive, the nurse should start CPR. A pericardial thump is not a treatment for ventricular fibrillation. If the client does not already have an IV, other members of the team can insert one after defibrillation. The clients code status should already be known by the nurse prior to this event.

A nurse assesses a client with multiple sclerosis after administering prescribed fingolimod (Gilenya). For which adverse effect should the nurse monitor?

a. Peripheral edema b. Black tarry stools *c. Bradycardia* d. Nausea and vomiting ANS: C Fingolimod (Gilenya) is an antineoplastic agent that can cause bradycardia, especially within the first 6 hours after administration. Peripheral edema, black and tarry stools, and nausea and vomiting are not adverse effects of fingolimod.

A nurse cares for a client who has kidney stones from secondary hyperoxaluria. Which medication should the nurse anticipate administering?

a. Phenazopyridine (Pyridium) b. Propantheline (Pro-Banthine) c. Tolterodine (Detrol LA) d. Allopurinol (Zyloprim) ANS: D Stones caused by secondary hyperoxaluria respond to allopurinol (Zyloprim). Phenazopyridine is given to clients with urinary tract infections. Propantheline is an anticholinergic. Tolterodine is an anticholinergic with smooth muscle relaxant properties.

A nurse reviews the medication list of a client recovering from a computed tomography (CT) scan with IV contrast to rule out small bowel obstruction. Which medication should alert the nurse to contact the provider and withhold the prescribed dose?

a. Pioglitazone (Actos) b.Glimepiride (Amaryl) c. Glipizide (Glucotrol) *d.Metformin (Glucophage*) ANS: D Glucophage should not be administered when the kidneys are attempting to excrete IV contrast from the body. This combination would place the client at high risk for kidney failure. The nurse should hold the metformin dose and contact the provider. The other medications are safe to administer after receiving IV contrast.

A nurse is teaching a client with chronic migraine headaches. Which statement related to complementary therapy should the nurse include in this clients teaching?

a. Place a warm compress on your forehead at the onset of the headache. b. Wear dark sunglasses when you are in brightly lit spaces. *c. Lie down in a darkened room when you experience a headache*. d. Set your alarm to ensure you do not sleep longer than 6 hours at one time. ANS: C At the onset of a migraine attack, the client may be able to alleviate pain by lying down and darkening the room. He or she may want both eyes covered and a cool cloth on the forehead. If the client falls asleep, he or she should remain undisturbed until awakening. The other options are not recognized therapies for migraines.

A nurse teaches an 80-year-old client with diminished touch sensation. Which statement should the nurse include in this clients teaching?

a. Place soft rugs in your bathroom to decrease pain in your feet. b. Bathe in warm water to increase your circulation. *c. Look at the placement of your feet when walking*. d. Walk barefoot to decrease pressure ulcers from your shoes. ANS: C Older clients with decreased sensation are at risk of injury from the inability to sense changes in terrain when walking. To compensate for this loss, the client is instructed to look at the placement of her or his feet when walking. Throw rugs can slip and increase fall risk. Bath water that is too warm places the client at risk for thermal injury. The client should wear sturdy shoes for ambulation.

A nurse assesses a client who is hospitalized with an exacerbation of Crohns disease. Which clinical manifestation should the nurse expect to find?

a. Positive Murphys sign with rebound tenderness to palpitation b. Dull, hypoactive bowel sounds in the lower abdominal quadrants c. High-pitched, rushing bowel sounds in the right lower quadrant d. Reports of abdominal cramping that is worse at night ANS: C The nurse expects high-pitched, rushing bowel sounds due to narrowing of the bowel lumen in Crohns disease. A positive Murphys sign is indicative of gallbladder disease, and rebound tenderness often indicates peritonitis. Dullness in the lower abdominal quadrants and hypoactive bowel sounds are not commonly found with Crohns disease. Nightly worsening of abdominal cramping is not consistent with Crohns disease.

A client at risk for developing hyperkalemia states, I love fruit and usually eat it every day, but now I cant because of my high potassium level. How should the nurse respond?

a. Potatoes and avocados can be substituted for fruit. b. If you cook the fruit, the amount of potassium will be lower. c. Berries, cherries, apples, and peaches are low in potassium. d. You are correct. Fruit is very high in potassium. ANS: C Not all fruit is potassium rich. Fruits that are relatively low in potassium and can be included in the diet include apples, apricots, berries, cherries, grapefruit, peaches, and pineapples. Fruits high in potassium include bananas, kiwi, cantaloupe, oranges, and dried fruit. Cooking fruit does not alter its potassium content.

A nurse cares for a client who has a serum potassium of 7.5 mEq/L and is exhibiting cardiovascular changes. Which prescription should the nurse implement first?

a. Prepare to administer sodium polystyrene sulfate (Kayexalate) 15 g by mouth. b. Provide a heart healthy, low-potassium diet. c. Prepare to administer dextrose 20% and 10 units of regular insulin IV push. d. Prepare the client for hemodialysis treatment. ANS: C A client with a high serum potassium level and cardiac changes should be treated immediately to reduce the extracellular potassium level. Potassium movement into the cells is enhanced by insulin by increasing the activity of sodium-potassium pumps. Insulin will decrease both serum potassium and glucose levels and therefore should be administered with dextrose to prevent hypoglycemia. Kayexalate may be ordered, but this therapy may take hours to reduce potassium levels. Dialysis may also be needed, but this treatment will take much longer to implement and is not the first prescription the nurse should implement. Decreasing potassium intake may help prevent hyperkalemia in the future but will not decrease the clients current potassium level.

A client with autoimmune idiopathic thrombocytopenic purpura (ITP) has had a splenectomy and returned to the surgical unit 2 hours ago. The nurse assesses the client and finds the abdominal dressing saturated with blood. What action is most important?

a. Preparing to administer a blood transfusion b. Reinforcing the dressing and documenting findings c. Removing the dressing and assessing the surgical site d. Taking a set of vital signs and notifying the surgeon ANS: D While some bloody drainage on a new surgical dressing is expected, a saturated dressing is not. This client is already at high risk of bleeding due to the ITP. The nurse should assess vital signs for shock and notify the surgeon immediately. The client may or may not need a transfusion. Reinforcing the dressing is an appropriate action, but the nurse needs to do more than document afterward. Removing the dressing increases the risk of infection; plus, it is not needed since the nurse knows where the bleeding is coming from.

A nurse assesses a client who has mitral valve regurgitation. For which cardiac dysrhythmia should the nurse assess?

a. Preventricular contractions b. Atrial fibrillation c. Symptomatic bradycardia d. Sinus tachycardia ANS: B Atrial fibrillation is a clinical manifestation of mitral valve regurgitation and stenosis. Preventricular contractions and bradycardia are not associated with valvular problems. These are usually identified in clients with electrolyte imbalances, myocardial infarction, and sinus node problems. Sinus tachycardia is a manifestation of aortic regurgitation due to a decrease in cardiac output.

A nurse delegates care to an unlicensed assistive personnel (UAP). Which statement should the nurse include when delegating hygiene for a client who has a vascular access device?

a. Provide a bed bath instead of letting the client take a shower. b.Use sterile technique when changing the dressing. c. Disconnect the intravenous fluid tubing prior to the clients bath. d.Use a plastic bag to cover the extremity with the device. ANS: D The nurse should ask the UAP to cover the extremity with the vascular access device with a plastic bag or wrap to keep the dressing and site dry. The client may take a shower with a vascular device. The nurse should disconnect IV fluid tubing prior to the bath and change the dressing using sterile technique if necessary. These options are not appropriate to delegate to the UAP.

A nurse plans care for an 83-year-old client who is experiencing age-related sensory perception changes. Which intervention should the nurse include in this clients plan of care?

a. Provide a call button that requires only minimal pressure to activate. b. Write the date on the clients white board to promote orientation. *c. Ensure that the path to the bathroom is free from equipment*. d. Encourage the client to season food to stimulate nutritional intake. ANS: C Dementia and confusion are not common phenomena in older adults. However, physical impairment related to illness can be expected. Providing opportunities for hazard-free ambulation will maintain strength and mobility (and ensure safety). Providing a call button, providing the date, and seasoning food do not address the clients impaired sensory perception.

After administering newly prescribed captopril (Capoten) to a client with heart failure, the nurse implements interventions to decrease complications. Which priority intervention should the nurse implement for this client?

a. Provide food to decrease nausea and aid in absorption. b. Instruct the client to ask for assistance when rising from bed. c. Collaborate with unlicensed assistive personnel to bathe the client. d. Monitor potassium levels and check for symptoms of hypokalemia. ANS: B Administration of the first dose of angiotensin-converting enzyme (ACE) inhibitors is often associated with hypotension, usually termed first-dose effect. The nurse should instruct the client to seek assistance before arising from bed to prevent injury from postural hypotension. ACE inhibitors do not need to be taken with food. Collaboration with unlicensed assistive personnel to provide hygiene is not a priority. The client should be encouraged to complete activities of daily living as independently as possible. The nurse should monitor for hyperkalemia, not hypokalemia, especially if the client has renal insufficiency secondary to heart failure.

A telemetry nurse assesses a client with third-degree heart block who has wide QRS complexes and a heart rate of 35 beats/min on the cardiac monitor. Which assessment should the nurse complete next?

a. Pulmonary auscultation b. Pulse strength and amplitude c. Level of consciousness d. Mobility and gait stability ANS: C A heart rate of 40 beats/min or less with widened QRS complexes could have hemodynamic consequences. The client is at risk for inadequate cerebral perfusion. The nurse should assess for level of consciousness, lightheadedness, confusion, syncope, and seizure activity. Although the other assessments should be completed, the clients level of consciousness is the priority.

After teaching a client who is recovering from a heart transplant to change positions slowly, the client asks, Why is this important? How should the nurse respond?

a. Rapid position changes can create shear and friction forces, which can tear out your internal vascular sutures. b. Your new vascular connections are more sensitive to position changes, leading to increased intravascular pressure and dizziness. c. Your new heart is not connected to the nervous system and is unable to respond to decreases in blood pressure caused by position changes. d. While your heart is recovering, blood flow is diverted away from the brain, increasing the risk for stroke when you stand up. ANS: C Because the new heart is denervated, the baroreceptor and other mechanisms that compensate for blood pressure drops caused by position changes do not function. This allows orthostatic hypotension to persist in the postoperative period. The other options are false statements and do not correctly address the clients question

A nurse is caring for a client who is receiving an epidural infusion for pain management. Which assessment finding requires immediate intervention from the nurse?

a. Redness at the catheter insertion site b.Report of headache and stiff neck c. Temperature of 100.1 F (37.8 C) d.Pain rating of 8 on a scale of 0 to 10 ANS: B Complications of epidural therapy include infection, bleeding, leakage of cerebrospinal fluid, occlusion of the catheter lumen, and catheter migration. Headache, neck stiffness, and a temperature higher than 101 F are signs of meningitis and should be reported to the provider immediately. The other findings are important but do not require immediate intervention.

A client is receiving plasmapheresis as treatment for Goodpastures syndrome. When planning care, the nurse places highest priority on interventions for which client problem?

a. Reduced physical activity related to the diseases effects on the lungs b. Inadequate family coping related to the clients hospitalization c. Inadequate knowledge related to the plasmapheresis process *d. Potential for infection related to the site for organism invasion* ANS: D Physical diagnoses take priority over psychosocial diagnoses, so inadequate family coping and inadequate knowledge are not the priority. The client has a potential for infection because plasmapheresis is an invasive procedure. Reduced activity is manifested by changes in vital signs, oxygenation, or electrocardiogram, and/or reports of chest pain or shortness of breath. There is no information in the question to indicate that the client is experiencing reduced physical activity

A nurse reviews the laboratory findings of a client with a urinary tract infection. The laboratory report notes a shift to the left in a clients white blood cell count. Which action should the nurse take?

a. Request that the laboratory perform a differential analysis on the white blood cells. b. Notify the provider and start an intravenous line for parenteral antibiotics. c. Collaborate with the unlicensed assistive personnel (UAP) to strain the clients urine for renal calculi. d. Assess the client for a potential allergic reaction and anaphylactic shock. ANS: B An increase in band cells creates a shift to the left. A left shift most commonly occurs with urosepsis and is seen rarely with uncomplicated urinary tract infections. The nurse will be administering antibiotics, most likely via IV, so he or she should notify the provider and prepare to give the antibiotics. The shift to the left is part of a differential white blood cell count. The nurse would not need to strain urine for stones. Allergic reactions are associated with elevated eosinophil cells, not band cells.

A nurse assesses a client who is prescribed furosemide (Lasix) for hypertension. For which acid-base imbalance should the nurse assess to prevent complications of this therapy?

a. Respiratory acidosis b. Respiratory alkalosis c. Metabolic acidosis d. Metabolic alkalosis ANS: D Many diuretics, especially loop diuretics, increase the excretion of hydrogen ions, leading to excess acid loss through the renal system. This situation is an acid deficit of metabolic origin.

A nurse cares for a client with arthritis who reports frequent asthma attacks. Which action should the nurse take first?

a. Review the clients pulmonary function test results. b. Ask about medications the client is currently taking. c. Assess how frequently the client uses a bronchodilator. d. Consult the provider and request arterial blood gases. ANS: B Aspirin and other nonsteroidal anti-inflammatory drugs (NSAIDs) can trigger asthma in some people. This results from increased production of leukotriene when aspirin or NSAIDs suppress other inflammatory pathways and is a high priority given the clients history. Reviewing pulmonary function test results will not address the immediate problem of frequent asthma attacks. This is a good intervention for reviewing response to bronchodilators. Questioning the client about the use of bronchodilators will address interventions for the attacks but not their cause. Reviewing arterial blood gas results would not be of use in a client between attacks because many clients are asymptomatic when not having attacks.

After teaching a client with diverticular disease, a nurse assesses the clients understanding. Which menu selection made by the client indicates the client correctly understood the teaching?

a. Roasted chicken with rice pilaf and a cup of coffee with cream b. Spaghetti with meat sauce, a fresh fruit cup, and hot tea c. Garden salad with a cup of bean soup and a glass of low-fat milk d. Baked fish with steamed carrots and a glass of apple juice ANS: D Clients who have diverticular disease are prescribed a low-residue diet. Whole grains (rice pilaf), uncooked fruits and vegetables (salad, fresh fruit cup), and high-fiber foods (cup of bean soup) should be avoided with a low-residue diet. Canned or cooked vegetables are appropriate. Apple juice does not contain fiber and is acceptable for a low-residue diet.

A nurse cares for a client who is prescribed magnetic resonance imaging (MRI) of the heart. The clients health history includes a previous myocardial infarction and pacemaker implantation. Which action should the nurse take?

a. Schedule an electrocardiogram just before the MRI. b. Notify the health care provider before scheduling the MRI. c. Call the physician and request a laboratory draw for cardiac enzymes. d. Instruct the client to increase fluid intake the day before the MRI. ANS: B The magnetic fields of the MRI can deactivate the pacemaker. The nurse should call the health care provider and report that the client has a pacemaker so the provider can order other diagnostic tests. The client does not need an electrocardiogram, cardiac enzymes, or increased fluids.

An emergency room nurse cares for a client who has been shot in the abdomen and is hemorrhaging heavily. Which action should the nurse take first?

a. Send a blood sample for a type and crossmatch. b. Insert a large intravenous line for fluid resuscitation. c. Obtain the heart rate and blood pressure. d. Assess and maintain a patent airway. ANS: D All of the options are important nursing actions in the care of a trauma client. However, airway always comes first. The client must have a patent airway, or other interventions will not be helpful.

A nurse cares for several clients on a neurologic unit. Which prescription for a client should direct the nurse to ensure that an informed consent has been obtained before the test or procedure?

a. Sensation measurement via the pinprick method b. Computed tomography of the cranial vault *c. Lumbar puncture for cerebrospinal fluid sampling* d. Venipuncture for autoantibody analysis ANS: C A lumbar puncture is an invasive procedure with many potentially serious complications. The other assessments or tests are considered noninvasive and do not require an informed consent.

A nurse reviews the laboratory results of a client who is receiving intravenous insulin. Which should alert the nurse to intervene immediately?

a. Serum chloride level of 98 mmol/L b.Serum calcium level of 8.8 mg/dL c. Serum sodium level of 132 mmol/L *d.Serum potassium level of 2.5 mmol/L* ANS: D Insulin activates the sodium-potassium ATPase pump, increasing the movement of potassium from the extracellular fluid into the intracellular fluid, resulting in hypokalemia. In hyperglycemia, hypokalemia can also result from excessive urine loss of potassium. The chloride level is normal. The calcium and sodium levels are slightly low, but this would not be related to hyperglycemia and insulin administration

15. A nurse assesses a client after administering prescribed levetiracetam (Keppra). Which laboratory tests should the nurse monitor for potential adverse effects of this medication?

a. Serum electrolyte levels *b. Kidney function tests* c. Complete blood cell count d. Antinuclear antibodies ANS: B Adverse effects of levetiracetam include coordination problems and renal toxicity. The other laboratory tests are not affected by levetiracetam.

.A nurse assesses a client who is being treated for hyperglycemic-hyperosmolar state (HHS). Which clinical manifestation indicates to the nurse that the therapy needs to be adjusted?

a. Serum potassium level has increased. b.Blood osmolarity has decreased. *c. Glasgow Coma Scale score is unchanged*. d.Urine remains negative for ketone bodies. ANS: C A slow but steady improvement in central nervous system functioning is the best indicator of therapy effectiveness for HHS. Lack of improvement in the level of consciousness may indicate inadequate rates of fluid replacement. The Glasgow Coma Scale assesses the clients state of consciousness against criteria of a scale including best eye, verbal, and motor responses. An increase in serum potassium, decreased blood osmolality, and urine negative for ketone bodies do not indicate adequacy of treatment.

A nurse reviews the chart of a client who has Crohns disease and a draining fistula. Which documentation should alert the nurse to urgently contact the provider for additional prescriptions?

a. Serum potassium of 2.6 mEq/L b. Client ate 20% of breakfast meal c. White blood cell count of 8200/mm3 d. Clients weight decreased by 3 pounds ANS: A Fistulas place the client with Crohns disease at risk for hypokalemia which can lead to serious dysrhythmias. This potassium level is low and should cause the nurse to intervene. The white blood cell count is normal. The other two findings are abnormal and also warrant intervention, but the potassium level takes priority.

.A nurse reviews laboratory results for a client with diabetes mellitus who presents with polyuria, lethargy, and a blood glucose of 560 mg/dL. Which laboratory result should the nurse correlate with the clients polyuria?

a. Serum sodium: 163 mEq/L b.Serum creatinine: 1.6 mg/dL c. Presence of urine ketone bodies *d.Serum osmolarity: 375 mOsm/kg* ANS: D Hyperglycemia causes hyperosmolarity of extracellular fluid. This leads to polyuria from an osmotic diuresis. The clients serum osmolarity is high. The clients sodium would be expected to be high owing to dehydration. Serum creatinine and urine ketone bodies are not related to the polyuria.

A nurse assesses a male client who is recovering from a urologic procedure. Which assessment finding indicates an obstruction of urine flow?

a. Severe pain b. Overflow incontinence c. Hypotension d. Blood-tinged urine ANS: B The most common manifestation of urethral stricture after a urologic procedure is obstruction of urine flow. This rarely causes pain and has no impact on blood pressure. The client may experience overflow incontinence with the involuntary loss of urine when the bladder is distended. Blood in the urine is not a manifestation of the obstruction of urine flow.

A nurse assesses a client who has appendicitis. Which clinical manifestation should the nurse expect to find?

a. Severe, steady right lower quadrant pain b. Abdominal pain associated with nausea and vomiting c. Marked peristalsis and hyperactive bowel sounds d. Abdominal pain that increases with knee flexion ANS: A Right lower quadrant pain, specifically at McBurneys point, is characteristic of appendicitis. Usually if nausea and vomiting begin first, the client has gastroenteritis. Marked peristalsis and hyperactive bowel sounds are not indicative of appendicitis. Abdominal pain due to appendicitis decreases with knee flexion.

A nurse assesses a client with Huntington disease. Which motor changes should the nurse monitor for in this client?

a. Shuffling gait *b. Jerky hand movements* c. Continuous chewing motions d. Tremors of the hands ANS: B An imbalance between excitatory and inhibitory neurotransmitters leads to uninhibited motor movements, such as brisk, jerky, purposeless movements of the hands, face, tongue, and legs. Shuffling gait, continuous chewing motions, and tremors are associated with Parkinson disease.

A nurse cares for a female client who has a family history of cystic fibrosis. The client asks, Will my children have cystic fibrosis? How should the nurse respond

a. Since many of your family members are carriers, your children will also be carriers of the gene. b. Cystic fibrosis is an autosomal recessive disorder. If you are a carrier, your children will have the disorder. c. Since you have a family history of cystic fibrosis, I would encourage you and your partner to be tested. d. Cystic fibrosis is caused by a protein that controls the movement of chloride. Adjusting your diet will decrease the spread of this disorder. ANS: C Cystic fibrosis is an autosomal recessive disorder in which both gene alleles must be mutated for the disorder to be expressed. The nurse should encourage both the client and partner to be tested for the abnormal gene. The other statements are not true.

A nurse assesses a client with atrial fibrillation. Which manifestation should alert the nurse to the possibility of a serious complication from this condition?

a. Sinus tachycardia b. Speech alterations c. Fatigue d. Dyspnea with activity ANS: B Clients with atrial fibrillation are at risk for embolic stroke. Evidence of embolic events includes changes in mentation, speech, sensory function, and motor function. Clients with atrial fibrillation often have a rapid ventricular response as a result. Fatigue is a nonspecific complaint. Clients with atrial fibrillation often have dyspnea as a result of the decreased cardiac output caused by the rhythm disturbance.

A nurse assesses a client who is recovering from a diskectomy 6 hours ago. Which assessment finding should the nurse address first?

a. Sleepy but arouses to voice b. Dry and cracked oral mucosa c. Pain present in lower back *d. Bladder palpated above pubis* ANS: D A distended bladder may indicate damage to the sacral spinal nerves. The other findings require the nurse to provide care but are not the priority or a complication of the procedure.

After teaching a client who is prescribed a restricted sodium diet, a nurse assesses the clients understanding. Which food choice for lunch indicates the client correctly understood the teaching?

a. Slices of smoked ham with potato salad b. Bowl of tomato soup with a grilled cheese sandwich c. Salami and cheese on whole wheat crackers d. Grilled chicken breast with glazed carrots ANS: D Clients on restricted sodium diets generally should avoid processed, smoked, and pickled foods and those with sauces and other condiments. Foods lowest in sodium include fish, poultry, and fresh produce. The ham, tomato soup, salami, and crackers are often high in sodium.

A client with peptic ulcer disease asks the nurse about taking slippery elm supplements. What response by the nurse is best?

a. Slippery elm has no benefit for this problem. b. Slippery elm is often used for this disorder. c. There is no evidence that this will work. d. You should not take any herbal remedies. ANS: B There are several complementary and alternative medicine regimens that are used for gastritis and peptic ulcer disease. Most have been tested on animals but not humans. Slippery elm is a common supplement used for this disorder.

A nurse cares for a client with a spinal cord injury. With which interdisciplinary team member should the nurse consult to assist the client with activities of daily living?

a. Social worker b. Physical therapist *c. Occupational therapist* d. Case manager ANS: C The occupational therapist instructs the client in the correct use of all adaptive equipment. In collaboration with the therapist, the nurse instructs family members or the caregiver about transfer skills, feeding, bathing, dressing, positioning, and skin care. The other team members are consulted to assist the client with unrelated issues.

A nurse evaluates prescriptions for a client with chronic atrial fibrillation. Which medication should the nurse expect to find on this clients medication administration record to prevent a common complication of this condition?

a. Sotalol (Betapace) b. Warfarin (Coumadin) c. Atropine (Sal-Tropine) d. Lidocaine (Xylocaine) ANS: B Atrial fibrillation puts clients at risk for developing emboli. Clients at risk for emboli are treated with anticoagulants, such as heparin, enoxaparin, or warfarin. Sotalol, atropine, and lidocaine are not appropriate for this complication.

A nurse cares for a client who has developed esophagitis after undergoing radiation therapy for lung cancer. Which diet selection should the nurse provide for this client?

a. Spaghetti with meat sauce, ice cream b. Chicken soup, grilled cheese sandwich c. Omelet, soft whole wheat bread d. Pasta salad, custard, orange juice ANS: C Side effects of radiation therapy may include inflammation of the esophagus. Clients should be taught that bland, soft, high-calorie foods are best, along with liquid nutritional supplements. Tomato sauce may prove too spicy for a client with esophagitis. A grilled cheese sandwich is too difficult to swallow with this condition, and orange juice and other foods with citric acid are too caustic.

A nurse cares for a client with infective endocarditis. Which infection control precautions should the nurse use?

a. Standard Precautions b. Bleeding precautions " c. Reverse isolation d. Contact isolation ANS: A The client with infective endocarditis does not pose any specific threat of transmitting the causative organism. Standard Precautions should be used. Bleeding precautions or reverse or contact isolation is not necessary.

A nurse witnesses a client begin to experience a tonic-clonic seizure and loss of consciousness. Which action should the nurse take?

a. Start fluids via a large-bore catheter. *b. Turn the clients head to the side*. c. Administer IV push diazepam. d. Prepare to intubate the client. ANS: B The nurse should turn the clients head to the side to prevent aspiration and allow drainage of secretions. Anticonvulsants are administered on a routine basis if a seizure is sustained. If the seizure is sustained (status epilepticus), the client must be intubated and should be administered oxygen, 0.9% sodium chloride, and IV push lorazepam or diazepam.

After assessing a client who is receiving an amiodarone intravenous infusion for unstable ventricular tachycardia, the nurse documents the findings and compares these with the previous assessment findings: Vital Signs Nursing Assessment Time: 0800 Temperature: 98 F Heart rate: 68 beats/min Blood pressure: 135/60 mm Hg Respiratory rate: 14 breaths/min Oxygen saturation: 96% Oxygen therapy: 2 L nasal cannula Time: 1000 Temperature: 98.2 F Heart rate: 50 beats/min Blood pressure: 132/57 mm Hg Respiratory rate: 16 breaths/min Oxygen saturation: 95% Oxygen therapy: 2 L nasal cannula Time: 0800 Client alert and oriented. Cardiac rhythm: normal sinus rhythm. Skin: warm, dry, and appropriate for race. Respirations equal and unlabored. Client denies shortness of breath and chest pain. Time: 1000 Client alert and oriented. Cardiac rhythm: sinus bradycardia. Skin: warm, dry, and appropriate for race. Respirations equal and unlabored. Client denies shortness of breath and chest pain. Client voids 420 mL of clear yellow urine. Based on the assessments, which action should the nurse take?

a. Stop the infusion and flush the IV. b. Slow the amiodarone infusion rate. c. Administer IV normal saline. d. Ask the client to cough and deep breathe. ANS: B IV administration of amiodarone may cause bradycardia and atrioventricular (AV) block. The correct action for the nurse to take at this time is to slow the infusion, because the client is asymptomatic and no evidence reveals AV block that might require pacing. Abruptly ceasing the medication could allow fatal dysrhythmias to occur. The administration of IV fluids and encouragement of coughing and deep breathing exercises are not indicated, and will not increase the clients heart rate.

A nurse cares for a client who has a pleural chest tube. Which action should the nurse take to ensure safe use of this equipment?

a. Strip the tubing to minimize clot formation and ensure patency. b. Secure tubing junctions with clamps to prevent accidental disconnections. c. Connect the chest tube to wall suction at the level prescribed by the provider. d. Keep padded clamps at the bedside for use if the drainage system is interrupted. ANS: D Padded clamps should be kept at the bedside for use if the drainage system becomes dislodged or is interrupted. The nurse should never strip the tubing. Tubing junctions should be taped, not clamped. Wall suction should be set at the level indicated by the devices manufacturer, not the provider.

A nurse teaches a client with a lower motor neuron lesion who wants to achieve bladder control. Which statement should the nurse include in this clients teaching?

a. Stroke the inner aspect of your thigh to initiate voiding. b. Use a clean technique for intermittent catheterization. c. Implement digital anal stimulation when your bladder is full. *d. Tighten your abdominal muscles to stimulate urine flow* ANS: D In clients with lower motor neuron problems such as spinal cord injury, performing a Valsalva maneuver or tightening the abdominal muscles are interventions that can initiate voiding. Stroking the inner aspect of the thigh may initiate voiding in a client who has an upper motor neuron problem. Intermittent catheterization and digital anal stimulation do not initiate voiding or bladder control.

A nurse plans care for a client with overflow incontinence. Which intervention should the nurse include in this clients plan of care to assist with elimination?

a. Stroke the medial aspect of the thigh. b. Use intermittent catheterization. c. Provide digital anal stimulation. d. Use the Valsalva maneuver. ANS: D In clients with overflow incontinence, the voiding reflex arc is not intact. Mechanical pressure, such as that achieved through the Valsalva maneuver (holding the breath and bearing down as if to defecate), can initiate voiding. Stroking the medial aspect of the thigh or providing digital anal stimulation requires the reflex arc to be intact to initiate elimination. Due to the high risk for infection, intermittent catheterization should only be implemented when other interventions are not successful.

The nurse asks a client who has experienced ventricular dysrhythmias about substance abuse. The client asks, Why do you want to know if I use cocaine? How should the nurse respond?

a. Substance abuse puts clients at risk for many health issues. b. The hospital requires that I ask you about cocaine use. c. Clients who use cocaine are at risk for fatal dysrhythmias. d. We can provide services for cessation of substance abuse. ANS: C Clients who use cocaine or illicit inhalants are particularly at risk for potentially fatal dysrhythmias. The other responses do not adequately address the clients question.

A nurse cares for a teenage girl with a new ileostomy. The client states, I cannot go to prom with an ostomy. How should the nurse respond?

a. Sure you can. Purchase a prom dress one size larger to hide the ostomy appliance. b. The pouch wont be as noticeable if you avoid broccoli and carbonated drinks prior to the prom. c. Lets talk to the enterostomal therapist about options for ostomy supplies and dress styles. d. You can remove the pouch from your ostomy appliance when you are at the prom so that it is less noticeable. ANS: C The ostomy nurse is a valuable resource for clients, providing suggestions for supplies and methods to manage the ostomy. A larger dress size will not necessarily help hide the ostomy appliance. Avoiding broccoli and carbonated drinks does not offer reassurance for the client. Ileostomies have an almost constant liquid effluent, so pouch removal during the prom is not feasible.

A nurse is teaching a client who has cystic fibrosis (CF). Which statement should the nurse include in this clients teaching?

a. Take an antibiotic each day. b. Contact your provider to obtain genetic screening. c. Eat a well-balanced, nutritious diet. d. Plan to exercise for 30 minutes every day. ANS: C Clients with CF often are malnourished due to vitamin deficiency and pancreatic malfunction. Maintaining nutrition is essential. Daily antibiotics and daily exercise are not essential actions. Genetic screening would not help the client manage CF better.

A nurse is teaching a client who experiences migraine headaches and is prescribed a beta blocker. Which statement should the nurse include in this clients teaching?

a. Take this drug only when you have prodromal symptoms indicating the onset of a migraine headache. *b. Take this drug as ordered, even when feeling well, to prevent vascular changes associated with migraine headaches*. c. This drug will relieve the pain during the aura phase soon after a headache has started. d. This medication will have no effect on your heart rate or blood pressure because you are taking it for migraines. ANS: B Beta blockers are prescribed as prophylactic treatment to prevent the vascular changes that initiate migraine headaches. Heart rate and blood pressure will also be affected, and the client should monitor these side effects. The other responses do not discuss appropriate uses of the medication

A nurse is teaching a client with multiple sclerosis who is prescribed cyclophosphamide (Cytoxan) and methylprednisolone (Medrol). Which statement should the nurse include in this clients discharge teaching?

a. Take warm baths to promote muscle relaxation. *b. Avoid crowds and people with colds*. c. Relying on a walker will weaken your gait. d. Take prescribed medications when symptoms occur. ANS: B The client should be taught to avoid people with any type of upper respiratory illness because these medications are immunosuppressive. Warm baths will exacerbate the clients symptoms. Assistive devices may be required for safe ambulation. Medication should be taken at all times and should not be stopped

A nurse wants to provide community service that helps meet the goals of Healthy People 2020 (HP2020) related to cardiovascular disease and stroke. What activity would best meet this goal?

a. Teach high school students heart-healthy living. b. Participate in blood pressure screenings at the mall. c. Provide pamphlets on heart disease at the grocery store. d. Set up an Ask the nurse booth at the pet store. ANS: B An important goal of HP2020 is to increase the proportion of adults who have had their blood pressure measured within the preceding 2 years and can state whether their blood pressure was normal or high. Participating in blood pressure screening in a public spot will best help meet that goal. The other options are all appropriate but do not specifically help meet a goal.

A nurse obtains the health history of a client who is recently diagnosed with lung cancer and identifies that the client has a 60pack-year smoking history. Which action is most important for the nurse to take when interviewing this client?

a. Tell the client that he needs to quit smoking to stop further cancer development. b. Encourage the client to be completely honest about both tobacco and marijuana use. c. Maintain a nonjudgmental attitude to avoid causing the client to feel guilty. d. Avoid giving the client false hope regarding cancer treatment and prognosis. ANS: C Smoking history includes the use of cigarettes, cigars, pipe tobacco, marijuana, and other controlled substances. Because the client may have guilt or denial about this habit, assume a nonjudgmental attitude during the interview. This will encourage the client to be honest about the exposure. Ask the client whether any of these substances are used now or were used in the past. Assess whether the client has passive exposure to smoke in the home or workplace. If the client smokes, ask for how long, how many packs per day, and whether he or she has quit smoking (and how long ago). Document the smoking history in pack-years (number of packs smoked daily multiplied by the number of years the client has smoked). Quitting smoking may not stop further cancer development. This statement would be giving the client false hope, which should be avoided, but is not as important as maintaining a nonjudgmental attitude.

A nurse teaches a client who is prescribed an insulin pump. Which statement should the nurse include in this clients discharge education?

a. Test your urine daily for ketones. b.Use only buffered insulin in your pump. c. Store the insulin in the freezer until you need it. *d.Change the needle every 3 days*. ANS: D Having the same needle remain in place through the skin for longer than 3 days drastically increases the risk for infection in or through the delivery system. Having an insulin pump does not require the client to test for ketones in the urine. Insulin should not be frozen. Insulin is not buffered

A nurse teaches a client with diabetes mellitus and a body mass index of 42 who is at high risk for coronary artery disease. Which statement related to nutrition should the nurse include in this clients teaching?

a. The best way to lose weight is a high-protein, low-carbohydrate diet. b. You should balance weight loss with consuming necessary nutrients. c. A nutritionist will provide you with information about your new diet. d. If you exercise more frequently, you wont need to change your diet. ANS: B Clients at risk for cardiovascular diseases should follow the American Heart Association guidelines to combat obesity and improve cardiac health. The nurse should encourage the client to eat vegetables, fruits, unrefined whole-grain products, and fat-free dairy products while losing weight. High-protein food items are often high in fat and calories. Although the nutritionist can assist with client education, the nurse should include nutrition education and assist the client to make healthy decisions. Exercising and eating nutrient-rich foods are both important components in reducing cardiovascular risk.

A nurse assesses a client who had an intraosseous catheter placed in the left leg. Which assessment finding is of greatest concern?

a. The catheter has been in place for 20 hours. b.The client has poor vascular access in the upper extremities. c. The catheter is placed in the proximal tibia. d.The clients left lower extremity is cool to the touch. ANS: D Compartment syndrome is a condition in which increased tissue perfusion in a confined anatomic space causes decreased blood flow to the area. A cool extremity can signal the possibility of this syndrome. All other findings are important; however, the possible development of compartment syndrome requires immediate intervention because the client could require amputation of the limb if the nurse does not correctly assess this perfusion problem.

A client with a new tracheostomy is being seen in the oncology clinic. What finding by the nurse best indicates that goals for the nursing diagnosis Impaired Self-Esteem are being met?

a. The client demonstrates good understanding of stoma care. b. The client has joined a book club that meets at the library. c. Family members take turns assisting with stoma care. d. Skin around the stoma is intact without signs of infection. ANS: B The client joining a book club that meets outside the home and requires him or her to go out in public is the best sign that goals for Impaired Self-Esteem are being met. The other findings are all positive signs but do not relate to this nursing diagnosis.

A nurse assesses a clients electrocardiograph tracing and observes that not all QRS complexes are preceded by a P wave. How should the nurse interpret this observation?

a. The client has hyperkalemia causing irregular QRS complexes. b. Ventricular tachycardia is overriding the normal atrial rhythm. c. The clients chest leads are not making sufficient contact with the skin. d. Ventricular and atrial depolarizations are initiated from different sites. ANS: D Normal rhythm shows one P wave preceding each QRS complex, indicating that all depolarization is initiated at the sinoatrial node. QRS complexes without a P wave indicate a different source of initiation of depolarization. This finding on an electrocardiograph tracing is not an indication of hyperkalemia, ventricular tachycardia, or disconnection of leads.

After teaching a client how to perform diaphragmatic breathing, the nurse assesses the clients understanding. Which action demonstrates that the client correctly understands the teaching?

a. The client lays on his or her side with his or her knees bent. b. The client places his or her hands on his or her abdomen. c. The client lays in a prone position with his or her legs straight. d. The client places his or her hands above his or her head. ANS: B To perform diaphragmatic breathing correctly, the client should place his or her hands on his or her abdomen to create resistance. This type of breathing cannot be performed effectively while lying on the side or with hands over the head. This type of breathing would not be as effective lying prone.

A nurse assesses a client after a thoracentesis. Which assessment finding warrants immediate action?

a. The client rates pain as a 5/10 at the site of the procedure. b. A small amount of drainage from the site is noted. c. Pulse oximetry is 93% on 2 liters of oxygen. d. The trachea is deviated toward the opposite side of the neck. ANS: D A deviated trachea is a manifestation of a tension pneumothorax, which is a medical emergency. The other findings are normal or near normal.

A nurse assesses a client with bladder cancer who is recovering from a complete cystectomy with ileal conduit. Which assessment finding should alert the nurse to urgently contact the health care provider?

a. The ileostomy is draining blood-tinged urine. b. There is serous sanguineous drainage present on the surgical dressing. c. The ileostomy stoma is pale and cyanotic in appearance. d. Oxygen saturations are 92% on room air. ANS: C A pale or cyanotic stoma indicates impaired circulation to the stoma and must be treated to prevent necrosis. Blood-tinged urine and serous sanguineous drainage are expected after this type of surgery. Oxygen saturation of 92% on room air is at the low limit of normal.

A nurse is assessing clients who have intravenous therapy prescribed. Which assessment finding for a client with a peripherally inserted central catheter (PICC) requires immediate attention?

a. The initial site dressing is 3 days old. b.The PICC was inserted 4 weeks ago. c. A securement device is absent. d.Upper extremity swelling is noted. ANS: D Upper extremity swelling could indicate infiltration, and the PICC will need to be removed. The initial dressing over the PICC site should be changed within 24 hours. This does not require immediate attention, but the swelling does. The dwell time for PICCs can be months or even years. Securement devices are being used more often now to secure the catheter in place and prevent complications such as phlebitis and infiltration. The IV should have one, but this does not take priority over the client whose arm is swollen.

A nurse cares for a client recovering from prosthetic valve replacement surgery. The client asks, Why will I need to take anticoagulants for the rest of my life? How should the nurse respond?

a. The prosthetic valve places you at greater risk for a heart attack. b. Blood clots form more easily in artificial replacement valves. c. The vein taken from your leg reduces circulation in the leg. d. The surgery left a lot of small clots in your heart and lungs. ANS: B Synthetic valve prostheses and scar tissue provide surfaces on which platelets can aggregate easily and initiate the formation of blood clots. The other responses are inaccurate.

A nurse cares for a client who had a colostomy placed in the ascending colon 2 weeks ago. The client states, The stool in my pouch is still liquid. How should the nurse respond?

a. The stool will always be liquid with this type of colostomy. b. Eating additional fiber will bulk up your stool and decrease diarrhea. c. Your stool will become firmer over the next couple of weeks. d. This is abnormal. I will contact your health care provider. ANS: A The stool from an ascending colostomy can be expected to remain liquid because little large bowel is available to reabsorb the liquid from the stool. This finding is not abnormal. Liquid stool from an ascending colostomy will not become firmer with the addition of fiber to the clients diet or with the passage of time.

A nurse cares for a client with chronic obstructive pulmonary disease (COPD). The client states that he no longer enjoys going out with his friends. How should the nurse respond?

a. There are a variety of support groups for people who have COPD. b. I will ask your provider to prescribe you with an antianxiety agent. c. Share any thoughts and feelings that cause you to limit social activities. d. Friends can be a good support system for clients with chronic disorders. ANS: C Many clients with moderate to severe COPD become socially isolated because they are embarrassed by frequent coughing and mucus production. They also can experience fatigue, which limits their activities. The nurse needs to encourage the client to verbalize thoughts and feelings so that appropriate interventions can be selected. Joining a support group would not decrease feelings of social isolation if the client does not verbalize feelings. Antianxiety agents will not help the client with social isolation. Encouraging a client to participate in activities without verbalizing concerns also would not be an effective strategy for decreasing social isolation.

A nurse cares for a middle-aged male client who has irritable bowel syndrome (IBS). The client states, I have changed my diet and take bulk-forming laxatives, but my symptoms have not gotten better. I heard about a drug called Amitiza. Do you think it might help? How should the nurse respond?

a. This drug is still in the research phase and is not available for public use yet. b. Unfortunately, lubiprostone is approved only for use in women. c. Lubiprostone works well. I will recommend this prescription to your provider. d. This drug should not be used with bulk-forming laxatives. ANS: B Lubiprostone (Amitiza) is a new drug for IBS with constipation that works by simulating receptors in the intestines to increase fluid and promote bowel transit time. Lubiprostone is currently approved only for use in women. Trials with increased numbers of male participants are needed prior to Food and Drug Administration approval for men.

A nurse prepares a client for coronary artery bypass graft surgery. The client states, I am afraid I might die. How should the nurse respond?

a. This is a routine surgery and the risk of death is very low. b. Would you like to speak with a chaplain prior to surgery? c. Tell me more about your concerns about the surgery. d. What support systems do you have to assist you? ANS: C The nurse should discuss the clients feelings and concerns related to the surgery. The nurse should not provide false hope or push the clients concerns off on the chaplain. The nurse should address support systems after addressing the clients current issue.

A nurse cares for a client with advanced Alzheimers disease. The clients caregiver states, She is always wandering off. What can I do to manage this restless behavior? How should the nurse respond?

a. This is a sign of fatigue. The client would benefit from a daily nap. *b. Engage the client in scheduled activities throughout the day*. c. It sounds like this is difficult for you. I will consult the social worker. d. The provider can prescribe a mild sedative for restlessness. ANS: B Several strategies may be used to cope with restlessness and wandering. One strategy is to engage the client in structured activities. Another is to take the client for frequent walks. Daily naps and a mild sedative will not be as effective in the management of restless behavior. Consulting the social worker does not address the caregivers concern

A nurse provides phone triage to a pregnant client. The client states, I am experiencing a burning pain when I urinate. How should the nurse respond?

a. This means labor will start soon. Prepare to go to the hospital. b. You probably have a urinary tract infection. Drink more cranberry juice. c. Make an appointment with your provider to have your infection treated. d. Your pelvic wall is weakening. Pelvic muscle exercises should help. ANS: C Pregnant clients with a urinary tract infection require prompt and aggressive treatment because cystitis can lead to acute pyelonephritis during pregnancy. The nurse should encourage the client to make an appointment and have the infection treated. Burning pain when urinating does not indicate the start of labor or weakening of pelvic muscles.

A nurse is caring for a client who has sleep apnea and is prescribed modafinil (Provigil). The client asks, How will this medication help me? How should the nurse respond?

a. This medication will treat your sleep apnea. b. This sedative will help you to sleep at night. c. This medication will promote daytime wakefulness. d. This analgesic will increase comfort while you sleep. ANS: C Modafinil is helpful for clients who have narcolepsy (uncontrollable daytime sleep) related to sleep apnea. This medication promotes daytime wakefulness.

A nurse teaches a client who has open vocal cord paralysis. Which technique should the nurse teach the client to prevent aspiration?

a. Tilt the head back as far as possible when swallowing. b. Tuck the chin down when swallowing. c. Breathe slowly and deeply while swallowing. d. Keep the head very still and straight while swallowing. ANS: B The client with open vocal cord paralysis may aspirate. The nurse should teach the client to tuck in his or her chin during swallowing to prevent aspiration. Tilting the head back would increase the chance of aspiration. Breathing slowly would not decrease the risk of aspiration, but holding the breath would. Keeping the head still and straight would not decrease the risk for aspiration.

After teaching a client who is diagnosed with new-onset status epilepticus and prescribed phenytoin (Dilantin), the nurse assesses the clients understanding. Which statement by the client indicates a correct understanding of the teaching?

a. To prevent complications, I will drink at least 2 liters of water daily. b. This medication will stop me from getting an aura before a seizure. c. I will not drive a motor vehicle while taking this medication. *d. Even when my seizures stop, I will continue to take this drug*. ANS: D Discontinuing antiepileptic drugs can lead to the recurrence of seizures or status epilepticus. The client does not need to drink more water and can drive while taking this medication. The medication will not stop an aura before a seizure.

After teaching a client to increase dietary potassium intake, a nurse assesses the clients understanding. Which dietary meal selection indicates the client correctly understands the teaching?

a. Toasted English muffin with butter and blueberry jam, and tea with sugar b. Two scrambled eggs, a slice of white toast, and a half cup of strawberries c. Sausage, one slice of whole wheat toast, half cup of raisins, and a glass of milk d. Bowl of oatmeal with brown sugar, a half cup of sliced peaches, and coffee ANS: C Meat, dairy products, and dried fruit have high concentrations of potassium. Eggs, breads, cereals, sugar, and some fruits (berries, peaches) are low in potassium. The menu selection of sausage, toast, raisins, and milk has the greatest number of items with higher potassium content.

After delegating care to an unlicensed assistive personnel (UAP) for a client who is prescribed habit training to manage incontinence, a nurse evaluates the UAPs understanding. Which action indicates the UAP needs additional teaching?

a. Toileting the client after breakfast b. Changing the clients incontinence brief when wet c. Encouraging the client to drink fluids d. Recording the clients incontinence episodes ANS: B Habit training is undermined by the use of absorbent incontinence briefs or pads. The nurse should re-educate the UAP on the technique of habit training. The UAP should continue to toilet the client after meals, encourage the client to drink fluids, and record incontinent episodes.

A nurse assesses a client who is recovering after a left-sided cardiac catheterization. Which assessment finding requires immediate intervention?

a. Urinary output less than intake b. Bruising at the insertion site c. Slurred speech and confusion d. Discomfort in the left leg ANS: C A left-sided cardiac catheterization specifically increases the risk for a cerebral vascular accident. A change in neurologic status needs to be acted on immediately. Discomfort and bruising are expected at the site. If intake decreases, a client can become dehydrated because of dye excretion. The second intervention would be to increase the clients fluid status. Neurologic changes would take priority.

A nurse assesses a client with diabetes mellitus. Which clinical manifestation should alert the nurse to decreased kidney function in this client?

a. Urine specific gravity of 1.033 *b.Presence of protein in the urine* c. Elevated capillary blood glucose level d.Presence of ketone bodies in the urine ANS: B Renal dysfunction often occurs in the client with diabetes. Proteinuria is a result of renal dysfunction. Specific gravity is elevated with dehydration. Elevated capillary blood glucose levels and ketones in the urine are consistent with diabetes mellitus but are not specific to renal function

A nurse teaches a young female client who is prescribed amoxicillin (Amoxil) for a urinary tract infection. Which statement should the nurse include in this clients teaching?

a. Use a second form of birth control while on this medication. b. You will experience increased menstrual bleeding while on this drug. c. You may experience an irregular heartbeat while on this drug. d. Watch for blood in your urine while taking this medication. ANS: A The client should use a second form of birth control because penicillin seems to reduce the effectiveness of estrogen-containing contraceptives. She should not experience increased menstrual bleeding, an irregular heartbeat, or blood in her urine while taking the medication.

A nurse teaches a client recovering from a heart transplant who is prescribed cyclosporine (Sandimmune). Which statement should the nurse include in this clients discharge teaching?

a. Use a soft-bristled toothbrush and avoid flossing. b. Avoid large crowds and people who are sick. c. Change positions slowly to avoid hypotension. d. Check your heart rate before taking the medication. ANS: B These agents cause immune suppression, leaving the client more vulnerable to infection. The medication does not place the client at risk for bleeding, orthostatic hypotension, or a change in heart rate.

A nurse teaches a client who is starting urinary bladder training. Which statement should the nurse include in this clients teaching?

a. Use the toilet when you first feel the urge, rather than at specific intervals. b. Try to consciously hold your urine until the scheduled toileting time. c. Initially try to use the toilet at least every half hour for the first 24 hours. d. The toileting interval can be increased once you have been continent for a week. ANS: B The client should try to hold the urine consciously until the next scheduled toileting time. Toileting should occur at specific intervals during the training. The toileting interval should be no less than every hour. The interval can be increased once the client becomes comfortable with the interval.

A nurse assesses a clients electrocardiogram (ECG) and observes the reading shown below: How should the nurse document this clients ECG strip?

a. Ventricular tachycardia b. Ventricular fibrillation c. Sinus rhythm with premature atrial contractions (PACs) d. Sinus rhythm with premature ventricular contractions (PVCs) ANS: D Sinus rhythm with PVCs has an underlying regular sinus rhythm with ventricular depolarization that sometimes precede atrial depolarization. Ventricular tachycardia and ventricular fibrillation rhythms would not have sinus beats present. Premature atrial contractions are atrial contractions initiated from another region of the atria before the sinus node initiates atrial depolarization.

A nurse assesses a client who has a history of migraines. Which clinical manifestation should the nurse identify as an early sign of a migraine with aura?

a. Vertigo b. Lethargy *c. Visual disturbances* d. Numbness of the tongue ANS: C Early warning of impending migraine with aura usually consists of visual changes, flashing lights, or diplopia. The other manifestations are not associated with an impending migraine with aura.

A nurse teaches a client with heart failure about energy conservation. Which statement should the nurse include in this clients teaching?

a. Walk until you become short of breath, and then walk back home. b. Gather everything you need for a chore before you begin. c. Pull rather than push or carry items heavier than 5 pounds. d. Take a walk after dinner every day to build up your strength. ANS: B A client who has heart failure should be taught to conserve energy. Gathering all supplies needed for a chore at one time decreases the amount of energy needed. The client should not walk until becoming short of breath because he or she may not make it back home. Pushing a cart takes less energy than pulling or lifting. Although walking after dinner may help the client, the nurse should teach the client to complete activities when he or she has the most energy. This is usually in the morning.

A nurse teaches a client about self-monitoring of blood glucose levels. Which statement should the nurse include in this clients teaching to prevent bloodborne infections?

a. Wash your hands after completing each test. *b.Do not share your monitoring equipment*. c. Blot excess blood from the strip with a cotton ball. d.Use gloves when monitoring your blood glucose. ANS: B Small particles of blood can adhere to the monitoring device, and infection can be transported from one user to another. Hepatitis B in particular can survive in a dried state for about a week. The client should be taught to avoid sharing any equipment, including the lancet holder. The client should be taught to wash his or her hands before testing. The client would not need to blot excess blood away from the strip or wear gloves.

A nurse assesses a client who is recovering from a lumbar puncture (LP). Which complication of this procedure should alert the nurse to urgently contact the health care provider?

a. Weak pedal pulses *b. Nausea and vomiting* c. Increased thirst d. Hives on the chest ANS: B The nurse should immediately contact the provider if the client experiences a severe headache, nausea, vomiting, photophobia, or a change in level of consciousness after an LP, which are all signs of increased intracranial pressure. Weak pedal pulses, increased thirst, and hives are not complications of an LP.

A nurse teaches a client who is at risk for mild hypernatremia. Which statement should the nurse include in this clients teaching?

a. Weigh yourself every morning and every night. b. Check your radial pulse twice a day. c. Read food labels to determine sodium content. d. Bake or grill the meat rather than frying it. ANS: C Most prepackaged foods have a high sodium content. Teaching clients how to read labels and calculate the sodium content of food can help them adhere to prescribed sodium restrictions and can prevent hypernatremia. Daily self-weighing and pulse checking are methods of identifying manifestations of hypernatremia, but they do not prevent it. The addition of substances during cooking, not the method of cooking, increases the sodium content of a meal.

A nurse cares for a client with right-sided heart failure. The client asks, Why do I need to weigh myself every day? How should the nurse respond?

a. Weight is the best indication that you are gaining or losing fluid. b. Daily weights will help us make sure that youre eating properly. c. The hospital requires that all inpatients be weighed daily. d. You need to lose weight to decrease the incidence of heart failure. ANS: A Daily weights are needed to document fluid retention or fluid loss. One liter of fluid equals 2.2 pounds. The other responses do not address the importance of monitoring fluid retention or loss.

A nurse teaches a client with diabetes mellitus about sick day management. Which statement should the nurse include in this clients teaching?

a. When ill, avoid eating or drinking to reduce vomiting and diarrhea. *b.Monitor your blood glucose levels at least every 4 hours while sick*. c. If vomiting, do not use insulin or take your oral antidiabetic agent. d.Try to continue your prescribed exercise regimen even if you are sick. ANS: B When ill, the client should monitor his or her blood glucose at least every 4 hours. The client should continue taking the medication regimen while ill. The client should continue to eat and drink as tolerated but should not exercise while sick.

A nurse cares for a client who has a chest tube. When would this client be at highest risk for developing a pneumothorax?

a. When the insertion site becomes red and warm to the touch b. When the tube drainage decreases and becomes sanguineous c. When the client experiences pain at the insertion site d. When the tube becomes disconnected from the drainage system ANS: D Intrathoracic pressures are less than atmospheric pressures; therefore, if the chest tube becomes disconnected from the drainage system, air can be sucked into the pleural space and cause a pneumothorax. A red, warm, and painful insertion site does not increase the clients risk for a pneumothorax. Tube drainage should decrease and become serous as the client heals. Sanguineous drainage is a sign of bleeding but does not increase the clients risk for a pneumothorax.

A nurse assesses a client who is prescribed 5-fluorouracil (5-FU) chemotherapy intravenously for the treatment of colon cancer. Which assessment finding should alert the nurse to contact the health care provider?

a. White blood cell (WBC) count of 1500/mm3 b. Fatigue c. Nausea and diarrhea d. Mucositis and oral ulcers ANS: A Common side effects of 5-FU include fatigue, leukopenia, diarrhea, mucositis and mouth ulcers, and peripheral neuropathy. However, the clients WBC count is very low (normal range is 5000 to 10,000/mm3), so the provider should be notified. He or she may want to delay chemotherapy by a day or two. Certainly the client is at high risk for infection. The other assessment findings are consistent with common side effects of 5-FU that would not need to be reported immediately.

A nurse cares for a client with end-stage heart failure who is awaiting a transplant. The client appears depressed and states, I know a transplant is my last chance, but I dont want to become a vegetable. How should the nurse respond?

a. Would you like to speak with a priest or chaplain? b. I will arrange for a psychiatrist to speak with you. c. Do you want to come off the transplant list? d. Would you like information about advance directives? ANS: D The client is verbalizing a real concern or fear about negative outcomes of the surgery. This anxiety itself can have a negative effect on the outcome of the surgery because of sympathetic stimulation. The best action is to allow the client to verbalize the concern and work toward a positive outcome without making the client feel as though he or she is crazy. The client needs to feel that he or she has some control over the future. The nurse personally provides care to address the clients concerns instead of pushing the clients issues off on a chaplain or psychiatrist. The nurse should not jump to conclusions and suggest taking the client off the transplant list, which is the best treatment option.

A nurse cares for an older adult client with heart failure. The client states, I dont know what to do. I dont want to be a burden to my daughter, but I cant do it alone. Maybe I should die. How should the nurse respond?

a. Would you like to talk more about this? b. You are lucky to have such a devoted daughter. c. It is normal to feel as though you are a burden. d. Would you like to meet with the chaplain? ANS: A Depression can occur in clients with heart failure, especially older adults. Having the client talk about his or her feelings will help the nurse focus on the actual problem. Open-ended statements allow the client to respond safely and honestly. The other options minimize the clients concerns and do not allow the nurse to obtain more information to provide client-centered care.

The nurse is caring for a client who is prescribed a long-acting beta2 agonist. The client states, The medication is too expensive to use every day. I only use my inhaler when I have an attack. How should the nurse respond?

a. You are using the inhaler incorrectly. This medication should be taken daily. b. If you decrease environmental stimuli, it will be okay for you to use the inhaler only for asthma attacks. c. Tell me more about your fears related to feelings of breathlessness. d. It is important to use this type of inhaler every day. Lets identify potential community services to help you. ANS: D Long-acting beta2 agonists should be used every day to prevent asthma attacks. This medication should not be taken when an attack starts. Asthma medications can be expensive. Telling the client that he or she is using the inhaler incorrectly does not address the clients financial situation, which is the main issue here. Clients with limited incomes should be provided with community resources. Asking the client about fears related to breathlessness does not address the clients immediate concerns.

A nurse teaches a client who is recovering from a colon resection. Which statement should the nurse include in this clients plan of care?

a. You may experience nausea and vomiting for the first few weeks. b. Carbonated beverages can help decrease acid reflux from anastomosis sites. c. Take a stool softener to promote softer stools for ease of defecation. d. You may return to your normal workout schedule, including weight lifting. ANS: C Clients recovering from a colon resection should take a stool softener as prescribed to keep stools a soft consistency for ease of passage. Nausea and vomiting are symptoms of intestinal obstruction and perforation and should be reported to the provider immediately. The client should be advised to avoid gas-producing foods and carbonated beverages, and avoid lifting heavy objects or straining on defecation.

A nurse teaches a client with functional urinary incontinence. Which statement should the nurse include in this clients teaching?

a. You must clean around your catheter daily with soap and water. b. Wash the vaginal weights with a 10% bleach solution after each use. c. Operations to repair your bladder are available, and you can consider these. d. Buy slacks with elastic waistbands that are easy to pull down. ANS: D Functional urinary incontinence occurs as the result of problems not related to the clients bladder, such as trouble ambulating or difficulty accessing the toilet. One goal is that the client will be able to manage his or her clothing independently. Elastic waistband slacks that are easy to pull down can help the client get on the toilet in time to void. The other instructions do not relate to functional urinary incontinence.

.A nurse cares for a client with diabetes mellitus who asks, Why do I need to administer more than one injection of insulin each day? How should the nurse respond?

a. You need to start with multiple injections until you become more proficient at self-injection. *b.A single dose of insulin each day would not match your blood insulin levels and your food intake patterns*. c. A regimen of a single dose of insulin injected each day would require that you eat fewer carbohydrates. d.A single dose of insulin would be too large to be absorbed, predictably putting you at risk for insulin shock. ANS: B Even when a single injection of insulin contains a combined dose of different-acting insulin types, the timing of the actions and the timing of food intake may not match well enough to prevent wide variations in blood glucose levels. One dose of insulin would not be appropriate even if the client decreased carbohydrate intake. Additional injections are not required to allow the client practice with injections, nor will one dose increase the clients risk of insulin shock.

A nurse cares for a client with amyotrophic lateral sclerosis (ALS). The client states, I do not want to be placed on a mechanical ventilator. How should the nurse respond?

a. You should discuss this with your family and health care provider. b. Why are you afraid of being placed on a breathing machine? c. Using the incentive spirometer each hour will delay the need for a ventilator. *d. What would you like to be done if you begin to have difficulty breathing?* ANS: D ALS is an adult-onset upper and lower motor neuron disease characterized by progressive weakness, muscle wasting, and spasticity, eventually leading to paralysis. Once muscles of breathing are involved, the client must indicate in the advance directive what is to be done when breathing is no longer possible without intervention. The other statements do not address the clients needs

A nurse teaches a client who is prescribed a central vascular access device. Which statement should the nurse include in this clients teaching?

a. You will need to wear a sling on your arm while the device is in place. b.There is no risk of infection because sterile technique will be used during insertion. c. Ask all providers to vigorously clean the connections prior to accessing the device. d.You will not be able to take a bath with this vascular access device. ANS: C Clients should be actively engaged in the prevention of catheter-related bloodstream infections and taught to remind all providers to perform hand hygiene and vigorously clean connections prior to accessing the device. The other statements are incorrect

A nurse cares for a client who tests positive for alpha1-antitrypsin (AAT) deficiency. The client asks, What does this mean? How should the nurse respond?

a. Your children will be at high risk for the development of chronic obstructive pulmonary disease. b. I will contact a genetic counselor to discuss your condition. c. Your risk for chronic obstructive pulmonary disease is higher, especially if you smoke. d. This is a recessive gene and should have no impact on your health. ANS: C The gene for AAT is a recessive gene. Clients with only one allele produce enough AAT to prevent chronic obstructive pulmonary disease (COPD) unless the client smokes. A client with two alleles is at high risk for COPD even if not exposed to smoke or other irritants. The client is a carrier, and children may or may not be at high risk depending on the partners AAT levels. Contacting a genetic counselor may be helpful but does not address the clients current question.

A nurse prepares a client for a colonoscopy scheduled for tomorrow. The client states, My doctor told me that the fecal occult blood test was negative for colon cancer. I dont think I need the colonoscopy and would like to cancel it. How should the nurse respond?

a. Your doctor should not have given you that information prior to the colonoscopy. b. The colonoscopy is required due to the high percentage of false negatives with the blood test. c. A negative fecal occult blood test does not rule out the possibility of colon cancer. d. I will contact your doctor so that you can discuss your concerns about the procedure. ANS: C A negative result from a fecal occult blood test does not completely rule out the possibility of colon cancer. To determine whether the client has colon cancer, a colonoscopy should be performed so the entire colon can be visualized and a tissue sample taken for biopsy. The client may want to speak with the provider, but the nurse should address the clients concerns prior to contacting the provider.

A nurse cares for a client with a new ileostomy. The client states, I dont think my friends will accept me with this ostomy. How should the nurse respond?

a. Your friends will be happy that you are alive. b. Tell me more about your concerns. c. A therapist can help you resolve your concerns. d. With time you will accept your new body. ANS: B Social anxiety and apprehension are common in clients with a new ileostomy. The nurse should encourage the client to discuss concerns. The nurse should not minimize the clients concerns or provide false reassurance.

A nurse cares for a postmenopausal client who has had two episodes of bacterial urethritis in the last 6 months. The client asks, I never have urinary tract infections. Why is this happening now? How should the nurse respond?

a. Your immune system becomes less effective as you age. b. Low estrogen levels can make the tissue more susceptible to infection. c. You should be more careful with your personal hygiene in this area. d. It is likely that you have an untreated sexually transmitted disease. ANS: B Low estrogen levels decrease moisture and secretions in the perineal area and cause other tissue changes, predisposing it to the development of infection. Urethritis is most common in postmenopausal women for this reason. Although immune function does decrease with aging and sexually transmitted diseases are a known cause of urethritis, the most likely reason in this client is low estrogen levels. Personal hygiene usually does not contribute to this disease process.

A nurse assesses a client who has diabetes mellitus. Which arterial blood gas values should the nurse identify as potential ketoacidosis in this client?

a. pH 7.38, HCO3 22 mEq/L, PCO2 38 mm Hg, PO2 98 mm Hg *b.pH 7.28, HCO3 18 mEq/L, PCO2 28 mm Hg, PO2 98 mm Hg* c. pH 7.48, HCO3 28 mEq/L, PCO2 38 mm Hg, PO2 98 mm Hg d.pH 7.32, HCO3 22 mEq/L, PCO2 58 mm Hg, PO2 88 mm Hg ANS: B When the lungs can no longer offset acidosis, the pH decreases to below normal. A client who has diabetic ketoacidosis would present with arterial blood gas values that show primary metabolic acidosis with decreased bicarbonate levels and a compensatory respiratory alkalosis with decreased carbon dioxide levels.


Ensembles d'études connexes

Post-Classical Era Review and Study Guide

View Set

understanding Business - chapter 12

View Set

Origins and Insetions (Extensor Indicis)

View Set

Chapter 59 Assessment and Management of Patients With Male Reproductive Disorders

View Set